You are on page 1of 366

Sumário

Aula 1: Integrais Duplas 11


Um Método Prático para Calcular Integrais Duplas . . . . 13
Cálculo de Integrais Duplas em Regiões mais Gerais . . . 14
Exercı́cios . . . . . . . . . . . . . . . . . . . . . . . . . 19

Aula 2: Mudança de Variáveis na Integral Dupla 21


Integrais Duplas em Coordenadas Polares . . . . . . . . . 23
Exercı́cios . . . . . . . . . . . . . . . . . . . . . . . . . 27

Aula 3: Aplicações das Integrais Duplas 29


Simetria em Integral Dupla . . . . . . . . . . . . . . . . 34
Exercı́cios . . . . . . . . . . . . . . . . . . . . . . . . . 38

Aula 4: Integrais Triplas 41


Redução do Cálculo de uma Integral Tripla a uma Integral
Dupla . . . . . . . . . . . . . . . . . . . . . . . . . . . . 42
Exercı́cios . . . . . . . . . . . . . . . . . . . . . . . . . 48

Aula 5: Mudança de Variáveis na Integral Tripla 49


Um caso particular de mudança de variáveis . . . . . . . 49
Integral Tripla em Coordenadas Esféricas . . . . . . . . . 52
Exercı́cios . . . . . . . . . . . . . . . . . . . . . . . . . 55

Aula 6: Curvas Parametrizadas 57


Integral de Linha de Campo Escalar . . . . . . . . . . . . 61

7
Exercı́cios . . . . . . . . . . . . . . . . . . . . . . . . . 67

Aula 7: Aplicações da Integral de Linha de Campo Escalar 69


Interpretação Geométrica no Plano . . . . . . . . . . . . 69
Interpretação Fı́sica . . . . . . . . . . . . . . . . . . . . 70
Campos Vetoriais . . . . . . . . . . . . . . . . . . . . . 74
Exercı́cios . . . . . . . . . . . . . . . . . . . . . . . . . 79

Aula 8: Preparação para a AP1 81


Exercı́cios . . . . . . . . . . . . . . . . . . . . . . . . . 81

Aula 9: Integral de Linha de Campo Vetorial 83


Integral de Linha de Campo Vetorial . . . . . . . . . . . 83
Campos Conservativos . . . . . . . . . . . . . . . . . . . 87
Cálculo de Funções Potenciais . . . . . . . . . . . . . . . 90
Exercı́cios . . . . . . . . . . . . . . . . . . . . . . . . . 92

Aula 10: Teorema de Green 95


O Teorema de Green . . . . . . . . . . . . . . . . . . . . 95
Teorema das Quatro Equivalências . . . . . . . . . . . . 101
Exercı́cios . . . . . . . . . . . . . . . . . . . . . . . . . 107

Aula 11: Superfı́cies Parametrizadas 109


Superfı́cies Parametrizadas . . . . . . . . . . . . . . . . . 109
Área de Superfı́cie . . . . . . . . . . . . . . . . . . . . . 114
Exercı́cios . . . . . . . . . . . . . . . . . . . . . . . . . 118

Aula 12: Integral de Superfı́cie de um Campo Escalar 121


Integral de Superfı́cie de um Campo Escalar . . . . . . . 121
Aplicações à Fı́sica . . . . . . . . . . . . . . . . . . . . . 123
Exercı́cios . . . . . . . . . . . . . . . . . . . . . . . . . 127

Aula 13: Integral de Superfı́cie de um Campo Vetorial 129

8
Integral de Superfı́cie de um Campo Vetorial . . . . . . . 129
Exercı́cios . . . . . . . . . . . . . . . . . . . . . . . . . 137

Aula 14: Teorema de Gauss 139


O Teorema de Gauss . . . . . . . . . . . . . . . . . . . . 139
Exercı́cios . . . . . . . . . . . . . . . . . . . . . . . . . 145

Aula 15: Teorema de Stokes 147


Exercı́cios . . . . . . . . . . . . . . . . . . . . . . . . . 153

Aula 16: Preparação para a AP2 155


Exercı́cios . . . . . . . . . . . . . . . . . . . . . . . . . 155

Solução dos Exercı́cios 157


.
Cálculo IV – Aula 1

Aula 1
Integrais Duplas
Objetivos
• Compreender a noção de integral dupla;
• estudar algumas propriedades;
• estudar o Teorema de Fubini para retângulos;
• estudar uma versão mais geral do Teorema de Fubini;
• calcular área e volume.

No Cálculo II, você aprendeu as integrais definidas. Agora, no Cálculo IV, pre-
tendemos estender essa idéia para integrais duplas e triplas de funções de duas
ou três variáveis.
Então consideremos uma função f : D ⊂ R2 → R, onde D é um conjunto
fechado e limitado (também conhecido como conjunto compacto). Como D é
limitado, então existe um retângulo R = [a, b] × [c, d], tal que D ⊂ R.

d = yn
R
Rij
D f
yj
∆y
yj−1

(x∗i , yj∗ )
y0 = c R
a = x0 xi−1 xi b = xn x
∆x

Vamos dividir o retângulo R em subretângulos Rij da seguinte maneira: dividimos


b−a
os intervalos [a, b] e [c, d] em n subintervalos de mesmo comprimento ∆x =
n
d−c
e ∆y = , respectivamente; traçamos retas verticais e horizontais pelas
n 
extremidades desses subintervalos. Vamos escolher x∗i , yj∗ ∈ Rij e formemos a
soma n X n n
X  X 
Sn = f x∗i , yj∗ ∆x∆y = f x∗i , yj∗ ∆A
j=1 i=1 i,j=1
 
onde f x∗i , yj∗ = 0 se x∗i , yj∗ ∈
/ D.

11
Integrais Duplas

Esta soma é dita soma de Riemann de f . Se existir o lim Sn = L, dizemos


n→∞
que
ZZ f é integrável e o
ZZ número L é dito
ZZintegral de f sobre D e é indicado por
f (x, y) dxdy ou f (x, y) dA ou f dA. Assim,
D D D

ZZ n
X 
f (x, y) dxdy = lim f x∗i , yj∗ ∆x∆y .
D n→∞
i,j=1
OBS.:

1. Prova-se que se f é contı́nua em D, então f é integrável.

2. Se f (x, y) ≥ 0 é contı́nua em D, então o gráfico de f (Gf ) está


acima do plano xy. Então o volume do sólido W que está abaixo
de Gf e acima de D é dado por
ZZ
V (W ) = f (x, y) dxdy .
D

Logo, para achar o volume do sólido W , integramos f (x, y)


(o “teto”) sobre D (o “piso”).

Gf : z = f (x, y) (“teto”)

(“piso”)
x
(x∗i , yj∗ ) Rij

3. Se f (x, y) = 1 em D então
ZZ ZZ
1 dxdy = dxdy = A(D) = área de D .
D D

12
Cálculo IV – Aula 1

4. Propriedades
ZZ ZZ ZZ
(i) (f + g) dA = f dA+ g dA
D D D
ZZ ZZ
(ii) kf dA = k f dA, k ∈ R
D D
ZZ ZZ ZZ
(iii) D = D1 ∪ D2 ⇒ f dA = f dA+ f dA
D D1 D2

D1
D2

Um Método Prático para Calcular Integrais Duplas

Teorema de Fubini: Se f (x, y) é contı́nua no retângulo D = [a, b]×[c, d],


então
ZZ Z b Z d  Z d Z b 
f (x, y) dxdy = f (x, y) dy dx = f (x, y) dx dy
D a c c a

ou
ZZ Z bZ d Z dZ b
f (x, y) dxdy = f (x, y) dydx = f (x, y) dxdy
D a c c a
| {z }
integrais iteradas ou repetidas

Exemplo 1
ZZ
Calcule xy 2 dxdy, sendo D = [0, 1] × [−1, 0].
D

Solução:
Temos ZZ Z 1Z 0
2
xy dxdy = xy 2 dydx .
D 0 −1

Primeiro calculamos a integral interna. Logo:


ZZ Z 1 h 3 i0 Z 1 Z 1
1 x2 1
h i
2 y 1 1 1
xy dxdy = x = x[0−(−1)] dx = x dx = = .
D 0 3 −1 3 0 3 0 3 2 0 6

13
Integrais Duplas

Cálculo de Integrais Duplas em Regiões mais Gerais

Suponhamos agora, que D seja diferente do retângulo [a, b]×[c, d]. Então vamos
definir dois tipos de região.

Definição 1

Dizemos que D é uma região do tipo I ou uma região simples vertical


se D for limitada à esquerda pela reta vertical x = a, à direita pela
reta vertical x = b, inferiormente pela curva de equação y = g1 (x) e
superiormente pela curva y = g2 (x), onde g1 e g2 são contı́nuas.

As figuras que se seguem ilustram regiões do tipo I:


y = g2 (x) y = g2 (x)
y y
D D

(x, y) (x, y)

y = g1 (x) y = g1 (x)

a x b x a x b x

y = g2 (x)
y

D
(x, y)

y = g1 (x)
a x b x

Logo, D = (x, y) ∈ R2 | a ≤ x ≤ b e g1 (x) ≤ y ≤ g2 (x) . Prova-se que:
ZZ Z bZ g2 (x)
f (x, y) dxdy = f (x, y) dydx .
D a g1 (x)

Definição 2

Dizemos que D é uma região do tipo II ou uma região simples horizontal


se D for limitada inferiormente e superiormente por retas horizontais
y = c e y = d, respectivamente, pela esquerda pela curva x = h1 (y) e
pela direita pela curva x = h2 (y), onde h1 e h2 são contı́nuas.

14
Cálculo IV – Aula 1

As figuras que se seguem ilustram regiões do tipo II:

y y

d d
x = h1 (x) D x = h1 (x)
x = h2 (x) D x = h2 (x)
x x

c c
x x

d
x = h1 (x)
D
x = h2 (x)
x

c
x


Logo, D = (x, y) ∈ R2 | c ≤ y ≤ d e h1 (y) ≤ x ≤ h2 (y) . Prova-se que:

ZZ Z dZ h2 (y)
f (x, y) dxdy = f (x, y) dxdy .
D c h1 (y)

Exemplo 1
Calcule por dois métodos a integral de f (x, y) = xy sobre a região D limitada
pelas curvas y = x e y = x2 .
Solução:
As curvas se interceptam quando x2 = x ou x(x − 1) = 0, logo x = 0 ou x = 1.
Assim, os pontos de interseção são (0, 0) e (1, 1). Logo, o esboço de D está
representado na figura que se segue.

15
Integrais Duplas

y=x

(x, y)
D y = x2

x 1 x

Método 1

Enquadrando D como tipo I, temos D = (x, y) ∈ R2 | 0 ≤ x ≤ 1 e x2 ≤ y ≤ x .
Então:
ZZ Z 1Z x Z 1 h ix
y2
xy dxdy = xy dydx = x dx
D 0 x2 0 2 x2
Z 1
1 
= x x2 − x4 dx
2 0
Z 1
1 
= x3 − x5 dx
2 0
h i1
1 x4 x6
= −
2 4 6 0
 
1 1 1
= −
6 4 6
1
= .
24

Método 2

x=y

y

D x= y

1 x

16
Cálculo IV – Aula 1

 √
Enquadrando D como tipo II, temos D = (x, y) ∈ R2 | 0 ≤ y ≤ 1 e y ≤ x ≤ y .
Então:
ZZ Z 1Z √y Z 1  2 √y Z 1
x 1 
xy dxdy = xy dxdy = y dy = y y − y 2 dy
D 0 y 0 2 y 2 0
Z 1
1 
= y 2 − y 3 dy
2 0
h i1
1 y3 y4
= −
2 3 4 0
 
1 1 1
= −
2 3 4
1
= .
24

Exemplo 2
Calcule, por integral dupla, a área da região plana D limitada pelas curvas y = x3

e y = x.
Solução:
O esboço de D está representado na figura que se segue.
y
y = x1/2 y = x3

1 y= x = x1/2

D y = x3

1 x

Podemos descrever por 


 0≤x≤1
D:
 x3 ≤ y ≤ x1/2

Então:
ZZ Z 1Z x1/2 Z 1
x4 1
h i
 2 3/2
A(D) = dxdy = dydx = x1/2 − x3 dx = x −
D 0 x3 0 3 4 0

2 1
= −
3 4
5
= u.a.
12

17
Integrais Duplas

Exemplo 3
Calcule o volume do tetraedo W com faces nos planos coordenados e no plano
x + y + z = 3.
Solução:
O plano x + y + z = 3 passa pelos pontos A = (3, 0, 0), B = (0, 3, 0) e
C = (0, 0, 3). Assim, o esboço de W é:

z y

C
3
teto de W x+y =3

y =3−x

W D

B
3 x
A y y=0
x D (piso)

Observemos que o teto de W é a porção do plano x + y + z = 3 ou


z = 3 − x − y = f (x, y) e o piso de W é o triângulo D. Então:
ZZ ZZ
V (W ) = f (x, y) dxdy = (3 − x − y) dxdy
D D
Z 3 Z 3−x
= (3 − x − y) dydx
0 0
Z 3 h i3−x
y2
= 3y − xy − dx
0 2 0
Z 3 h i
(3 − x)2
= 3(3 − x) − x(3 − x) − dx
0 2
Z 3
1
= (9 − 6x + x2 ) dx
2 0
h i3
1 x3
= 9x − 3x2 +
2 3 0

9
= u.v.
2

18
Cálculo IV – Aula 1

Exercı́cios
ZZ
2
1. Calcule I = yex+y dxdy, sendo D = [−1, 1] × [0, 1].
D

2. Determine a região D e troque a ordem de integração das seguintes inte-


grais:
Z 1Z √ Z 1Z √
x 1− y
a) I = f (x, y) dydx. b) I = √ f (x, y) dxdy.
0 x3 0 − 1−y 2

3. A integral abaixo não pode ser calculada exatamente, em termos de funções


elementares, com a ordem de integração dada. Inverta a ordem de inte-
gração e faça os cálculos em
Z 1Z 1
2
I= ex dxdy .
0 y

4. Utilizando integral dupla, calcule a área da região D limitada pelas curvas


y 2 = −x, x − y = 4, y = −1 e y = 2.
ZZ
y √
5. Calcule 2
dxdy, onde D é a região limitada por y = 0, y = x
D1+x
e x = 4.

6. Inverta a ordem de integração e calcule seu valor em


Z 1Z √y Z 4Z 1
x3 x3
I= √
e dxdy + √
e dxdy .
0 y/2 1 y/2

7. Encontre por integração dupla a área da região no plano xy, limitada pelas
curvas y = x2 e y = 4x − x2 .

8. Determine a área da região limitada pelas curvas x = y 3 , x + y = 2


e y = 0.

9. Calcule o volume do sólido W , no primeiro octante, limitado pelas


superfı́cies z = 4 − x2 , y = 0, z = 0 e x + y = 2.

10. Calcule o volume do sólido W limitado pelos planos x = 0, z = 0, x+y = 9


e pelo cilindro parabólico z = 9 − y 2 .

19
Integrais Duplas

20
Cálculo IV – Aula 2

Aula 2
Mudança de Variáveis na Integral Dupla
Objetivos
• Aprender a fazer mudança de variáveis em integrais duplas;
• estudar uma mudança de variáveis bastante usada: coordenadas polares.

No Cálculo II, você aprendeu a fórmula da mudança de variável para uma função
de uma variável: Z Z
b d
f (x) dx = f (g(u)) g ′ (u) du .
a c
Para as integrais duplas, temos uma fórmula análoga.
Uma mudança de variáveis num subconjunto do R2 é dada por uma trans-
formação

ϕ : Duv ⊂ R2 −→ R2

(u, v) 7−→ (x, y) = ϕ(u, v) = x(u, v), y(u, v)

de classe C 1 e injetora no interior de Duv .


v y

ϕ f
Duv Dxy = ϕ(Duv )

u x R

Suponhamos que o jacobiano de ϕ, Jϕ(u, v), seja diferente de 0, isto é,



∂x ∂x
∂(x, y) ∂u ∂v

J = Jϕ(u, v) = = 6= 0 .
∂(u, v) ∂y ∂y

∂u ∂v

Prova-se que dxdy = |J| dudv.


Seja Dxy = ϕ (Duv ). Então se f (x, y) é contı́nua em Dxy , temos:
ZZ ZZ

f (x, y) dxdy = f x(u, v), y(u, v) |J| dudv .
Dxy Duv

21
Mudança de Variáveis na Integral Dupla

OBS.: Pelo teorema da função inversa, o jacobiano de ϕ−1 é dado


por

∂u ∂u

∂x ∂y  1
−1
Jϕ (x, y) = = Jϕ(u, v) −1 = .
∂v ∂v
J (ϕ(u, v))
∂x ∂y

Exemplo 1
Calcule,
ZZ utilizando uma mudança de variáveis conveniente, a integral
(x + y)6
dxdy, sendo Dxy a região limitada pelas retas y + x = 3, y + x = 5,
Dxy y−x
y − x = 1 e y − x = 3.
Solução:
O esboço de Dxy está representado na figura a seguir.
y

3 Dxy

3 5 x

Façamos u = x + y, v = y − x, que nos dá


 
u−v
 u + v = 2y  x=

2
ou
 u − v = 2x  y = u+v

2
Temos:

∂x ∂x 1 1
∂(x, y) ∂u −
∂v 2 2 1
J= = = = 6= 0 .
∂(u, v) ∂y ∂y 2
1 1


∂u 2 ∂v 2
1
Como dxdy = |J| dudv, temos dxdy = dudv.
2
A seguir, vamos determinar Duv .
Como Dxy é limitado por y + x = 3, y + x = 5, y − x = 1 e y − x = 3, então
Duv é limitado por u = 3, u = 5, v = 1 e v = 3.

22
Cálculo IV – Aula 2

v
3

Duv
1

3 5 u

Segue da fórmula da mudança de variáveis que:


ZZ ZZ ZZ
(x + y)6 u6 1 1 u6
dxdy = · dudv = dudv
Dxy y − x Duv v 2 2 Duv v
Z 5 Z 3
1 6 1
= u dvdu
2 3 1 v
Z 5 h i3
1
= u6 ln v du
2 3 1
Z 5
ln 3
= u6 du
2 3
h i5
ln 3 u7
=
2 7 3

ln 3
= (57 − 37 ) .
14

Integrais Duplas em Coordenadas Polares

No Cálculo II, você aprendeu coordenadas polares (r, θ), onde r é a distância
de um ponto P = (x, y) à origem e θ é o ângulo (em radianos) formado pelo
eixo x positivo e o raio polar OP .

P (x, y)

r
y
θ
O x x

Da figura, vemos que x = r cos θ, y = r sen θ donde x2 + y 2 = r2 .

23
Mudança de Variáveis na Integral Dupla

Então consideremos a mudança de variáveis dada por


(
x = r cos θ
ϕ:
y = r sen θ

onde r ≥ 0 e θ0 ≤ θ ≤ θ0 + 2π, para algum θ0 ∈ R.


O jacobiano de ϕ é dado por

∂x ∂x
∂(x, y)

∂r

∂θ cos θ −r sen θ
J = Jϕ = = = = r cos2 θ+r sen2 θ = r .
∂(r, θ) ∂y ∂y

sen θ r cos θ
∂r ∂θ

Então ZZ ZZ
f (x, y) dxdy = f (r cos θ, r sen θ) r drdθ .
D Drθ

OBS.:

1. O termo dxdy não é substituı́do por drdθ, mas por rdrdθ.

2. A área de D em coordenadas polares é dada por


ZZ
A(D) = r drdθ .
Drθ

Exemplo 1
ZZ
2 +y 2 √
Calcule ex dxdy, onde D é a região limitada pela curva y = 1 − x2
D
e o eixo x.
Solução:
O esboço de D está representado na figura que se segue.

y
1

r=1
D (r, θ)
θ
−1 1 x
r=0

24
Cálculo IV – Aula 2

Passando para coordenadas polares, temos:





 x= r cos θ

y= r sen θ


 dxdy = rdrdθ
x + y2 =
2
r2

Observemos que em D o ângulo θ varia de 0 (no eixo polar = eixo x positivo)


a π (no ponto (−1, 0)). Fixado θ, tal que 0 ≤ θ ≤ π, o raio polar r varia de 0
a 1. Então, Drθ é dado por:
(
0≤θ≤π
Drθ : .
0≤r≤1

Logo,
ZZ ZZ Z 1Z π Z 1
x2 +y 2 r2 r2 2
e dxdy = e r drdθ = e r dθdr = π er r dr .
D Drθ 0 0 0

1
Temos d(r2 ) = 2r dr, donde r dr = d(r2 ). Então,
2
ZZ Z 1 h i1
x2 +y 2 π r2 2 π r2 π
e dxdy = e d(r ) = e = (e − 1) .
D 2 0 2 0 2

Exemplo 2
ZZ
Calcule I = y dxdy, onde D é limitado por x2 + y 2 = 2y. Solução:
D

Completando quadrado em x2 + y 2 = 2y, temos x2 + (y − 1)2 = 1. Logo,


temos uma circunferência de centro (0, 1) e raio 1. Assim, o esboço de D está
representado na figura que se segue.

y
2

1 D

Calcular I, enquadrando D como tipo I ou tipo II, é uma tarefa difı́cil (verifique!),

25
Mudança de Variáveis na Integral Dupla

então passemos para coordenadas polares, temos:





 x = r cos θ

y = r sen θ


 dxdy = rdrdθ
 2
x + y 2 = r2

Passando x2 + y 2 = 2y para coordenadas polares, temos r2 = 2r sen θ ou


r = 2 sen θ. Observemos que como o eixo x é tangente à circunferência na
origem, então θ varia de 0 a π. Fixando θ, tal que 0 ≤ θ ≤ π, o raio polar r
varia de 0 a 2 sen θ. Logo, o conjunto Drθ é dado por
(
0≤θ≤π
Drθ :
0 ≤ r ≤ 2 sen θ
Então, ZZ
I = r sen θ · r drdθ
Drθ
ZZ
= r2 sen θ drdθ
Drθ
Z π Z 2 sen θ
= sen θ r2 drdθ
0 0
Z π h 3 i2 sen θ
r
= sen θ dθ
0 3 0
Z π
8
= sen4 θ dθ .
3 0
Vale a pena lembrar que
2  1 − cos 2θ 2 1 
• sen4 θ = sen2 θ = = 1 − 2 cos 2θ + cos2 2θ
2 4
Z  
1 sen 2u
• cos2 u du = u+ +C
2 2

Então, Z π
2 
I = 1 − 2 cos 2θ + cos2 2θ dθ
3 0
Z π
2 1 
= · 1 − 2 cos 2θ + cos2 2θ d(2θ)
3 2 0
h   iπ
1 1 sen 4θ
= 2θ − 2 sen 2θ + 2θ +
3 2 2 0
h iπ
1 sen 4θ
= 3θ − 2 sen 2θ +
3 4 0

= π.

26
Cálculo IV – Aula 2

Exercı́cios

1. Considere a transformação do plano uv no plano xy definida por



T (u, v) = (x, y) = u + v, u2 − v . Seja Duv a região do plano uv
limitada pelos eixos u e v e pela reta u + v = 2. Seja Dxy = T (Duv ), a
imagem de Duv por T .

a) Esboce Dxy .
ZZ
1
b) Calcule √ dxdy.
Dxy 1 + 4x + 4y

2. Calcule a seguinte integral dupla


ZZ  
π x−y
cos · dxdy
D 2 x+y

onde D é a região do plano xy, limitada pelas retas x + y = 1, x + y = 4,


x = 0 e y = 0.
x−y
Sugestão: Use a mudança de variáveis u = e v = x + y.
x+y

x2 y2
3. Mostre que a área da região limitada pela elipse 2 + 2 = 1 (a > 0, b > 0)
a b
é igual a πab.

4. Calcule ZZ

I= sen 4x2 + y 2 dxdy ,
D
 2 2
onde D = (x, y) | 4x + y ≤ 1 , y ≥ 0 .

5. Calcule a área da região D do plano xy, definida por



D = (x, y) | x2 + (y − 2)2 ≤ 4 e x2 + y 2 ≥ 4 .

6. Calcule a integral iterada transformando-a, antes, para coordenadas pola-


res:
Z 4Z √4y−y2 p
a) I = x2 + y 2 dxdy
0 0
Z 0 Z 0
2
b) I = √
p dydx
−1 − 1−x2 1+ x2 + y 2
Z 1Z √
1+ 1−y 2 p
c) I = x2 + y 2 dxdy
0 y

27
Mudança de Variáveis na Integral Dupla

ZZ
7. Passe para coordenadas polares e calcule xy dxdy, onde B é a região
B
x2 + y 2 − 2y ≤ 0 , x ≥ 0.

8. Passe para coordenadas polares e calcule


Z 3Z y
1
I= p dxdy
1 0
2 x + y2

9. Achar o volume do sólido limitado superiormente pela esfera de equação


x2 + y 2 + z 2 = 4, inferiormente pelo plano xy e lateralmente pelo cilindro
de equação x2 + y 2 = 1.

10. Determine o volume do sólido W limitado pelas superfı́cies x2 + y 2 = 2y,


p
z = 0 e z = x2 + y 2 .

28
Cálculo IV – Aula 3

Aula 3
Aplicações das Integrais Duplas
Objetivos
• Estudar algumas aplicações fı́sicas como massa, centro de massa e mo-
mento de inércia;
• explorar simetrias em integrais duplas.

Massa

Seja D ⊂ R2 , uma região compacta, representando uma lâmina plana delgada.


Suponhamos que a função contı́nua e positiva δ : D ⊂ R2 → R representa a
densidade superficial de massa (massa por unidade de área).

Rij
R

(x∗i , yj∗ )

Considerando-se n2 subretângulos Rij de algum retângulo R que contém D e



uma escolha x∗i , yj∗ ∈ Rij , observamos que a soma
n X
X n

δ x∗i , yj∗ ∆A
j=1 i=1

 
é uma aproximação da massa M de D, onde δ x∗i , yj∗ = 0 se x∗i , yj∗ ∈/ D.
Logo, é razoável definir a massa M de D com
ZZ
M= δ(x, y) dxdy .
D

OBS.: Se δ(x, y) for constante e igual a k, então a massa


M será igual a kA(D). Neste caso, dizemos que a lâmina
D é homogênea.

29
Aplicações das Integrais Duplas

Centro de Massa

a) Seja um sistema finito de partı́culas P1 = (x1 , y1 ), P2 = (x2 , y2 ), · · · ,


Pn = (xn , yn ), com massas mi , i = 1, · · · , n, respectivamente. Lembrando
da Fı́sica que os momentos de massa desse sistema, em relação aos eixos x e y,
são definidos por:

n
X n
X
Mx = m i y i e My = m i xi .
i=1 i=1

O centro de massa do sistema é o ponto (x, y) que se comporta como se a


n
X
massa total M = mi do sistema estivesse concentrada nesse ponto. Logo,
i=1
M x = M y e M y = M x ou

n
X n
X
mi xi mi yi
My Mx
x= = i=1n e y= i=1
= n .
M X M X
mi mi
i=1 i=1

b) Se considerarmos no lugar de um sistema finito de partı́culas, uma lâmina


plana D com densidade superficial de massa dada por uma função contı́nua
e positiva δ(x, y), fazemos uma partição de algum retângulo R contendo D,

obtendo subretângulos Rij . Escolhemos x∗i , yj∗ ∈ Rij . Logo, a massa de Rij
  
pode ser aproximada por δ x∗i , yj∗ ∆A, onde δ x∗i , yj∗ = 0 se δ x∗i , yj∗ ∈/ D.
Então
n
X n
X
 
Mx ≃ yj∗ δ x∗i , yj∗ ∆A e My ≃ x∗i δ x∗i , yj∗ ∆A .
i,j=1 i,j=1

Logo, definimos Mx e My por


ZZ ZZ
Mx = yδ (x, y) dA e My = xδ (x, y) dA .
D D

O centro de massa (x, y) da lâmina D é definido por


ZZ ZZ
xδ (x, y) dA yδ (x, y) dA
D D
x= e y= .
M M

30
Cálculo IV – Aula 3

OBS.: Se δ(x, y) = k, k constante, o ponto (x, y) é dito


centróide e temos as seguintes fórmulas
ZZ ZZ
x dxdy y dxdy
D D
x= Z Z e y= Z Z .
dxdy dxdy
D D

Momento de Inércia

O momento de inércia de uma lâmina D em relação a um eixo E é dado por

ZZ
IE = r2 (x, y)δ(x, y) dxdy
D

onde r(x, y) é a distância de (x, y) ao eixo E.


Assim, os momentos de inércia de D em relação aos eixos x e y, respectivamente,
são dados por

ZZ ZZ
2
Ix = y δ(x, y) dxdy e Iy = x2 δ(x, y) dxdy .
D D

O momento de inércia polar em relação à origem é dado por

ZZ
I0 = (x2 + y 2 ) δ(x, y) dxdy = Ix + Iy .
D

Exemplo 1
Determine o centro de massa e o momento de inércia em relação ao eixo x, da
região D limitada por x = y 2 e x − y = 2, sendo δ(x, y) = 3.
Solução:
As curvas se interceptam quando y 2 − y − 2 = 0, logo y = −1, y = 2. Assim,
o esboço de D está representado na figura que se segue.

31
Aplicações das Integrais Duplas

2 (4, 2)
2
x=y

x=2+y
D

2 4 x
−1
(1, −1)

−2


Descrevemos D como tipo II : D = (x, y) | −1 ≤ y ≤ 2 , y 2 ≤ x ≤ 2 + y .
A massa de D é:

ZZ ZZ Z 2 Z 2+y
M= δ(x, y) dA = 3 dA = 3 dxdy
D D −1 y2
Z 2
= 3 (2 + y − y 2 ) dy
−1

y3 2
h i
y2
= 3 2y + −
2 3 −1
h   i
8 1 1
= 3 4+2− − −2 + +
3 2 3
27
= .
2

O centro de massa (x, y) é dado por

ZZ ZZ
xδ (x, y) dA yδ (x, y) dA
D D
x= , y= .
M M

32
Cálculo IV – Aula 3

ZZ
Cálculo de xδ(x, y) dA
D

ZZ Z 2 Z 2+y
xδ(x, y) dA = 3 x dxdy
D −1 y2
Z 2 h i2+y
x2
= 3 dy
−1 2 y2
Z 2
3 
= 4 + 4y + y 2 − y 4 dy
2 −1
h i2
3 y3 y5
= 4y + 2y 2 + −
2 3 5 −1
h   i
3 8 32 1 1
= 8+8+ − − −4 + 2 − +
2 3 5 3 5
3 72
= ×
2 5
108
= .
5

ZZ
Cálculo de yδ(x, y) dA
D

ZZ Z 2 Z 2+y
yδ(x, y) dA = 3 y dxdy
D −1 y2
Z 2

= 3 y 2 + y − y 2 dy
−1
Z 2 
= 3 2y + y 2 − y 3 dy
−1

y4 2
h i
y3
= 3 y2 + −
3 4 −1
h   i
8 1 1
= 3 4+ −4 − 1− −
3 3 4
27
= .
4

Logo,
108 27
5 8 4 1
x= 27 = e y= 27 = .
2
5 2
2
 
8 1
Assim, o centro de massa (x, y) está localizado em , .
5 2

33
Aplicações das Integrais Duplas

O momento de inércia em relação ao eixo x é:


ZZ ZZ
2
Ix = y δ(x, y) dA = 3 y 2 dA
D D
Z 2 Z 2+y
= 3 y 2 dxdy
−1 y2
Z 2

= 3 y 2 2 + y − y 2 dy
−1
Z 2 
= 3 2y 2 + y 3 − y 4 dy
−1
h i2
2y 3 y4 y5
= 3 + −
3 4 5 −1
h   i
16 32 2 1 1
= 3 +4− − − + +
3 5 3 4 5
189
= .
20

Simetria em Integral Dupla

a) Seja D ⊂ R2 , simétrica em relaçãoZ Z ao eixo y e f (x, y) ı́mpar na variável x,


isto é, f (−x, y) = −f (x, y). Então, f (x, y) dxdy = 0. Com efeito, como D
D
tem simetria em relação ao eixo y, observamos que D está limitada à direita pela
curva x = x(y) e à esquerda pela curva x = −x(y). Supondo que a projeção de
D sobre o eixo y seja o intervalo [c, d], temos o seguinte esboço para D:
y
d

x = −x(y) x = x(y)
c

Então,
ZZ Z "Z # Z
d x(y) d
f (x, y) dxdy = f (x, y) dx dy = 0 dy = 0 .
D c −x(y) c
| {z }
= 0 (∗)

(∗) Aqui, usamos um fato do Cálculo II:


Z a
g(x) dx = 0 se g(x) é uma função ı́mpar .
−a
34
Cálculo IV – Aula 3

b) Analogamente,Z Zse D tem simetria em relação ao eixo x e f (x, y) é ı́mpar na


variável y, então f (x, y) dxdy = 0. Veja o esboço para D na figura a seguir.
D

y = y(x)
y

a b x

y = −y(x)

Exemplo 1
Calcule ZZ

I= xy 6 + (x4 + y 4 ) sen y + 1 dxdy ,
D

onde D é o disco x + y ≤ a2 , (a > 0).


2 2

Solução:
Por propriedade, temos que
ZZ ZZ ZZ
6 4 4
I= xy dxdy + (x + y ) sen y dxdy + dxdy .
D D D
| {z } | {z } | {z }
I1 I2 I3

• Como f (x, y) = xy 6 é ı́mpar na variável x e D tem simetria em relação


ao eixo y, então I1 = 0.

• Como g(x, y) = (x4 + y 4 ) sen y é ı́mpar na variável y e D tem simetria em


relação ao eixo x, então I2 = 0.
ZZ
• Como dxdy = A(D), então I3 = πa2 . Logo,
D

I = 0 + 0 + πa2 = πa2 .

RECOMENDAÇÃO

Nas integrais duplas, busque as si-


metrias e as funções ı́mpares. Não
calcule cegamente!!!

35
Aplicações das Integrais Duplas

OBS.:

1. Se a densidade δ(x, y) é uma função par na variável x (isto é,


δ(−x, y) = δ(x, y)), então xδ(x, y) é ı́mpar
Z Zna variável x. Se D
tem simetria em relação ao eixo y, então xδ(x, y) dxdy = 0
D
e portanto, x = 0.
Analogamente, se δ(x, y) é uma função par na variável y e se D
tem simetria em relação ao eixo x, então y = 0.

2. Se D é uma lâmina homogênea e tem simetria em relação ao


eixo y, então x = 0.
Analogamente, se D é homogênea e tem simetria em relação ao
eixo x, então y = 0.

Exemplo 2
Uma lâmina delgada D ocupa a região x2 + y 2 ≤ 1, y ≥ 0, de modo que
a densidade em qualquer ponto é proporcional à distância do ponto à origem.
Determine

a) a massa M de D

b) o centro de massa

Solução:
O esboço de D está representado na figura que se segue.

−1 1 x

p
Como a distância de (x, y) à origem é x2 + y 2 então a densidade é dada
p
por δ(x, y) = k x2 + y 2 onde k é uma constante.

36
Cálculo IV – Aula 3

ZZ ZZ p
a) Como M = δ(x, y) dxdy, então M = k x2 + y 2 dxdy. Passando
D D
para coordenadas polares, temos:



 x= r cos θ

y= r sen θ
.


 dxdy = rdrdθ
x2 + y 2 = r2

Além disso, Drθ é dado por:


(
0≤r≤1
Drθ : .
0≤θ≤π
Então, ZZ ZZ
M =k r · r drdθ = k r2 drdθ
Drθ Drθ
Z 1 Z π
2
= k r dθdr
0 0
Z 1
= kπ r2 dr
0


= u.m.
3
b) Como δ(x, y) é uma função par e D tem simetria em relação ao eixo y, então
x = 0. Sabemos que ZZ
yδ(x, y) dxdy
D
y= ,
M
onde ZZ ZZ p
yδ(x, y) dxdy = k y x2 + y 2 dxdy
D D
ZZ
= k r sen θ · r · r drdθ
Drθ
ZZ
= k r3 sen θ drdθ
Drθ
Z 1 Z π
3
= k r sen θ dθdr
0 0
Z 1
 π
= k − cos θ 0
r3 dr
0

 r4 1
= 2k
4 0
k
= .
2

37
Aplicações das Integrais Duplas

Logo,
k
2 3
y= kπ
= .
3

 
3
Portanto, o centro de massa está localizado em 0, .

Exercı́cios

1. Calcule as coordenadas (x, y) do centro de massa de uma chapa homogênea


D com o formato de um triângulo isósceles com base 10 cm e altura 5 cm.

2. Suponha que uma lâmina D tem a forma da região triangular de vértices


(0, 0), (3, 0) e (3, 5) e a densidade δ(x, y) em cada ponto P = (x, y) ∈ D
é igual à distância de P ao eixo y. Calcule a massa total e as coordenadas
(x, y) do centro de massa de D.

3. Determine as coordenadas do centro de massa da região D determinada


por x2 + y 2 ≥ 1, x2 + y 2 ≤ 4 e y ≥ 0, se a densidade de massa em cada
ponto é proporcional à distância do ponto à origem.

4. Uma lâmina plana D tem a forma da região delimitada pelas curvas


y = x2 + 1 e y = x + 3. Sua densidade de massa no ponto (x, y) é
proporcional à distância desse ponto ao eixo x. Calcule

a) a massa de D.
b) a primeira coordenada x do centro de massa de D.

5. Calcule o momento de inércia de um disco circular homogêneo de diâmetro


10 cm.

a) em relação a seu próprio centro.


b) em relação a seu diâmetro.

6. Calcule a massa de uma lâmina delimitada por (x − 1)2 + (y − 2)2 = 1, se


a densidade em um ponto é proporcional à distância desse ponto a (1, 2).

7. Uma lâmina homogênea tem a forma de um triângulo retângulo isósceles


com lados iguais de medida a. Ache o momento de inércia em relação a
um dos lados iguais.

8. Calcule a massa de uma chapa√


D limitada pelas curvas x2 + y 2 = 2x,
x2 + y 2 = 4x, y = x e y = 33 x, sabendo que a densidade de massa em
um ponto é inversamente proporcional à distância do ponto a origem.

38
Cálculo IV – Aula 3

9. Mostre que o momento de inércia em relação ao eixo y de uma placa


homogênea de massa M , limitada por x + y = 4, x = 4, y = 4 é igual
a 8M .

10. Seja uma lâmina delgada representada pela região D determinada por
y ≤ x, y ≥ −x, x2 + y 2 ≥ 2x e x2 + y 2 ≤ 4x. Se a densidade
1
em cada ponto P = (x, y) da lâmina é dada por δ(x, y) = p 2 2 ,
x +y
determine:
a) a massa de D;
b) o momento de inércia polar em relação à origem.

39
Aplicações das Integrais Duplas

40
Cálculo IV – Aula 4

Aula 4
Integrais Triplas
Objetivos

• Compreender a noção de integral tripla;

• reduzir o cálculo de uma integral tripla a uma integral dupla.

Na aula 1, você aprendeu a noção de integral dupla. agora, você verá o conceito
de integral tripla.
Seja f : W ⊂ R3 → R, onde W é uma região sólida do R3 (região limitada
e fechada de R3 ). Como W é limitada, então existe um paralelepı́pedo (ou
caixa) R = [a, b] × [c, d] × [p, q], contendo W . Dividimos R em n3 subcai-
xas Rijk , por planos paralelos aos planos coordenados, todas de mesmo volume

∆V = ∆x∆y∆z, escolhemos x∗i , yj∗ , zk∗ ∈ Rijk e formamos a soma

n X
X n X
n

Sn = f x∗i , yj∗ , zk∗ ∆V
i=1 j=1 k=1

 
onde f x∗i , yj∗ , zk∗ = 0 se x∗i , yj∗ , zk∗ ∈/ W , dita soma de Riemann de f .
Se existir lim Sn = L, dizemos que f é integrável e o número L é dito integral
n→∞
tripla de f sobre o sólido W e é indicado por
ZZZ ZZZ ZZZ
f (x, y, z) dxdydz ou f (x, y, z) dV ou f dV .
W W W

OBS.:

1) Se f é contı́nua em W então f é integrável.


ZZZ
2) Se f (x, y, z) = 1 em W , então dxdydz = V (W ).
W
ZZZ ZZZ ZZZ
3) (f + g) dV = f dV + g dV .
W W W

41
Integrais Triplas

ZZZ ZZZ
4) kf dV = k f dV , k ∈ R.
W W

5) Se δ(x, y, z) é contı́nua e positiva em W , e representa a den-


sidade volumétrica de massa (massa por unidade de volume),
então a massa M de W é dada por
ZZZ
M= δ(x, y, z) dxdydz .
W

6) O centro de massa (x, y, z) é dado por


ZZZ
x · δ(x, y, z) dV
W
x=
M
ZZZ
y · δ(x, y, z) dV
W
y=
M
ZZZ
z · δ(x, y, z) dV
W
z= .
M

7) O momento de inércia em relação a um eixo E é dado por


ZZZ
IE = r2 (x, y, z) · δ(x, y, z) dV
W

onde r(x, y, z) = distância de (x, y, z) ao eixo E.


ZZZ
Se eixo E = eixo z, então Iz = (x2 + y 2 )δ (x, y, z) dV .
Z Z ZW
Se eixo E = eixo y, então Iy = (x2 + z 2 )δ (x, y, z) dV .
Z Z ZW
Se eixo E = eixo x, então Ix = (y 2 + z 2 )δ (x, y, z) dV .
W

Redução do Cálculo de uma Integral Tripla a uma Integral


Dupla

Observamos que um domı́nio de integração pode ser descrito como uma reunião
de regiões dadas por:

W1 = (x, y, z) | (x, y) ∈ Dxy e z1 (x, y) ≤ z ≤ z2 (x, y)
42
Cálculo IV – Aula 4

1 W
onde Dxy = projxOy (projeção de W1 sobre o plano xy) e z1 (x, y), z2 (x, y)
contı́nuas;

W2 = (x, y, z) | (x, z) ∈ Dxz e y1 (x, z) ≤ y ≤ y2 (x, z)

2W
onde Dxz = projxOz e y1 (x, z), y2 (x, z) contı́nuas;

W3 = (x, y, z) | (y, z) ∈ Dyz e x1 (y, z) ≤ x ≤ x2 (y, z)

3W
onde Dyz = projyOz e x1 (y, z), x2 (y, z) contı́nuas.
Os esboços de W1 , W2 e W3 são:

z z
z = z2 (x, y)
y = y1 (x, z)
y = y2 (x, z)
W1
Dxz W2
(x, y, z)
(x, z) (x, y, z)

z = z1 (x, y)

y y

Dxy (x, y)

x x

Dyz
(y, z)

W3
(x, y, z) x = x1 (y, z)
x

x = x2 (y, z)

43
Integrais Triplas

Prova-se que

ZZZ ZZ "Z #
z2 (x,y)
f (x, y, z) dxdydz = f (x, y, z) dz dxdy
W1 Dxy z1 (x,y)

ZZZ ZZ "Z #
y2 (x,z)
f (x, y, z) dxdydz = f (x, y, z) dy dxdz
W2 Dxz y1 (x,z)

ZZZ ZZ "Z #
x2 (y,z)
f (x, y, z) dxdydz = f (x, y, z) dx dydz .
W3 Dyz x1 (y,z)

Exemplo 1
ZZZ
2
Calcule ex dxdydz onde W é o conjunto 0 ≤ x ≤ 1, 0 ≤ y ≤ x e
W
0 ≤ z ≤ 1.
Solução:
Definimos W por:


W = (x, y, z) | (x, z) ∈ Dxz e 0 ≤ y ≤ x

onde Dxz = [0, 1] × [0, 1]. Logo:


ZZZ ZZ Z x 
x2 x2
e dxdydz = e dy dxdz
W Dxz 0
ZZ
2
= xex dxdz
Dxz
Z 1Z 1
2
= xex dxdz
0 0
" 2
#1 Z
1
ex
= dz
2 0
0

e−1
= .
2

Exemplo 2
Calcule o volume do sólido limitado pelos parabolóides de equação
z = x2 + y 2 e z = 8 − x2 − y 2 .

44
Cálculo IV – Aula 4

Solução:
Inicialmente, calculemos a interseção das superfı́cies:


 z = x2 + y 2 
⇔ x2 + y 2 = 8 − x 2 − y 2 ⇔ 2 x2 + y 2 = 8 ⇔ x 2 + y 2 = 4 .
 z = 8 − x2 − y 2

Logo, a interseção dos parabolóides é a circunferência x2 + y 2 = 4, situada no


plano z = 4.

z = 8 − x2 − y 2
W

4 (x, y, z)

z = x2 + y 2

Dxy
2 y
2
(x, y)

Descrevemos W por:


W = (x, y, z) | (x, y) ∈ Dxy e x2 + y 2 ≤ z ≤ 8 − x2 − y 2

ZZZ
2 2
onde Dxy é o disco x + y ≤ 4. Como V (W ) = dxdydz, então
W

"Z #
ZZ 8−x2 −y 2 ZZ
 
V (W ) = dz dxdy = 8 − 2(x2 + y 2 ) dxdy .
Dxy x2 +y 2 Dxy

45
Integrais Triplas

Passando para coordenadas polares, temos:

Z 2Z 2π Z 2
2

V (W ) = (8 − 2r )r dθdr = 2π 8r − 2r3 dr
0 0 0
 2
2 r4
= 2π 4r −
2 0

= 2π(16 − 8)

= 16π u.v.

Exemplo 3
Calcule a massa do sólido W , no primeiro octante, limitado pelos planos y = 0,
z = 0, x + y = 2, x + 2y = 4 e o cilindro y 2 + z 2 = 4, sendo a densidade igual
à distância de (x, y, z) ao plano xz.
Solução:
O esboço de W está representado na figura que se segue.

Dyz

(y, z)

W x=2−y

2
2
4 y
x x = 4 − 2y

Podemos definir W por:


W = (x, y, z) ∈ R3 | (y, z) ∈ Dyz e 2 − y ≤ x ≤ 4 − 2y

onde Dyz é tal que y 2 + z 2 ≤ 4, y ≥ 0 e z ≥ 0.

46
Cálculo IV – Aula 4

z
2

Dyz

2 y
ZZZ
Como M = δ(x, y, z) dxdydz, onde δ(x, y, z) = |y| = y, pois y ≥ 0, então:
W
ZZZ Z Z Z 4−2y 
M= y dxdydz = y dx dydz
W Dyz 2−y
ZZ
= y(4 − 2y − 2 + y) dydz
Dyz
ZZ

= 2y − y 2 dydz .
Dyz

Passando para coordenadas polares, temos:




 y = r cos θ
z = r sen θ

 dydz = rdrdθ

e Drθ é dado por: (


0≤r≤2
Drθ :
0 ≤ θ ≤ π/2
Então: Z π/2Z 2 
M = 2r cos θ − r2 cos2 θ r drdθ
0 0
Z π/2Z 2 
= 2r2 cos θ − r3 cos2 θ drdθ
0 0
Z π/2  2
2r3 r4 2
= cos θ − cos θ dθ
0 3 4 0
Z π/2  
16
= cos θ − 4 cos2 θ dθ
0 3
h 16  
4 sen 2θ iπ/2
= sen θ − θ+
3 2 2 0

16 π
= (1 − 0) − 2 ·
3 2
16
= − π u.m.
3
47
Integrais Triplas

Exercı́cios
ZZZ
1. Calcule x dxdydz onde W é o sólido limitado pelos planos coorde-
W
nados e pelo plano x + y + z = 1.

2. Seja W o sólido limitado pelas superfı́cies z + x2 = 4, y + z = 4, y = 0 e


z = 0.
a) Esboce W .
b) Calcule, por integral tripla, o volume do sólido W .
3. Calcule a massa do sólido W situado no primeiro octante, limitado pelos
planos x = 0, y = 2x e pelo cilindro y 2 + z 2 = 2, supondo que a densidade
no ponto (x, y, z) é proporcional à distância deste ponto ao plano xy.

4. Seja W um sólido limitado pelas superfı́cies y 2 + z = 4, y + z = 2, x = 0


e x = 2. ZZZ
a) Esboce W . b) Calcule eax dV .
W

5. Seja W um sólido limitado pelas superfı́cies z = y 2 , z = 2 − y 2 , x = 0 e


x + z = 4.
a) Esboce W .
b) Calcule o volume de W .
6. Calcule o volume da região W no primeiro octante limitada pelos planos
y = 0, z = 0, x + y = 2, 2y + x = 6 e o cilindro y 2 + z 2 = 4.

7. Determine o volume do sólido limitado pelas superfı́cies z = 3x2 ,


z = 4 − x2 , y = 0 e z + y = 6.

8. Seja o sólido W limitado pelas superfı́cies x2 + y 2 = 1, z + y = 2 e z = 0.


a) Esboce W .
b) Calcule a massa de W , supondo que a densidade em (x, y, z) é dada
por δ(x, y, z) = z.
9. Um sólido tem a forma de um cilindro circular de raio de base a e al-
tura h. Determine a massa supondo que a densidade em cada ponto seja
proporcional à distância do ponto a uma das bases do cilindro.

10. Calcule a massa de um sólido homogêneo limitado pelas superfı́cies z = y,


z = 4 − x2 , z = 0 e y = −4 com z ≥ 0.

48
Cálculo IV – Aula 5

Aula 5
Mudança de Variáveis na Integral Tripla
Objetivos

• Aprender a fazer mudança de variáveis em integrais triplas;


• estudar a mudança de variáveis cilı́ndricas;
• estudar a mudança de variáveis esféricas.

Aqui temos um resultado similar à mudança de variáveis em integral dupla:


ZZZ ZZZ

f (x, y, z) dxdydz = f x(u, v, w), y(u, v, w), z(u, v, w) |J| dudvdw
W Wuvw

onde
∂x ∂x ∂x

∂u ∂v ∂w

∂(x, y, z) ∂y ∂y ∂y
J= = 6= 0
∂(u, v, w) ∂u ∂v ∂w


∂z ∂z ∂z

∂u ∂v ∂w
é o jacobiano da mudança de variáveis

ϕ(u, v, w) = (x, y, z) = x(u, v, w), y(u, v, w), z(u, v, w)

e Wuvw = ϕ(W ).

Um caso particular de mudança de variáveis

Coordenadas cilı́ndricas

z
As coordenadas cilı́ndricas (r, θ, z) são defini-
das por

 P
 x = r cos θ

2 2 2
y = r sen θ ∴ x + y = r z


 z=z y
r y
θ
com r ≥ 0, θ0 ≤ θ ≤ θ0 + 2π, para algum θ0 x
e z ∈ R.
x

49
Mudança de Variáveis na Integral Tripla

As coordenadas r e θ são as mesmas que as coordenadas polares e, portanto,


as suas variações são encontradas na projeção de W no plano xy. A variação
de z é encontrada diretamente no sólido. Supondo que z1 (x, y) ≤ z ≤ z2 (x, y),
então a variação de z será

z1 (r cos θ, r sen θ) ≤ z ≤ z2 (r cos θ, r sen θ) .

Calculando o jacobiano da transformação cilı́ndrica, encontramos


∂(x, y, z)
J= =r (Verifique!)
∂(r, θ, z)
Logo:

ZZZ ZZZ
f (x, y, z) dxdydz = f (r cos θ, r sen θ, z) r drdθdz
W Wrθz

é a fórmula da integral tripla em coordenadas cilı́ndricas.

Exemplo 1
ZZZ 
Calcule zx2 + zy 2 dxdydz, sendo W o sólido limitado pelo cilindro
W
x2 + y 2 ≤ 1, pelo plano z = 0 e pelo parabolóide z = 4 − x2 − y 2 .
Solução:
De z = 4 − x2 − y 2 e x2 + y 2 = 1, temos z = 3. Isto significa que as superfı́cies
apresentam interseção no plano z = 3. O esboço de W é a figura que se segue.
Passando para coordenadas cilı́ndricas, temos


 x = r cos θ




 y = r sen θ
z= z .




 dxdydz = r drdθdz
 x2 + y 2 = r 2

Seja P = (x, y, z) ∈ W . Uma reta por P , paralela ao eixo z, intercepta a


fronteira de W em z = 0 e z = 4 − x2 − y 2 = 4 − r2 . Logo, 0 ≤ z ≤ 4 − r2 .
Como a projeção de W no plano xy é o disco x2 + y 2 ≤ 1, então 0 ≤ r ≤ 1
e 0 ≤ θ ≤ 2π. Logo o conjunto Wrθz é dado por:

 0≤r≤1

Wrθz : 0 ≤ θ ≤ 2π .

 0 ≤ z ≤ 4 − r2

50
Cálculo IV – Aula 5

4
z = 4 − x2 − y 2

W
P = (x, y, z)

1 y
1 z=0

Temos,

ZZZ ZZZ
2 2
 
zx + zy dxdydz = z x2 + y 2 dxdydz
W W
ZZZ
= zr2 · r drdθdz
Wrθz
ZZZ
= zr3 drdθdz
Wrθz
Z 1 Z 2π Z 4−r 2
3
= r z dzdθdr
0 0 0
Z 1 h 2 i4−r2 Z 2π
3 z
= r dθdr
0 2 0 0
Z 1
= π r3 (4 − r2 )2 dr
0
Z 1 
= π 16r3 − 8r5 + r7 dr
0

r8 1
h i
4r6
= π 4r4 − +
3 8 0

67π
= .
24

51
Mudança de Variáveis na Integral Tripla

Integral Tripla em Coordenadas Esféricas

z
As coordenadas esféricas (ρ, φ, θ) são definidas
por

P
 x = ρ sen φ cos θ

2 2 2 2 φ
y = ρ sen φ sen θ ∴ x + y + z = ρ ρ
 z
 z = ρ cos φ y
y
θ
com ρ ≥ 0, 0 ≤ φ ≤ π, θ0 ≤ θ ≤ θ0 + 2π, x
para algum θ0 .
x

A coordenada ρ mede a distância do ponto P à origem (portanto ρ ≥ 0).


A coordenada θ é a mesma que a coordenada cilı́ndrica e sua variação é en-
contrada na projeção de W no plano xy. A coordenada φ é o ângulo entre
o eixo z positivo (onde φ = 0) e a semireta OP . A variação máxima de φ
é 0 ≤ φ ≤ π.
Calculando o jacobiano da transformação esférica, temos:

∂(x, y, z)
J= = ρ2 sen φ (Verifique!)
∂(ρ, φ, θ)

Logo:

ZZZ
f (x, y, z) dV =
W
ZZZ
= f (ρ sen φ cos θ, ρ sen φ sen θ, ρ cos φ) ρ2 sen φ dρdφdθ
Wρφθ

é a fórmula da integral tripla em coordenadas esféricas.

Exemplo 1
Calcule o volume da esfera W : x2 + y 2 + z 2 ≤ a2 , com a > 0.

52
Cálculo IV – Aula 5

Solução:
O esboço de W está representado na figura que se segue.
z
a

a y
a

Passando para coordenadas esféricas, temos




 x = ρ sen φ cos θ




 y = ρ sen φ sen θ
z = ρ cos φ .

 2


 dV = dxdydz = ρ sen φ dρdφdθ
 x 2 + y 2 + z 2 = ρ2

A equação da esfera x2 + y 2 + z 2 = a2 fica ρ = a. Logo, o conjunto Wρφθ


é dado por: 
 0≤ρ≤a

Wρφθ : 0≤φ≤π .

 0 ≤ θ ≤ 2π
ZZZ
Como V (W ) = dxdydz então:
W
ZZZ
V (W ) = ρ2 sen φ dρdφdθ
Wρφθ
Z a Z π Z 2π
2
= ρ sen φ dθdφdρ
0 0 0
Z a Z π
2
= 2π ρ sen φ dφdρ
0 0
Z a


= 2π − cos φ 0 ρ2 dρ
0
h 3 ia
ρ
= 4π
3 0

4πa3
= u.v.
3

53
Mudança de Variáveis na Integral Tripla

Exemplo 2
x2 y2 z2
Calcule o volume do elipsóide W : + + ≤ 1, (a, b, c > 0).
a2 b2 c2
Solução:
Façamos a mudança de variáveis

 x = au

y = bv

 z = cw

Temos
∂x ∂x ∂x


∂u ∂v ∂w a 0 0


∂(x, y, z) ∂y ∂y ∂y
J= = = 0 b 0 = abc 6= 0 .
∂(u, v, w) ∂u ∂v ∂w

0 0 c
∂z ∂z ∂z

∂u ∂v ∂w
Logo,
dxdydz = |J| dudvdw = abc dudvdw .
x2 2 2
O elipsóide W : + yb2 + zc2 ≤ 1 é transformado na esfera Wuvw : u2 +v 2 +w2 ≤ 1.
ZaZ2 Z
Como V (W ) = dxdydz, então:
W
ZZZ
V (W ) = |J| dudvdw
Wuvw
ZZZ
= abc dudvdw
Wuvw

= abc V (Wuvw )

4
= abc · · π · 13
3
4
= πabc .
3

54
Cálculo IV – Aula 5

Exercı́cios

1. Calcule a massa do sólido limitado pelo parabolóide z = x2 + y 2 e pelo


plano z = 4, sendo a densidade em cada ponto do sólido dada por
δ(x, y, z) = x2 + y 2 )1/2 .
p
2. Calcule a massa do cone x2 + y 2 ≤ z ≤ 1, sendo a densidade no ponto
(x, y, z) proporcional ao quadrado da distância do ponto ao eixo z.
ZZZ p
3. Calcule x2 + y 2 + z 2 dV , onde W é limitado inferiormente pelo
W
coneq

z = 3 x2 + y 2 e superiormente pela esfera x2 + y 2 + z 2 = 4.

4. Considere a integral iterada dada em coordenadas cilı́ndricas:


Z πZ 2Z √
4−r 2
I= r2 sen θ dzdrdθ .
0 0 0

a) Expresse I em coordenadas esféricas segundo a ordem de integração


dθdϕdρ.
b) Calcule o valor de I.
r
1 2
5. Seja W um sólido dado por x2 + y 2 + z 2 ≤ 4z e z ≥ (x + y 2 ) .
3
a) Esboce W .
b) Calcule o volume de W .
6. Calcule o volume do conjunto W dado por x2 + y 2 + (z − 1)2 ≤ 1
e z ≥ x2 + y 2 .
ZZZ
1
7. Calcule 2 2 2
dV , sendo W a região interior ao cone
p W x +y +z
z = x2 + y 2 , limitada superiormente pela esfera x2 + y 2 + z 2 = 4
e inferiormente pela esfera x2 + y 2 + z 2 = 1.
p
8. Calcule a massa do sólido W inferior ao cone z = 3(x2 + y 2 ) e limitado
pela esfera x2 + y 2 + (z − 1)2 = 1, sendo a densidade igual ao quadrado
da distância de (x, y, z) ao plano z = 0.

9. Considere a integral iterada


Z 2Z √ √
4−x2Z 4−x2 −y 2 p
I= z x2 + y 2 dzdydx .
0 0 0

a) Expresse I em coordenadas cilı́ndricas e calcule o valor de I.


b) Expresse I em coordenadas esféricas (sem calculá-la).

55
Mudança de Variáveis na Integral Tripla

10. Considere o sólido homogêneo, limitado pelo plano z = 0, o cilindro


p
x2 + y 2 = 2y e pelo cone z = x2 + y 2 . Calcule o momento de inércia
em relação ao eixo z.

56
Cálculo IV – Aula 6

Aula 6
Curvas Parametrizadas
Objetivos

• Parametrizar curvas planas e espaciais;

• compreender a noção de integral de linha de campo escalar;

• estudar algumas propriedades.

Parametrização de curvas

Parametrizar uma curva C ⊂ Rn (n = 2 ou 3) consiste em apresentar uma


função vetorial σ : I ⊂ R → Rn (n = 2 ou 3), onde I é um intervalo
e σ(I) = C.
y

σ σ(t)

t C

x
I

Exemplo 1
Sendo A, B ∈ Rn (n = 2 ou 3), parametrize o segmento de reta C de extremi-
dade inicial A e final B.
Solução:
−→ −−→ −→
Se P ∈ C, então OP = OB+tAB , 0 ≤ t ≤ 1 ou P −0 = B−0+t(B−A), com
0 ≤ t ≤ 1 ou P = B + t(B − A), com 0 ≤ t ≤ 1. Então, uma parametrização
do segmento C é dada por

σ(t) = B + t(B − A) ,

com 0 ≤ t ≤ 1.

57
Curvas Parametrizadas

Exemplo 2
Seja C ⊂ plano xy, gráfico de uma função y = f (x) , x ∈ I.

(x, f (x)) C

x x
I

Então, uma parametrização de C é dada por

σ(t) = (t, f (t)) , t ∈ I .

Exemplo 3
Seja C a circunferência x2 + y 2 = a2 , a > 0. Seja P = (x, y) ∈ C. Seja t o
ângulo em radianos entre o eixo positivo x e a semireta OP .

y
C
a
P
a
t y
x a x

Observe que quando t aumenta de 0 a 2π, o ponto P = (x, y) = (a cos t, a sen t)


se move, uma vez sobre C no sentido anti-horário a partir do ponto (a, 0). Então,
uma parametrização de C é

σ1 (t) = (a cos t, a sen t) , 0 ≤ t ≤ 2π .

Observe que σ2 (t) = (a sen t, a cos t) , 0 ≤ t ≤ 2π é também uma parame-


trização de C, pois x2 + y 2 = a2 . Neste caso, quando t aumenta de 0 a 2π, o
ponto P se move uma vez ao longo de C no sentido horário a partir do ponto
(0, a).

58
Cálculo IV – Aula 6

Observe que σ3 (t) = (a cos(2π − t), a sen(2π − t)) = (a cos t, −a sen t), com
0 ≤ t ≤ 2π é outra parametrização de C e P se move ao longo de C no sentido
horário a partir do ponto (a, 0).

Exemplo 4
Seja a circunferência C : (x − x0 )2 + (y − y0 )2 = a2 , de centro (x0 , y0 ) e raio a.
Efetuando uma mudança de variáveis u = x − x0 e v = y − y0 , temos

u2 + v 2 = a 2

que é uma circunferência no plano uv, de centro (0, 0) e raio a. Logo,


(
u = a cos t
, 0 ≤ t ≤ 2π .
v = a sen t

Substituindo acima, temos


(
x = x0 + a cos t
, 0 ≤ t ≤ 2π .
y = y0 + a sen t

Assim, uma parametrização diferenciável de C é dada por

σ(t) = (x0 + a cos t, y0 + a sen t) , 0 ≤ t ≤ 2π .

Exemplo 5
2 2
(x − x0 ) (y − y0 ) x − x0 y − y0
Seja uma elipse C : + = 1. Fazendo u = ev= ,
a2 b2 a b
mostramos que

σ(t) = (x0 + a cos t, y0 + b sen t) , 0 ≤ t ≤ 2π

é uma parametrização de C.

Exemplo 6
Seja C uma curva do espaço dada pela interseção do cilindro x2 + y 2 = 1 com
o plano x + z = 2.

a) Esboce C.
b) Apresente uma parametrização diferenciável para C.
59
Curvas Parametrizadas
z
Solução:
a) Inicialmente, façamos o esboço
do cilindro x2 + y 2 = 1. Desenhe-
mos, no plano xy, a circunferência
x2 + y 2 = 1. Pelos pontos (1, 0, 0),
(−1, 0, 0), (0, 1, 0) e (0, −1, 0) tra- (−1, 0, 0)
cemos paralelas ao eixo z. (0, −1, 0)
(0, 1, 0) y
(1, 0, 0)

Para esboçar o plano x + z = 2, traçamos a reta x + z = 2 no plano xz. Observe


que a equação do plano não contém a variável y. Por isso, por pontos da reta
traçamos paralelas ao eixo y.

y
2

Agora, juntemos as duas figuras, procurando destacar alguns pontos de in-


terseção. A reta x + z = 2 intercepta o cilindro nos pontos A1 e A2 . Por
outro lado, a reta do plano, paralela ao eixo y, passando por (0, 0, 2) intercepta
o cilindro nos pontos B1 e B2 . A curva C passa por A1 , B1 , A2 e B2 .

z
A2

C
B1
2 B2

A1

1
1 y
2
x
60
Cálculo IV – Aula 6

b) Seja (x, y, z) ∈ C. Logo, x e y satisfazem x2 + y 2 = 1. Assim, x = cos t e


y = sen t, com 0 ≤ t ≤ 2π. Como z = 2 − x, então z = 2 − cos t. Logo,

σ(t) = (cos t, sen t, 2 − cos t) , 0 ≤ t ≤ 2π

é uma parametrização de C.

Exemplo 7
Seja C a curva no espaço representada pela função vetorial de equação
σ(t) = (a cos t, a sen t, bt), 0 ≤ t ≤ 4π , a > 0, b > 0. Esboce C, dita
hélice circular.
Solução:
De x = a cos t , y = a sen t, temos x2 + y 2 = a2 . Isto significa que C está
contida no cilindro x2 + y 2 = a2 . Como z = bt, quando t vai de 0 a 4π, o ponto
(x, y, z) percorre a hélice contida no cilindro.

z
C

(a, 0, 4π)

a y
a
x

Integral de Linha de Campo Escalar

Nesta aula definiremos uma integral similar a uma integral definida. Sejam dados
um campo escalar em R3 ou uma função real de três variáveis f : R3 → R e
uma curva C em R3 , dada por σ(t) = (x(t), y(t), z(t)) , t ∈ [a, b], com σ de
classe C 1 (veja a figura que se segue).

61
Curvas Parametrizadas

z
a = t0 σ(ti−1 )
σ(ti )
∆t
ti−1 σ σ(ti∗ )
ti

ti∗
C
b = tn y

Dividimos o intervalo [a, b] em n subintervalos Ii , i = 1, · · · , n, de mesmo


b−a
comprimento ∆t = . Logo, a curva C fica dividida em n subarcos de
n
comprimento ∆s1 , ∆s2 , · · · , ∆sn , onde ∆si ≃ kσ ′ (t∗i )k ∆t para algum t∗i ∈ Ii .
Formemos a soma

n
X n
X
f (σ(t∗i )) ∆si = f (σ(t∗i )) kσ ′ (t∗i )k ∆t ,
i=1 i=1

dita soma de Riemann de f (σ(t)) kσ ′ (t)k.


Definimos a integral de linha de f sobre C por

Z Z n
X
f ds = f (x, y, z) ds = lim f (σ(t∗i )) kσ ′ (t∗i )k ∆t
C C n→∞
i=1

se o limite existir.

OBS.:

1) Se f é uma função contı́nua então o limite existe e, portanto


Z Z b
f (x, y, z) ds = f (σ(t)) kσ ′ (t)k dt =
C a | {z }
ds
Z b q
= f (x(t), y(t), z(t)) (x′ (t))2 + (y ′ (t))2 + (z ′ (t))2 dt .
a

62
Cálculo IV – Aula 6

2) Se f (x, y) é uma função contı́nua em R2 e C uma curva em R2 ,


dada por σ(t) = (x(t), y(t)) , t ∈ [a, b], com σ de classe C 1 ,
então definimos
Z Z Z b
f ds = f (x, y) ds = f (σ(t)) kσ ′ (t)k dt =
C C a | {z }
ds
Z b q
= f (x(t), y(t)) (x′ (t))2 + (y ′ (t))2 dt .
a

3) Se f (x, y) = 1 (ou f (x, y, z) = 1), então


Z
f ds = comprimento de C .
C

4) A integral de linha de um campo escalar f não depende da pa-


rametrização de C e nem de sua orientação, isto é, denotando
por C − a curva C percorrida em outro sentido, então
Z Z
f ds = f ds .
C− C

C− B

5) Se C é uma curva dada por uma parametrização


σ : [a, b] → Rn (n = 2 ou 3), C 1 por partes, isto é, σ é
contı́nua e existe uma partição a = t0 < t1 < ... < tn = b de

[a, b] de modo que σi = σ [ti−1 , ti ] é de classe C 1 , i = 1, · · · , n,
então Z n Z
X
f ds = f ds
C i=1 Ci

onde Ci = σi ([ti−1 , ti ]).

C1 C3
C2

63
Curvas Parametrizadas

Exemplo 1
Seja C a interseção do cilindro
Z parabólico x = y 2 com a parte do plano z = y,
tal que 0 ≤ y ≤ 1. Calcule y ds.
C

Solução:
Façamos y = t. Logo, x = t2 e z = t. Como 0 ≤ y ≤ 1, então temos que
0 ≤ t ≤ 1. Assim, uma parametrização de C é dada por σ(t) = (t2 , t, t), com
0 ≤ t ≤ 1, donde σ ′ (t) = (2t, 1, 1). Como, ds = kσ ′ (t)k dt então temos que
√ √
ds = 4t2 + 1 + 1 dt = 2 + 4t2 dt. Logo:
Z Z 1 √ Z 1
1/2
y ds = 2
t 2 + 4t dt = 2 + 4t2 t dt .
C 0 0
d(2 + 4t2 )
Observe que d(2 + 4t2 ) = 8t dt, donde t dt = . Logo:
8
Z Z 1
1 1/2
y ds = 2 + 4t2 d(2 + 4t2 )
C 8 0
3/2 1
1 2
= · 2 + 4t2
8 3 0

1 
= 63/2 − 23/2
12
 √ √ 
1
= 3 6− 2 .
6

Exemplo 2
Z
Calcule x ds, onde C é formado pelo segmento de reta C1 de (0, 2) a (0, 1),
C
seguido do arco C2 da parábola y = 1 − x2 de (0, 1) a (1, 0).
Solução:
O esboço de C está representado na figura a seguir.
y
2
C1

1
C2

1 x

Como C = C1 ∪ C2 , temos:
Z Z Z Z Z
x ds = x ds + x ds = x ds + x ds .
C C1 C2 C1− C2
64
Cálculo IV – Aula 6

Z
Cálculo de x ds
C1−

Uma parametrização de C1− é dada por σ(t) = (0, t), com 1 ≤ t ≤ 2. Logo,
σ ′ (t) = (0, 1), donde kσ ′ (t)k = 1 e, portanto, ds = kσ ′ (t)k dt = dt.
Assim,
Z Z 2
x ds = 0 dt = 0 .
C1− 1

Z
Cálculo de x ds
C2

Uma parametrização de C2 é dada por σ(t) = (t, 1 − t2 ), com 0 ≤ t ≤ 1, donde



σ ′ (t) = (1, −2t). Logo ds = kσ ′ (t)k dt = 1 + 4t2 dt. Então,

Z Z 1 √ Z 1 1/2
x ds = 2
t 1 + 4t dt = 1 + 4t2 dt .
C2 0 0

d(1 + 4t2 )
Observe que t dt = . Logo,
8

Z Z 1
1 1/2
x ds = 1 + 4t2 d(1 + 4t2 )
C2 8 0
3/2 1
1 2
= · 1 + 4t2
8 3 0
 √ 
1
= 5 5−1 .
12

Portanto,
Z  √ 
1
x ds = 5 5−1 .
C 12

Exemplo 3
2 2 2
Seja a curva C obtida como interseção
Z da semi-esfera x + y + z = 4 , y ≥ 0
com o plano x + z = 2. Calcule f (x, y, z) ds, onde f (x, y, z) é dada por
C
f (x, y, z) = xy.
Solução:
O esboço de C está representado na figura que se segue.
65
Curvas Parametrizadas

z
2

2 y
2

Seja (x, y, z) ∈ C. Então x2 + y 2 + z 2 = 4 , y ≥ 0 e x + z = 2. Donde,


x2 + y 2 + (2 − x)2 = 4, com y ≥ 0 ou 2x2 − 4x + y 2 = 0, com y ≥ 0 ou
y2
2(x − 1)2 + y 2 = 2, com y ≥ 0 ou (x − 1)2 + = 1 , y ≥ 0. Logo, a projeção de
2 √
C sobre o plano xy é a semi-elipse de centro (1, 0) e semi-eixos 1 e 2 . Então,

 x=√
 1 + cos t
y = 2 sen t .

 z = 2 − (1 + cos t) = 1 − cos t


Como y ≥ 0 então 2 sen t ≥ 0, donde 0 ≤ t ≤ π. Logo, uma parametrização
para C é dada por
 √ 
σ(t) = 1 + cos t, 2 sen t, 1 − cos t , 0 ≤ t ≤ π .

Temos

 √ 
σ (t) = − sen t, 2 cos t, sen t
donde
√ √
ds = kσ ′ (t)k dt = sen2 t + 2 cos2 t + sen2 t dt = 2 dt .
Então,
Z Z Z π √ √
f (x, y, z) ds = xy ds = (1 + cos t) 2 sen t 2 dt
C C 0
Z π
= 2 (sen t + sen t cos t) dt
0

sen2 t π
h i
= 2 − cos t +
2 0

= 4.

66
Cálculo IV – Aula 6

Exercı́cios
Z
1
1. Calcule 2 2 2
ds, onde C é dada por γ(t) = (cos t, sen t, t),
C x +y +z
com 0 ≤ t ≤ 2π.
Z p

→ −

2. Calcule x2 + y 2 ds ao longo da curva γ(t) = (4 cos t) i +(4 sen t) j +
→ C

+3t k , com −2π ≤ t ≤ 2π.

3. Seja C a semicircunferência x2 + y 2 = a2 , y ≥ 0 e a > 0.


a) Parametrize C.
Z 
b) Calcule x2 − y 2 ds.
C

4. Seja C o segmento de extremidades A = (1, 2, 3) e B = (4, 5, 6).

a) Parametrize C.
Z
b) Calcule (x + y + z) ds.
C
Z
5. Calcular a integral (x − y) ds, onde C é a circunferência x2 + y 2 = ax.
C
Z
6. Calcule y 2 ds, onde C é a semicircunferência x2 + y 2 = 2x, com y ≥ 0.
C
Z
7. Calcule 8x ds, onde a curva C é formada pelo arco C1 da parábola
C
y = x2 de (0, 0) a (1, 1), seguido pelo segmento de reta vertical C2 de
(1, 1) a (1, 2).

8. Seja C a curva interseção das superfı́cies x2 + y 2 + z 2 = 4, z ≥ 0 e y = x,


√ √ √
que liga o ponto A = ( 2 , 2 , 0) ao ponto B = (1, 1, 2 ).
a) Esboce C.
b) Parametrize C.
Z
c) Calcule x2 ds.
C
Z
9. Calcular x ds, onde C é a interseção do cilindro parabólico y = x2 com
C
a parte do plano z = x, tal que 0 ≤ x ≤ 1.

10. Seja C a curva interseção da semi-esfera


Z x2 +y 2 +z 2 = 8−2(x+y) , z ≥ 0
com o plano x + y = 2. Calcule (x2 + y 2 )z ds.
C

67
Curvas Parametrizadas

68
Cálculo IV – Aula 7

Aula 7
Aplicações da Integral de Linha de Campo
Escalar
Objetivos
• Apresentar uma interpretação geométrica;
• apresentar algumas aplicações à Fı́sica;
• apresentar os campos vetoriais;
• estudar alguns operadores diferenciais.

Interpretação Geométrica no Plano

Seja f (x, y) ≥ 0 e contı́nua. Então o gráfico de f , Gf , está acima do plano xy.

(x, y, f (x, y)) Gf

S y
(x, y)
∆s

x
C

A partir da curva C ⊂ plano xy, construa


Z a superfı́cie S de base C e “altura”
f (x, y) em (x, y) ∈ C. A integral f (x, y) ds representa a área de um lado da
C
superfı́cie S.

Exemplo 1
A base de uma superfı́cie é dada por x2 + y 2 = 2 , x ≥ 0. Se a altura da
superfı́cie em (x, y) é f (x, y) = x, x ≥ 0, achar a área de um lado da superfı́cie.

69
Aplicações da Integral de Linha de Campo Escalar

Solução:
O esboço de S está representado na figura que se segue.

z = f (x, y) = x
S

√ √
2 2 y
(x, y)
x

Z Z
A área de um lado de S é dada por f (x, y) ds = x ds, onde C é parametri-
√ √ C C

zada por σ(t) = 2 cos t, 2 sen t , com −π/2 ≤ t ≤ π/2 (pois x ≥ 0).
 √ √  √ √

Se σ (t) = − 2 sen t, 2 cos t , então kσ ′ (t)k = 2 sen2 t + 2 cos2 t = 2
donde,

ds = kσ ′ (t)k dt = 2 dt .

Então
Z Z √
π/2 √ π/2

x ds = 2 cos t 2 dt = 2 sen t = 4 u.a.
C −π/2 −π/2

Interpretação Fı́sica

Se δ(x, y) representa aZ densidade (massa por unidade de comprimento) de um


arame C ⊂ R2 , então δ(x, y) ds representa a massa total do arame:
C

Z
M= δ(x, y) ds .
C

70
Cálculo IV – Aula 7

OBS.:

1. O centro de massa (x, y) do arame é dado por


Z
Mx = xδ(x, y) ds
C
Z
My = yδ(x, y) ds
C

2. O momento de inércia de C ⊂ R2 em relação a um eixo E é


dado por Z
IE = r2 (x, y)δ(x, y) ds
C

onde r(x, y) = distância de (x, y) ao eixo E.

3. Seja uma curva C ⊂ R3 , representando um arame de densidade


δ = δ(x, y, z) em (x, y, z) ∈ C. Então, observe as seguintes
fórmulas:
Z
(i) Comprimento do arame: L = ds
C
Z
(ii) Massa do arame: M = δ(x, y, z) ds
C
(iii) Centro de massa do arame (x, y, z), onde
Z
Mx = xδ(x, y, z) ds
C
Z
My = yδ(x, y, z) ds
C
Z
Mz = zδ(x, y, z) ds
C

(iv) Momento de inércia do arame em relação a um eixo E:


Z
IE = r2 (x, y, z)δ(x, y, z) ds
C

onde r(x, y, z) = distância de (x, y, z) ao eixo E.

Exemplo 1
Um arame fino tem a forma de uma semicircunferência x2 + y 2 = 4, y ≥ 0. Se
a densidade linear é uma constante k, determine a massa e o centro de massa
do arame.
71
Aplicações da Integral de Linha de Campo Escalar

Solução:

O esboço de C está representado


ao lado. Temos y
Z Z
xk ds x ds 2
C C
x= Z = Z
k ds ds C
C C
Z
y ds
−2 2 x
y = ZC
ds
C
Z Z
1
onde ds = L = · 2πr = 2π, pois r = 2. Como M = k ds então
C
Z 2 C

M =k ds = 2kπ. Uma parametrização de C é dada por:


C

σ(t) = (2 cos t, 2 sen t) , 0 ≤ t ≤ π .



Se σ ′ (t) = (−2 sen t, 2 cos t, ) então kσ ′ (t)k = 4 sen2 t + 4 cos2 t = 2. Como
ds = kσ ′ (t)k dt, então ds = 2 dt. Temos:
Z Z π
 π
x ds = (2 cos t)2 dt = 4 sen t 0 = 0
C 0
Z Z π  π
y ds = (2 sen t)2 dt = 4 − cos t 0 = 8
C 0

Logo,
8 4
x=0 e y= = .
2π π
Portanto, (x, y) = (0, 4/π).

Exemplo 2
Calcule o momento de inércia em relação ao eixo z de um arame C cuja forma é
a interseção das superfı́cies x2 + y 2 + z 2 = 4 e y = x, sabendo que sua densidade
é uma constante.
Solução:
Como a interseção de uma esfera com um plano é uma circunferência, segue
que C é uma circunferência contida no plano y = x. Para esboçá-la procuremos
encontrar pontos de interseção das duas superfı́cies. Observe que o plano x = y
contém o eixo z. Logo, os pontos A1 = (0, 0, 2) e A2 = (0, 0, −2) estão em C.

72
Cálculo IV – Aula 7

A1 = (0, 0, 2)
C

B2

B1 2
y
2
x

A2 = (0, 0, −2)

Por outro lado, a reta y = x do plano xy intercepta a esfera em dois pontos: B1


e B2 . Ligando os pontos A1 , A2 , B1 e B2 , encontramos a curva C.
Para parametrizar C, resolvemos o sistema
(
x2 + y 2 + z 2 = 4
.
y= x
Temos 2x2 +z 2 = 4 ou x2 /2+z 2 /4 = 1, que representa a projeção de C no plano
xz. Portanto, se (x, y, z) ∈ C, então x e z satisfazem a elipse x2 /2 + z 2 /4.

Logo, x = 2 cos t e z = 2 sen t, com 0 ≤ t ≤ 2π. Como y = x, então

y = 2 cos t. Portanto,
√ √ 
σ(t) = 2 cos t, 2 cos t, 2 sen t , 0 ≤ t ≤ 2π
é uma parametrização de C.
√ √ 
Se σ ′ (t) = − 2 sen t, − 2 sen t, 2 cos t então:

kσ ′ (t)k = 2 sen2 t + 2 sen2 t + 4 cos2 t = 2 .
Assim, ds = kσ ′ (t)k dt = 2 dt. O momento de inércia em relação ao eixo z é
dado por
Z Z
2 2
 
Iz = x + y δ(x, y) ds = k x2 + y 2 ds
C C
Z 2π 
= k 2 cos2 t + 2 cos2 t 2 dt
0
Z 2π
= 8k cos2 t dt
0
h i2π
1 sen 2t
= 8k · t+
2 2 0

= 8kπ .
73
Aplicações da Integral de Linha de Campo Escalar

Campos Vetoriais

Definição de um Campo Vetorial

Definição:

Sejam P e Q funções reais de x e y, definidas em D ⊂ R2 . A função




vetorial F : D ⊂ R2 → R2 definida por

→ −
→ −

F (x, y) = (P (x, y), Q(x, y)) = P (x, y) i + Q(x, y) j

é chamada de campo vetorial definido em D ⊂ R2 .


y



F (x, y)

D (x, y)

x


Outra notação: F (x, y) = (P, Q).

Definição:

Sejam P , Q e R funções reais de x, y e z, definidas em D ⊂ R3 . Temos




que a função vetorial F : D ⊂ R3 → R3 definida por


F (x, y, z) = (P (x, y, z), Q(x, y, z), R(x, y, z))

→ −
→ −

= P (x, y, z) i + Q(x, y, z) j + R(x, y, z) k

é chamada de campo vetorial definido em D ⊂ R3 .




z F (x, y, z)

(x, y, z)

74
Cálculo IV – Aula 7

Os campos vetoriais são úteis para representar os campos de forças, campos de


velocidades e campos elétricos.


Geometricamente, visualizamos um campo vetorial F no plano esboçando veto-


res F (x, y) com origem em (x, y).

Exemplo 1

→ −
→ −

O campo vetorial F (x, y) = (x, y) = x i + y j , (x, y) ∈ R2 , está representado
por:
y

Exemplo 2

→ −
→ − →
Represente geometricamente o campo vetorial F (x, y) = (−y, x) = −y i +x j ,
com (x, y) ∈ R2 .
Solução:

→ p −

Observemos que k F (x, y)k = y 2 + x2 = k(x, y)k, isto é, os vetores F (x, y)
e (x, y) têm mesmo comprimento. Além disso,


F (x, y) · (x, y) = (−y, x) · (x, y) = −yx + xy = 0 ,


donde F (x, y) ⊥ (x, y).
Então o esboço do campo está representado na figura que se segue.

75
Aplicações da Integral de Linha de Campo Escalar

Definição:


Dizemos que o campo vetorial F é contı́nuo, de classe
C k , k ∈ N∗ ou C ∞ se as funções componentes P e
Q (ou P , Q, R) são contı́nuas, de classe C k ou C ∞ ,
respectivamente.

Operadores Diferenciais


Se F = (P, Q, R) é campo vetorial diferenciável em um conjunto aberto D do


R3 , então o divergente de F é um campo escalar definido por


→ ∂P ∂Q ∂R
div F = + + (1)
∂x ∂y ∂z


→ −
→ ∂P ∂Q
Se F = (P, Q) é de classe C 1 em um aberto D do R2 , então div F = + .
∂x ∂y


O rotacional de F é um campo vetorial definido por
→  ∂R ∂Q  −
− →  ∂P ∂R −
→ 
∂Q ∂P −
→
rot F = − i + − j + − k (2)
∂y ∂z ∂z ∂x ∂x ∂y

Vamos expressar (1) e (2) usando a notação de operador. Então, consideremos


o operador diferencial vetorial ∇ (“del”) dado por
∂ −
→ ∂ −
→ ∂ −
→ ∂ ∂ ∂
∇= i + j + k = , , .
∂x ∂y ∂z ∂x ∂y ∂z

O operador ∇ sobre uma função escalar f (ou um campo escalar) produz o


gradiente de f :  
∂f ∂f ∂f
∇f = , , .
∂x ∂y ∂z

76
Cálculo IV – Aula 7



Consideremos o “produto vetorial” de ∇ pelo campo vetorial F = (P, Q, R):

−→ −
→ −

i j k


∇ × F = ∂/∂x ∂/∂y ∂/∂z

P Q R


∂/∂y ∂/∂z → ∂/∂x ∂/∂z −
→ ∂/∂x ∂/∂y −

= i −

j +

k

Q R
P R P Q
     
∂R ∂Q −
→ ∂R ∂P −
→ ∂Q ∂P −

= − i − − j + − k
∂y ∂z ∂x ∂z ∂x ∂y
     
∂R ∂Q −
→ ∂P ∂R −
→ ∂Q ∂P −

= − i + − j + − k
∂y ∂z ∂z ∂x ∂x ∂y



= rot F .

Logo,

→ −

rot F = ∇ × F .


Consideremos o “produto interno” de ∇ pelo campo F :
→ ∂ ∂ ∂
− ∂P ∂Q ∂R −

∇· F = , , · (P, Q, R) = + + = div F .
∂x ∂y ∂z ∂x ∂y ∂z
Assim,

→ −

div F = ∇ · F .

Exemplo 1
Calcule o divergente e o rotacional do campo vetorial

→ −
→ −
→ −

F (x, y, z) = xy i + yz j + zx k .

Solução:
Temos

→ −
→ ∂ ∂ ∂
div F = ∇ · F = (xy) + (yz) + (zx) = y + z + x .
∂x ∂y ∂z
e

−→ −
→ −

i j k

→ −

rot F = ∇ × F = ∂/∂x ∂/∂y ∂/∂z

xy yz zx


→ −
→ −
→ −
→ −
→ −

= (0 − y) i + (0 − z) j + (0 − x) k = −y i − z j − x k .

77
Aplicações da Integral de Linha de Campo Escalar

A seguir, apresentaremos algumas propriedades para o rotacional e o divergente.




Se f e F são de classe C 2 , então

→ −

(i) rot(gradf ) = 0 ou ∇ × (∇f ) = 0

→ −
→
(ii) div rot F = 0 ou ∇ · ∇ × F = 0
(iii) div (gradf ) = lap f ou ∇ · (∇f ) = ∇2 f onde

∂2f ∂2f ∂2f


lap f = ∇2 f = + +
∂x2 ∂y 2 ∂z 2

é dito laplaciano de f .
 −→ −
→ −

(iv) ∇ · f F = f ∇ · F + ∇f · F .
As demonstrações de (i) e (ii) seguem das definições e do Teorema de Schwartz.
A demonstração de (iii) segue das definições. Demonstraremos a propriedade

→ −

(iv). Escrevendo F = (P, Q, R), temos f F = (f P, f Q, f R). Então
 −
→ ∂ ∂ ∂
∇· fF = (f P ) + (f Q) + (f R)
∂x ∂y ∂z
∂P ∂f ∂Q ∂f ∂R ∂f
= f + ·P +f + ·Q+f + ·R
∂x ∂x ∂y ∂y ∂z ∂z
   
∂P ∂Q ∂R ∂f ∂f ∂f
= f + + + , , · (P, Q, R)
∂x ∂y ∂z ∂x ∂y ∂z

→ −

= f ∇ · F + ∇f · F .

como querı́amos demonstrar.



→ −
→ −

OBS.: Se F (x, y) = P (x, y) i + Q(x, y) j , então:
→  ∂Q ∂P −
− →
rot F = − k .
∂x ∂y

78
Cálculo IV – Aula 7

Exercı́cios

1. Calcule a massa de um fio que tem a forma do arco de elipse


C : x2 + y 2 /4 = 1, no primeiro quadrante, se a densidade em cada ponto
(x, y) do fio é dada por δ(x, y) = xy.

2. Calcule a primeira coordenada do centro de massa de um fio homogêneo


→ 2√2 5/2 −
− → t4 −→
que está ao longo de uma curva γ(t) = t i + 5 t j + k , 0 ≤ t ≤ 2,
4
se a densidade for δ(x, y, z) = 10x.

3. Um arame tem a forma da curva C obtida como interseção da porção da


esfera x2 + y 2 + z 2 = 1, x ≥ 0, com o plano y + z = 1.

a) Esboce a curva C.
b) Apresente uma parametrização diferenciável para C.
c) Sabendo-se que a densidade em cada ponto do arame é dada por
δ(x, y, z) = yz, calcule a massa total do arame.

4. Deseja-se construir uma peça de zinco que tem a forma da superfı́cie do


cilindro x2 + y 2 = 4, compreendida entre os planos z = 0 e x + y + z = 2,
z ≥ 0. Se o metro quadrado do zinco custa M reais, calcule o preço total
da peça. Faça um esboço da peça.

5. Calcule a área de um lado da superfı́cie S cuja base é a circunferência


x2 +y 2 = 1 no plano xy e a altura em cada ponto (x, y) é f (x, y) = 1−x2 .
Esboce a superfı́cie.

6. Um pintor deseja pintar ambos os lados de uma cerca cuja base está dada
pela curva C : x2/3 + y 2/3 = (20)2/3 , para x ≥ 0 e y ≥ 0. A altura está
dada por f (x, y) = y. Se o pintor cobra R reais por metro quadrado,
quanto ele receberá?

7. Calcule o momento de inércia de um fio retilı́neo, homogêneo de compri-


mento L, em torno de um eixo perpendicular ao fio e passando por uma
das extremidades do fio, em função de sua massa.

8. Um fio delgado tem a forma do segmento de reta que une os pontos (1, 1)
a (2, 2). Determine o momento de inércia em relação ao eixo y = −1,
supondo que a densidade no ponto (x, y) é proporcional à distância do
ponto ao eixo y.

79
Aplicações da Integral de Linha de Campo Escalar

9. Calcule o divergente e o rotacional do seguinte campo:


 

→ y x
a) F (x, y) = − p 2 2 , p 2 2 , (x, y) 6= (0, 0)
x +y x +y


→ −
→ −
→ −

b) F (x, y, z) = yez i + xez j + xyez k
10. Quando o divergente de um campo vetorial é nulo, dizemos que o campo
é solenoidal. Quando o rotacional de um campo vetorial é nulo, dizemos
que o campo é irrotacional. Prove que, o campo − →r /r3 é solenoidal e

→ −
→ −

também irrotacional fora da origem sendo −

r (x, y, z) = x i + y j + z k ,
r = k− →
r k.

80
Cálculo IV – Aula 8

Aula 8
Preparação para a AP1
Queridos alunos de Cálculo IV,
Estamos na semana de preparação para a primeira prova presencial. Espero,
sinceramente, que estejam em dia com a a matéria.
Selecionei alguns exercı́cios para prepará-los para a prova.
Sucesso!!!

Exercı́cios

1. Dada a integral
ZZ Z 0 Z 1 Z 1Z 1
I= f (x, y) dxdy = f (x, y) dxdy + f (x, y) dxdy .
D −1 −y 0 y

a) Esboce a região D.

b) Expresse a soma de integrais do segundo membro como uma só inte-


gral na qual a ordem de integração está invertida.
1
c) Calcule o valor de I para f (x, y) = √ .
1 + x2
2. Seja uma lâmina homogênea com a forma da região



 y ≤ x

 2
x + y2 ≤ 1
D:


 y ≥−x

 x ≥ 0.

Determine
a) O centro de massa de D.
b) O momento de inércia em relação ao eixo x.
3. Calcule a integral dupla passando para coordenadas polares
ZZ p
I= a2 − x2 − y 2 dxdy
D

onde D é a metade superior do disco x2 + y 2 ≤ ax, a > 0.

81
Preparação para a AP1

4. Usando coordenadas polares, calcule


Z 4Z √4y−y2 Z 2Z √4−y2

a) dxdy b) x2 + y 2 dxdy
0 0 0 y

5. Calcule a integral passando para coordenadas esféricas


Z 1Z √1−x2Z √1−x2 −y2 p
I= x2 + y 2 + z 2 dzdydx .
0 0 0

6. Calcule o momento de inércia em relação ao eixo z do sólido homogêneo


W , limitado pelas superfı́cies z = x2 + y 2 , x2 + y 2 = 1 e z = 0.

7. Encontre o momento de inércia em relação ao eixo z do sólido homogêneo


limitado pela esfera x2 + y 2 + z 2 = 4.

8. Seja uma mola que tem a forma de uma hélice circular

C : γ(t) = (4 cos t, 4 sen t, 3t) , 0 ≤ t ≤ 4π .

Sabendo que a densidade em (x, y, z) é dada por δ(x, y, z) = x2 + y 2 ,


calcule
a) A massa M da hélice.
b) O momento de inércia em relação ao eixo z.
Z
9. Calcule xy ds, onde C é a interseção da superfı́cie x + y = 2 com a
C
superfı́cie x2 + y 2 + z 2 = 2(x + y).

10. Encontre o momento de inércia em relação ao eixo z de um fio homogêneo


fino que se encontra ao longo da curva
 √ 
~ ~ 2 2 3/2 ~
γ(t) = (t cos t)i + (t sen t)j + 3 t k,

com 0 ≤ t ≤ 1.

82
Cálculo IV – Aula 9

Aula 9
Integral de Linha de Campo Vetorial
Objetivos
• Definir integrais de linha;
• estudar algumas propriedades;
• estudar uma classe de campos vetoriais que tem a propriedade de que a
integral de linha não depende do caminho;
• cálculo de funções potenciais.

Integral de Linha de Campo Vetorial

Motivação

Considere uma partı́cula que se move ao longo de uma curva



C : γ(t) = x(t), y(t) , t ∈ [a, b], sob a ação de um campo de forças

→ −
→ −

F (x, y) = P (x, y) i + Q(x, y) j . Queremos calcular o trabalho realizado pela


força F quando a partı́cula se desloca de A = γ(a) até B = γ(b).


Da fı́sica, temos que no caso em que F é constante e C é um segmento de reta,

→ −→
o trabalho é dado pelo produto escalar W = F · AB.
No caso geral, dividimos o intervalo [a, b] em n subintervalos [ti−1 , ti ],
i = 1, . . . , n, de mesmo comprimento ∆t = ti − ti−1 . Temos n subarcos
 
γ [ti−1 , ti ] = Ci e n segmentos [Ai−1 , Ai ], Ai = γ(ti ) = x(ti ), y(ti ) , com
i = 1, . . . , n.

Ai
a = t0

Ai+1
ti−1 γ
ti

b = tn

83
Integral de Linha de Campo Vetorial



Supondo F constante ao longo do segmento [Ai−1 , Ai ], o trabalho ao longo de
Ci é aproximadamente igual ao produto escalar


→  −−−−→ −
→ 
Wi ∼
= F γ(ti ) · Ai−1 Ai = F γ(ti ) · (Ai − Ai−1 )
 
= P x(ti ), y(ti ) ∆x + Q x(ti ), y(ti ) ∆y

onde ∆x = x(ti ) − x(ti−1 ) e ∆y = y(ti ) − y(ti−1 ).


Pelo Teorema do Valor Médio, temos ∆x = x′ (t∗i ) ∆t, com t∗i ∈ ]ti−1 , ti [ e
∆y = y ′ (t∗∗ ∗∗
i ) ∆t, com ti ∈ ]ti−1 , ti [ . Logo,
h  ′ ∗  ′ ∗∗ i
Wi ∼
= P x(ti ), y(ti ) x ti + Q x(ti ), y(ti y ti
) ∆t

donde
n h
X    i
W ∼
= P x(ti ), y(ti ) x′ t∗i + Q x(ti ), y(ti ) y ′ t∗∗
i ∆t = Sn .
i=1

Assim, definimos W = lim Sn . Então


∆t→0

Z bh
 ′  ′ i
W = P x(t), y(t) x (t) + Q x(t), y(t) y (t) dt .
a

Esta motivação sugere a definição que se segue.

Definição:
Seja C ⊂ R3 , uma curva regular, dada por uma parametrização γ : [a, b] → R3
  −

de classe C 1 , tal que γ ′ (t) 6= 0, para todo t ∈ a, b . Seja F = (P, Q, R) um


campo vetorial contı́nuo sobre C. Então a integral de linha do campo F ao
Z

→ →
longo de C, denotado por F · d− r , é definida por
C
Z

→ −
F · d→
r =
C
Z b


= F (γ(t)) · γ ′ (t)dt
a
Z bh i
  
= P x(t), y(t), z(t) x′ (t) + Q x(t), y(t), z(t) y ′ (t) + R x(t), y(t), z(t) z ′ (t) dt.
a

84
Cálculo IV – Aula 9

OBS.:

1. Seja C uma curva regular por partes: C = C1 ∪ C2 ∪ . . . ∪ Cn .


Então
Z Z Z

→ − → −
→ − → −
→ −
F ·dr = F · d r + ... + F · d→r
C C1 Cn


→ Z −
→ −
2. A integral de linha de um campo vetorial F , F · d→
r , não
C
depende da parametrização de C, desde que não se inverta sua
orientação. Isto é, denotando por C − a curva C percorrida em
outro sentido, então
Z Z

→ − → −
→ −
F ·dr =− F · d→r
C− C

3. Se C é uma curva fechada (γ(a) = γ(b)) e está orientada no sen-


I

→ −
tido anti-horário, denotamos a integral de linha por F · d→r.
C +
I

→ −
Caso contrário, denotamos por F · d→r.
C−

Exemplo 1

→ −
→ −
→ −
→ −

Seja F (x, y, z) = x i + y j + z k . Temos que a integral de linha F ao longo
de uma hélice C : γ(t) = (cos t, sen t, t), com 0 ≤ t ≤ 2π é dada por

Z Z b

→ − −

F · d→
r = F (γ(t)) · (γ ′ (t)) dt
C a
Z 2π
= (cos t, sen t, t) · (− sen t, cos t, 1) dt
0
Z 2π
= (− cos t sen t + sen t cos t + t) dt
0
Z 2π
= t dt
0
h 2 i2π
t
=
2 0

= 2π 2 .

85
Integral de Linha de Campo Vetorial

Uma Outra Notação

Sabemos que dx = x′ (t) dt, dy = y ′ (t) dt e dz = z ′ (t) dt. Se usarmos a



→ −
→ −

convenção d−

r = dx i + dy j + dz k = (dx, dy, dz), temos
Z

→ −
F · d→
r =
C
Z
= (P, Q, R) · (dx, dy, dz)
C
Z
= P dx + Q dy + R dz
C
Z bh i
  
= P x(t), y(t), z(t) x′ (t) + Q x(t), y(t), z(t) y ′ (t) + R x(t), y(t), z(t) z ′ (t) dt .
a
Z
Logo, uma outra notação é P dx + Q dy + R dz.
C

Exemplo 2
Z 
Calcule y dx + x2 + y 2 dy, onde C é formado pelos segmentos que ligam
C
(−2, 0) a (0, 0) e (0, 0) a (0, 2).
y
Solução:
(0, 2)
O esboço de C = C1 ∪ C2 está re- C2
presentado na figura ao lado.

(−2, 0) (0, 0) x
C1

C1 e C2 podem ser parametrizadas por



 x=t
C1 : , −2 ≤ t ≤ 0, donde dx = dt e dy = 0 .
 y = 0,

 x=0
C2 : , 0 ≤ t ≤ 2, donde dx = 0 e dy = dt .
 y = t,

Temos
Z Z 0
2 2
 
y dx + x + y dy = 0 dt + t2 + 02 = 0
C1 −2
Z Z 2 Z 2 h 3 i2
2 2
 2 2
 t 8
y dx + x + y dy = t·0+ 0 +t dt = t2 dt = = .
C2 0 0 3 0 3

86
Cálculo IV – Aula 9

Logo,
Z
 8 8
y dx + x2 + y 2 dy = 0 + = .
C 3 3

Campos Conservativos


Dizemos que F : D ⊂ Rn → Rn , (n = 2 , 3) é um campo conservativo ou um
campo gradiente se existir um campo escalar diferenciável ϕ : D ⊂ Rn → R,


tal que ∇ϕ = F em D.


O campo escalar ϕ : D ⊂ Rn → R é dito função potencial de F em D.

Exemplo

→ −
→ −
→ −

O campo vetorial F (x, y, z) = (2x + 3yz) i + 3xz j + 3xy k é um campo
conservativo em R3 , pois existe ϕ(x, y, z) = x2 + 3xyz diferenciável em R3 , tal


que ∇ϕ = F em R3 .

A seguir apresentaremos alguns resultados dos campos conservativos.



Teorema 1: Seja F : D ⊂ Rn → Rn , (n = 2 , 3) um campo vetorial de

→ −
→ − →
classe C 1 . Se F é conservativo, então rot F = 0 .

Demonstração:

→ −

Suponhamos n = 3. Então F = (P, Q, R). Se F é conservativo, então existe

→ −
→ −
→ −

ϕ : D ⊂ R3 → R tal que ∇ϕ = F . Logo, rot F = ∇ × F = ∇ × (∇ϕ) = 0
por propriedade dos operadores diferenciais.

Mais adiante veremos um exemplo de um campo vetorial, não conservativo, com


rotacional nulo.

OBS.: O Teorema 1 também pode ser enunciado da seguinte maneira:



→ −
→ −

“Se rot F 6= 0 em D então F não é conservativo em
D” .

87
Integral de Linha de Campo Vetorial

Exemplo 1

→ 2x − → 2y −

Temos que F (x, y) = i + 2 2 j é um campo conservativo em
x2 + y 2 x +y
 −

R2 − {(0, 0)} pois existe ϕ(x, y) = ln x2 + y 2 tal que ∇ϕ = F em
R2 − {(0, 0)}.

Exemplo 2

→ −
→ −

Temos que F (x, y) = −2y i + 2x j não é um campo conservativo. Ora, temos
que 
∂P −

→ → −
→ −
→ − →
∂Q
rot F (x, y) = − k = (2 − (−2)) k = 4 k 6= 0 .
∂x ∂y


→ −

Teorema 2: Seja F : D ⊂ Rn → Rn , (n = 2 , 3) de classe C 2 . Se F


é conservativo, isto é, F = ∇ϕ em D, e se C é qualquer curva regular
por partes com ponto inicial A e ponto final B, então
Z Z

→ − →
F ·dr = ∇ϕ · d−

r = ϕ(B) − ϕ(A) .
C C

Demonstração:
Seja γ(t) , com t ∈ [a, b] uma parametrização de C, de classe C 1 e tal que
γ ′ (t) 6= ~0 em ]a, b[ com γ(a) = A e γ(b) = B. Então
Z Z b Z b
F~ · d~r = F~ (γ(t)) · γ (t) dt =

∇ϕ(γ(t)) · γ ′ (t) dt .
C a a

Pela regra da cadeia temos que

∇ϕ(γ(t)) · γ ′ (t) = (ϕ ◦ γ)′ (t) .

Então:
Z Z b
 b
~
F · d~r = (ϕ ◦ γ)′ (t) dt = ϕ ◦ γ(t) a = ϕ(γ(b)) − ϕ(γ(a)) = ϕ(B) − ϕ(A)
C a

como querı́amos demonstrar.

88
Cálculo IV – Aula 9

Este resultado é conhecido como Teorema Fundamental do Cálculo para Inte-


grais de Linha. É dele que concluı́mos que a integral de linha de um campo
conservativo só depende dos pontos A e B e não depende da trajetória que
os une.


n n
ITeorema 3: Se F : D ⊂ R → R , (n = 2 , 3) é conservativo, então

→ −
F · d→
r = 0 qualquer que seja o caminho fechado.
C

Demonstração:
A demonstração segue do Teorema 2 pois C sendo um caminho fechado, tem-se
que o ponto final B coincide com o ponto inicial A, donde ϕ(B) − ϕ(A) = 0 e,
portanto a integral de linha é zero.

Este Teorema também pode ser enunciado da seguinte maneira:


I

→ − −

”Se F · d→
r 6= 0 para alguma curva fechada C então F não é
C
conservativo” .

Exemplo 3

→ −Z

→ −
→ −

Calcule F · d→ r , onde F (x, y) = x i + y j e C é dada pela equação
C 
γ(t) = arctg t, cos t4 , com 0 ≤ t ≤ 1.
Solução:


Observemos que F é um campo conservativo em R2 com função potencial
1 
ϕ(x, y) = x2 + y 2 . Assim
2
Z

→ −
F · d→r = ϕ (γ(1)) − ϕ (γ(0)) = ϕ(arctg 1, cos 1) − ϕ(arctg 0, cos 0)
C
 
π
= ϕ , cos 1 − ϕ(0, 1)
4
 
1 π2 1 
= + cos2 1 − 02 + 12
2 16 2
 
1 π2
= − 1 + cos2 1 .
2 16


A seguir exibiremos um campo vetorial não conservativo com rotacional 0 , o
que mostra que a recı́proca do Teorema 1 é falsa.

89
Integral de Linha de Campo Vetorial

Exemplo 4

→ −y − → x −→
Seja F (x, y) = i + 2 2 j , (x, y) ∈ D = R2 − {(0, 0)}. Como
x2 + y 2 x +y
∂Q ∂P −
→ − → I

→ −
= (verifique!) então rot F = 0 em D. Calculemos F · d→
r , onde C
∂x ∂y C
é a circunferência γ(t) = (a cos t, a sen t), 0 ≤ t ≤ 2π. Temos
I Z

→ − → −y x
F ·dr = 2 2
dx + 2 2 dy
C Cx +y x +y
Z 2π h    i
−a sen t a cos t
= (−a sen t) + (a cos t) dt
0 a2 a2
Z 2π 
= sen2 t + cos2 t dt
0

= 2π 6= 0 (1)


Se F fosse conservativo, terı́amos encontrado, pelo Teorema 3, que
I

→ − −

F · d→
r = 0, o que contradiz (1). Logo, F não é conservativo.
C+

Na Aula 10, veremos para o caso n = 2, que impondo certas condições ao




domı́nio de F , prova-se que a recı́proca do Teorema 1 é verdadeira.

Cálculo de Funções Potenciais

Exemplo 5

→ 
Sabe-se que F (x, y) = 2xy 2 − y 3 , 2x2 y − 3xy 2 + 2 é um campo gradiente.
Determine uma função potencial.
Solução:
Para determinar uma função potencial ϕ(x, y) devemos ter
∂ϕ
= 2xy 2 − y 3 (2)
∂x
∂ϕ
= 2x2 y − 3xy 2 + 2 (3)
∂y

Integrando (2) em relação a x, temos

ϕ(x, y) = x2 y 2 − xy 3 + f (y) (4)

Integrando (3) em relação a y, temos

ϕ(x, y) = x2 y 2 − xy 3 + 2y + g(x) (5)

90
Cálculo IV – Aula 9

De (4) e (5), vemos que tomando f (y) = 2y e g(x) = 0, segue que uma função
potencial é
ϕ(x, y) = x2 y 2 − xy 3 + 2y .

Exemplo 6

→ −
→ −
→ −

Sabe-se que F (x, y, z) = 2xy i + x2 + z cos(yz) j +y cos(yz) k é um campo
conservativo. Determine uma função potencial.
Solução:
Devemos ter:
∂ϕ
= 2xy (6)
∂x
∂ϕ
= x2 + z cos(yz) (7)
∂y
∂ϕ
= y cos(yz) (8)
∂z
Integrando (6), (7) e (8) em relação a x, y e z respectivamente, temos

ϕ(x, y, z) = x2 y + f (y, z) (9)

ϕ(x, y, z) = x2 y + sen(yz) + g(x, z) (10)

ϕ(x, y, z) = sen(yz) + h(x, y) (11)

De (9), (10) e (11), devemos ter f (y, z) = sen(yz), g(x, z) = 0 e


h(x, y) = x2 y. Logo,
ϕ(x, y, z) = x2 y + sen(yz)


é uma função potencial de F .

91
Integral de Linha de Campo Vetorial

Exercı́cios

1. Um objeto percorre uma elipse 4x2 + 25y 2 = 100 no sentido anti-horário




e se encontra submetido à força F (x, y) = (−3y, 3x). Ache o trabalho
realizado.

2. Calcule o trabalho realizado pelo campo de forças F (x, y) = x2 −

y 2 , 2xy ao mover uma partı́cula ao longo da fronteira do quadrado li-
mitado pelos eixos coordenados e pelas retas x = a e y = a, a > 0 no
sentido anti-horário.
I

→ − −

F · d→

3. Calcule r para F (x, y) = x2 , x + y onde C é a fronteira do
C+
triângulo de vértices (0, 0), (1, 0) e (1, 1), orientada no sentido anti-horário.
Z
4. Calcule zdx+ydy −xdz, onde C é a interseção das superfı́cies y +z = 8
C
e x2 + y 2 + z 2 − 8z = 0, orientada no sentido anti-horário quando vista de
cima (ou equivalentemente, no sentido horário quando vista da origem).


5. Calcule o trabalho realizado pelo campo de forças F (x, y, z) = (y, z, x)
ao mover uma partı́cula ao longo da curva C interseção das superfı́cies
x + y = 2 e x2 + y 2 + z 2 = 2(x + y), percorrida uma vez no sentido
anti-horário quando vista da origem.
Z
6. Calcule −2y dx + 3z dy + x dz, sendo C a interseção das superfı́cies
C
x + 4y = 1 e x2 + z 2 = 1, com y ≥ 0 e z ≥ 0, percorrida uma vez do
2 2

ponto (1, 0, 0) ao ponto (−1, 0, 0).

7. Ache o trabalho que realiza a força



→ −
→ −

F (x, y) = y sen xy − y 2 − 3 i + (x sen xy − 2xy) j

para transladar uma partı́cula de (4, 0) a (0, 0) ao longo da curva


C: y = 2 − |2 − x|, com 0 ≤ x ≤ 4.

8. .
a) Determine uma função potencial para o campo gradiente

→ −→ −→ −

F (x, y, z) = −4xe3y + zexz i + −6x2 e3y + 4y 2 j +(xexz + cos z) k .
Z

→ −
b) Calcule F · d→
r , se C é a curva descrita por
C


→ → π −
− →
γ(t) = (t − 1)(t − 2) i + t j + t5 k ,
2
com 0 ≤ t ≤ 1.

92
Cálculo IV – Aula 9

Z (1,4)
9. Mostre que 2xy dx + x2 dy é independente do caminho e calcule a
(2,−1)
integral.
Z

→ −
→ 
10. Calcule F · d~r, onde F (x, y, z) = 2x sen z, z 3 − ey , x2 cos z + 3yz 2
C 
e C : γ(t) = t, t(t − 1), t2 + 1 , com 0 ≤ t ≤ 1.

93
Integral de Linha de Campo Vetorial

94
Cálculo IV – Aula 10

Aula 10
Teorema de Green
Objetivos
• Estudar um teorema que estabelece uma ligação importante entre integrais
de linha e integrais duplas;


• estudar condições sobre o domı́nio de F para que valha a recı́proca do
Teorema 1, da aula 16, isto é, em que domı́nios, campos de rotacional
nulo são conservativos?

O Teorema de Green

Teorema: Seja D uma região fechada e limitada de R2 , cuja fronteira


∂D é formada por um número finito de curvas simples, fechadas e C 1
por partes, duas a duas disjuntas, orientadas no sentido que deixa D à
esquerda das curvas, (isto é, ∂D está orientada positivamente). Seja

→ −

F = P (x, y) i + Q(x, y) j um campo vetorial de classe C 1 em um
conjunto aberto U com D ⊂ U . Então
I I ZZ  

→ − → ∂Q ∂P
F ·dr = P dx + Q dy = − dxdy
∂D+ ∂D+ D ∂x ∂y

C1

D
C2
C3

C4

No caso, ∂D = C1 ∪ C2 ∪ C3 ∪ C4 e
I I I I I

→ − → −
→ − → −
→ − → −
→ − −
→ −
F ·dr = F ·dr + F ·dr + F · d→
r + F · d→
r .
∂D+ C1+ C2− C3− C4−

OBS.: Geralmente, usamos o Teorema de Green, quando


I

→ −
F · d→r é difı́cil de ser calculada diretamente.
C+

95
Teorema de Green

Exemplo 1

→ −
→ −

Seja F (x, y) = (2x + y) i + (3y + 4x) j . Vamos calcular as duas integrais do
enunciado do Teorema de Green, para D a região triangular de vértices (0, 0),
(1, 0) e (0, 1).

B = (0, 1)

D x+y =1

O A = (1, 0) x

Temos ∂D = OA ∪ AB ∪ BO.
Z

→ −
Cálculo de F · d→
r
OA

Temos OA : y = 0, 0 ≤ x ≤ 1, donde dy = 0. Então


Z Z Z 1

→ − →  1
F ·dr = P (x, 0) dx = 2x dx = x2 0 = 1 .
OA OA 0

Z

→ −
Cálculo de F · d→
r
AB

Temos AB : x = 1 − y, 0 ≤ y ≤ 1, donde dx = −dy. Então


Z Z

→ − →
F ·dr = P (1 − y, y) (−dy) + Q(1 − y, y) dy
AB AB
Z 1    
= − 2(1 − y) + y dy + 3y + 4(1 − y) dy
0
Z 1
= (−2 + 2y − y + 3y + 4 − 4y) dy
0
Z 1
= 2 dy
0
h i1
= 2y
0

= 2.

96
Cálculo IV – Aula 10

Z Z

→ − → −
→ −
Cálculo de F ·dr =− F · d→
r
BO OB

Temos OB : x = 0, 0 ≤ y ≤ 1, donde dx = 0. Então


Z Z Z 1 h 2 i1

→ − → 3y 3
F ·dr =− Q(0, y) dy = − (3y + 0) dy = − =− .
BO OB 0 2 0 2

Somando, temos I

→ − 3 3
F · d→
r =1+2− = .
∂D+ 2 2
Por outro lado,
ZZ   ZZ
∂Q ∂P 1 3
− dxdy = (4 − 1) dxdy = 3A(D) = 3 · · 1 · 1 = .
D ∂x ∂y D 2 2

Exemplo 2

→ −
→ −

Seja F (x, y) = −x2 y i +xy 2 j e D o disco de centro (0, 0) e raio 1. Calculemos
I

→ −
F · d→r , para ∂D orientada no sentido anti-horário.
∂D +

Solução:
Do Teorema de Green, temos
I ZZ   ZZ

→ − → ∂Q ∂P 
F ·dr = − dxdy = y 2 + x2 dxdy .
∂D+ D ∂x ∂y D

y
1

∂D
D
1 x

Passando para coordenadas polares, temos





 x = r cos θ

y = r sen θ


 dxdy = rdrdθ
 2
x + y 2 = r2

e Drθ é dado por (


0≤r≤1
Drθ :
0 ≤ θ ≤ 2π

97
Teorema de Green

Então I Z Z

→ −
F · d→
r = 2
r · r drdθ = r3 drdθ
∂D+ Drθ Drθ
Z 1 Z 2π
3
= r dθdr
0 0
Z 1
= 2π r3 dr
0
h 4 i1
r
= 2π
4 0

π
= .
2

Exemplo 3

→ −y −→ x − →
Seja F (x, y) = 2 2 i + 2 2 j definido em D = R2 − {(0, 0)}. Calcule-
x +y x +y
mos:
I

→ −
a) F · d→r , sendo C1 : x2 + y 2 = a2 , a > 0;
C1+
I

→ −
b) F · d→
r , sendo C2 uma curva fechada, C 1 por partes, que envolve a
C2+
origem.

Solução:
a) Observemos que a região limitada por C1 não está contida em D, pois
(0, 0) ∈
/ D. Então não podemos aplicar o Teorema de Green. Sendo assim,
usaremos a definição. Parametrizando C1 , temos x = a cos t e y = a sen t, com
0 ≤ t ≤ 2π donde dx = −a sen t dt e dy = a cos t dt. Então
I I

→ − → −y x
F ·dr = 2 2
dx + 2 2 dy
C1+ C1+ x +y x +y
Z 2π h i
−a sen t a cos t
= 2
(−a sen t) + 2 (a cos t) dt
0 a a
Z 2π 
= sen2 t + cos2 t dt
0
Z 2π
= dt
0

= 2π .

98
Cálculo IV – Aula 10

b)

C2

Aqui também não podemos aplicar o Teorema de Green, pois (0, 0) está na região
limitada por C2 e (0, 0) ∈
/ D. Usar a definição é impossı́vel pois nem conhecemos
uma equação de C2 . Então o que fazer?

y
A idéia é de isolar (0, 0)
por uma circunferência
a C2 C1 : x2 + y 2 = a2 com o
C1
raio a adequado de modo que
a x C1 esteja no interior da região
R
limitada por C2 , orientada no
sentido horário.

Seja R a região limitada por C1 e C2 . Logo, ∂R = C2 ∪ C1 . Como R não


contém (0, 0), então podemos aplicar o Teorema de Green em R. Temos
I ZZ  

→ − → ∂Q ∂P
F ·dr = − dxdy .
∂R+ R ∂x ∂y
  I
∂Q ∂P −

Como −= 0 (Verifique!) então F · d−
→r = 0. Logo,
I ∂x ∂y
I I ∂R +
I

→ −
→ −
→ −

F · d−

r + F · d−→
r = 0 donde F · d−→r − F · d−

r = 0
+ − + +
C2I CI
1 C2 C1

→ − −
→ −
ou F · d→
r = F · d→
r = 2π por (a).
C2+ C1+

Exemplo 4
a) Se D é uma região plana qualquer à qual
I se aplica o Teorema de
I Green,
mostre que a área de D é dada por A(D) = − y dx ou A(D) = x dy
I ∂D+ ∂D+
1
ou A(D) = − y dx + x dy.
2 C

99
Teorema de Green

b) Aplique uma das fórmulas acima para mostrar que a área limitada pela elipse
x2 y2
2
+ 2 = 1 é πab.
a b

Solução:
a)

Pelo Teorema de Green, tem-se


I I ZZ  
∂0 ∂(−y)
− y dx = − y dx + 0 dy = − dxdy
∂D+ ∂D+ D ∂x ∂y
ZZ
= (0 + 1) dxdy
D
ZZ
= dxdy
D

= A(D) .

I
Logo, A(D) = − y dx. Analogamente, prova-se as outras fórmulas.
∂D+

b) O esboço de D está representado na figura que se segue.

b
∂D
D
a x

I
A área de D é dada por A(D) = − y dx onde ∂D é parametrizada por
∂D+

(
γ(t) = (a cos t, b sen t)
, 0 ≤ t ≤ 2π .
γ ′ (t) = (−a sen t, b cos t)

100
Cálculo IV – Aula 10

Então
Z 2π Z 2π
A(D) = (−b sen t)(−a sen t) dt = ab sen2 t dt
0 0
h i2π
1 sen 2t
= ab · t−
2 2 0

1
= ab · · 2π
2

= πab u.a.

Teorema das Quatro Equivalências

Condições sobre D

(i) D é aberto.

(ii) D é conexo (isto é, dois pontos quaisquer de D podem ser ligados por uma
curva contida em D).

(iii) D é “sem buracos” (isto é, qualquer curva fechada de D delimita uma
região inteiramente contida em D).

Um conjunto satisfazendo as condições (i), (ii) e (iii) é dito um conjunto sim-


plesmente conexo. A seguir daremos exemplos de conjuntos simplesmente
conexos.
111111111111111
000000000000000
y
000000000000000
111111111111111 y 11111111
00000000 y
000000000000000
111111111111111 00000000
11111111
00000000
11111111
000000000000000
111111111111111 00000000
11111111 1111111111
0000000000
000000000000000
111111111111111
D = R2 D
00000000
11111111 0000000000
1111111111
0000000000
1111111111
000000000000000
111111111111111 00000000
11111111 0000000000
1111111111
D
000000000000000
111111111111111
000000000000000
111111111111111 00000000
11111111 0000000000
1111111111
000000000000000
111111111111111 00000000
11111111 0000000000
1111111111
000000000000000
111111111111111 x x 0000000000
1111111111
0000000000
1111111111
x
000000000000000
111111111111111 0000000000
1111111111
000000000000000
111111111111111
000000000000000
111111111111111 0000000000
1111111111
000000000000000
111111111111111
000000000000000
111111111111111

Agora, daremos exemplos de conjuntos não simplesmente conexos.

101
Teorema de Green

111111111111111
000000000000000
y y 111111111111111
000000000000000
y
000000000000000
111111111111111 000000000000000
111111111111111
000000000000000
111111111111111
000000000000000
111111111111111 1111111111
0000000000 000000000000000
111111111111111
000000000000000
111111111111111
000000000000000
111111111111111
D = R2 − {(0, 0)} 0000000000
1111111111 000000000000000
111111111111111
000000000000000
111111111111111 0000000000
1111111111
D
0000000000
1111111111 000000000000000
111111111111111
000000000000000
111111111111111 0000000000
1111111111 000000000000000
111111111111111
000000000000000
111111111111111
000000000000000
111111111111111 0000000000
1111111111 000000000000000
111111111111111
000000000000000
111111111111111
000000000000000
111111111111111x 0000000000 x
1111111111 000000000000000
111111111111111 x
000000000000000
111111111111111 0000000000
1111111111
0000000000
1111111111 000000000000000
111111111111111
000000000000000
111111111111111 0000000000
1111111111 000000000000000
111111111111111
000000000000000
111111111111111
000000000000000
111111111111111 000000000000000
111111111111111
000000000000000
111111111111111
D = R2 − eixo x
000000000000000
111111111111111 000000000000000
111111111111111

OBS.: Seja D ⊂ R3 . Dizemos que D é um conjunto sim-


plesmente conexo se D é aberto, conexo e “sem buracos”
(no sentido de que qualquer curva fechada de D delimita
uma superfı́cie inteiramente contida em D).

Exemplo
O R3 , uma bola aberta em R3 , o R3 − {(0, 0, 0)} são conjuntos simplesmente
conexos. O R3 sem uma reta não é simplesmente conexo.



Teorema 1: Seja F um campo de classe C 1 em um domı́nio D de R2 ,

→ − → −

simplesmente conexo. Se rot F = 0 então F é conservativo.

Demonstração
Z
Do Teorema de Green segue que F~ · d~r = 0, para todo caminho fechado de
C
D. Tomemos dois pontos A e B quaisquer de D e sejam C1 e C2 caminhos C 1
por partes ligando A e B.

C2 AB

A C1
Z
Seja o caminho fechado C = C1 ∪ C2− . Logo, temos que F~ · d~r = 0 ou
Z Z Z Z Z C Z
F~ ·d~r + −
F~ ·d~r = 0 ou F~ ·d~r − F~ ·d~r = 0, donde F~ ·d~r = F~ ·d~r.
C1 C2 Z C1 C2 C1 C2

Isto mostra que F~ · d~r não depende do caminho.


C

102
Cálculo IV – Aula 10

Então, fixemos um ponto


Z A = (a1 , a2 ) em D e para cada X = (x, y) ∈ D,
definimos ϕ(x, y) = F~ · d~r onde C e D é um caminho C 1 por partes ligando
C
A a X. Como essa integral não depende do caminho C escolhido, ϕ é uma função
bem definida. Finalmente, mostra-se sem grandes dificuldades que ∇ϕ = F~ em
D e, portanto, o teorema está demonstrado.

Do Teorema 1 e de teoremas da aula 16, enunciamos um teorema contendo


quatro equivalências.


Teorema das quatro equivalências: Seja F = (P, Q) : D ⊂ R2 → R2 um campo
de classe C 1 em D. Se D ⊂ R2 é um conjunto simplesmente conexo, então as
seguintes afirmações são equivalentes:
∂Q ∂P
a) = em D.
∂x ∂y
I

→ −
b) F · d→
r = 0 qualquer que seja a curva fechada C de D.
C
Z

→ −
c) F · d→
r não depende do caminho C de D.
C



d) F é conservativo.

Exemplo 1
 
π
Considere a curva C dada por σ(t) = − cos , et−1 , 1 ≤ t ≤ 2. Calcule
Z

→ − → −
→ 2
t
F · d r , onde F (x, y) = −y sen x, 2y cos x .
C

Solução:


Como F é de classe C 1 em R2 (que é um conjunto simplesmente conexo) e
∂Q ∂P
= −2y sen x = , então pelo teorema das quatro equivalências, segue que
Z∂x ∂y

→ −
F · d→ r não depende do caminho que liga σ(1) = (1, 1) e σ(2) = (0, e).
C
Então considere C = C1 ∪ C2 , onde C1− : y = 1, 0 ≤ x ≤ 1, donde dy = 0 e
C2 : x = 0, 1 ≤ y ≤ e, donde dx = 0.

103
Teorema de Green

(0, e)
C2

C1
(1, 1)

Temos
Z Z Z Z 1

→ − → −
→ − →
F ·dr =− F ·dr =− P (x, 1) dx = − (− sen x) dx
C1 C1− C1− 0
Z 1
= sen x dx
0
1

= − cos x
0

= 1 − cos 1
Z Z Z e Z e e

→ −
F · d→
r = Q(0, y) dy = 2y cos 0 dy = 2
2y dy = y = e2 − 1 .
C2 C2 1 1 1

Logo, Z

→ −
F · d→
r = 1 − cos 1 + e2 − 1 = e2 − cos 1 .
C

Uma Solução Alternativa




Pelo teorema das quatro equivalências segue que F é conservativo. Logo, existe
ϕ(x, y) definido em R2 , tal que

∂ϕ
= −y 2 sen x (1)
∂x
∂ϕ
= 2y cos x (2)
∂y

Integrando (1) e (2) em relação a x e y respectivamente, temos

ϕ(x, y) = y 2 cos x + f (y)

ϕ(x, y) = y 2 cos x + g(x) .

104
Cálculo IV – Aula 10

Tomando f (y) = 0 e g(x) = 0, temos que ϕ(x, y) = y 2 cos x é uma função




potencial de F . Logo,
Z

→ −
F · d→
r = ϕ (σ(2)) − ϕ (σ(1)) = ϕ(0, e) − ϕ(1, 1)
C

= e2 cos 0 − 12 cos 1

= e2 − cos 1 .

Exemplo 2
Z

Considere a integral de linha (kxey + y) dx + x2 ey + x − ky dy.
C
a) Determine a constante k para que esta integral seja independente do caminho.

b) Calcule o valor da integral de A = (0, 0) a B = (1, 1) para o valor de k


encontrado em (a).

Solução:


a) O campo F é definido em R2 que é um conjunto simplesmente conexo.

→ − →
Pelo teorema das quatro equivalências é necessário que rot F = 0 para que a
integral independa do caminho. Então

→ − → ∂Q ∂P
rot F = 0 ⇔ = em R2
∂x ∂y

⇔ 2xey + 1 = kxey + 1

⇔ 2xey = kxey

⇔ 2x = kx pois ey 6= 0 para todo y ∈ R

⇔ k = 2.


→ −

Portanto, para k = 2 segue que rot F = 0 , donde pelo teorema das equi-
valências temos que a integral independe do caminho.
b) Temos que

→ −
→ −

k = 2 ⇒ F (x, y) = (2xey + y) i + x2 ey + x − 2y j .

105
Teorema de Green

1 B = (1, 1)

C2

A = (0, 0) 1 x
C1

Como a integral independe do caminho, tomemos C = C1 ∪C2 , onde C1 : y = 0,


com 0 ≤ x ≤ 1 donde dy = 0 e C2 : x = 1, com 0 ≤ y ≤ 1, donde dx = 0.
Temos
Z Z Z 1 Z 1 h i1

→ −
F · d→
r = P (x, 0) dx = 0
2xe dx = 2x dx = x2 = 1
C1 C1 0 0 0
Z Z Z 1 h i1

→ −
F · d→
r = Q(1, y) dy = (ey + 1 − 2y) dy = ey + y − y 2 = e − 1 .
C2 C2 0 0

Somando temos, Z

→ −
F · d→
r = 1 + e − 1 = e.
C

Uma Solução Alternativa




Também do teorema das equivalências resulta que F é conservativo, isto é,
existe ϕ(x, y) definido em R2 , tal que

∂ϕ
= 2xey + y (3)
∂x
∂ϕ
= x2 ey + x − 2y (4)
∂y

Integrando (3) e (4) em relação a x e y respectivamente, temos

ϕ(x, y) = x2 ey + xy + f (y)

ϕ(x, y) = x2 ey + xy − y 2 + g(x) .

Devemos tomar f (y) = −y 2 e g(x) = 0. Assim ϕ(x, y) = x2 ey + xy − y 2 é uma




função potencial de F . Logo,
Z

→ −
F · d→ r = ϕ(B) − ϕ(A) = ϕ(1, 1) − ϕ(0, 0) = e + 1 − 1 − 0 + 0 − 0 = e .
C

106
Cálculo IV – Aula 10

Exercı́cios

1. Verifique o teorema de Green, calculando as duas integrais do enunciado


2 
para P (x, y) = xy 3 − x2 y , Q(x, y) = x2 y 2 , D é o triângulo de vértices
3
(0, 0), (1, 0) e (1, 1).
I
2. Calcule x−1 ey dx + (ey ln x + 2x) dy, onde C é a fronteira da região
C
limitada por x = y 4 + 1 e x = 2, orientada no sentido anti-horário.

3. Calcule
Z p 
I= (y − x + arctg x) dx + 2x − y + 1 + y2 dy ,
C

sabendo que C é a fronteira da região limitada pelas curvas y = x + 2 e


y = x2 , orientada positivamente.
x2 y2
4. Se D é a região interior à elipse + = 1 e exterior à circunferência
25 9
2 2
x + y = 4, calcule a integral de linha
Z  
2 
I= 2xy + ex dx + x2 + 2x + cos y 2 dy
C

onde C = ∂D está orientada positivamente.

5. Calcule Z (0,0)
−xy(1 + x)−1 dx + ln(1 + x) dy ,
(4,0)

onde C é formada por x + 2y = 4 e x = 0.

6. Seja  
→ y−1 −x
F (x, y) = − y , ,
x2 + (y − 1)2 x2 + (y − 1)2

com (x, y) 6= (0, 1). Calcule a integral de linha do campo F ao longo de
C1 e C2 , orientadas no sentido anti-horário, onde

a) C1 : x2 + (y − 1)2 = 1
b) C2 : x2 + y 2 = 16

7. Calcule as integrais
Z
x y t

a) 2 2
dx+ dy, sendo C : γ(t) = e , sen t , 0≤t≤π
Cx +y x2 + y 2
Z
2 
b) 7x6 y dx + x7 dy, sendo C : γ(t) = t, et −1 , 0 ≤ t ≤ 1
C

107
Teorema de Green

8. Mostre que a seguinte integral independe do caminho e calcule seu valor:


Z (3,4)
2xy dx − x2 dy
I= .
(1,−2) x4

9. Mostre que a seguinte integral de linha


Z

I = (3 + 2xy) dx + x2 − 3y 2 dy
C

é independente do caminho e calcule o valor de I quando C é a curva dada


por

→ −

σ(t) = et sen t i + et cos t j , 0 ≤ t ≤ π .

10. Seja C uma curva simétrica em relação ao eixo y, que vai de (4, 0) a
(−4, 0), como mostrada na figura que se segue. Sabendo-se que a área da
região delimitada por C
 e2 pelo eixo
 x vale 16, calcule o trabalho realizado

→ x −
→ −

pela força F (x, y) = + xy 3 i + (2x + arctg y) j .
4
y

(−4, 0) (4, 0) x

108
Cálculo IV – Aula 11

Aula 11
Superfı́cies Parametrizadas
Objetivos

• Estudar as superfı́cies parametrizadas visando as integrais de superfı́cie;

• estudar as áreas de superfı́cies parametrizadas.

Superfı́cies Parametrizadas

Definição: Dizemos que S ⊂ R3 é uma superfı́cie parametrizada se existir


uma função vetorial contı́nua

ϕ : D ⊂ R2 −→ R3

(u, v) 7−→ ϕ(u, v) = x(u, v), y(u, v), z(u, v)

tal que S = ϕ(D).


S = ϕ(D)
v z

ϕ(u, v)
(u, v) ϕ
D

u
y

As funções x = x(u, v), y = y(u, v), z = z(u, v) são chamadas equações


paramétricas de S.
Se ϕ for diferenciável em (u0 , v0 ) ∈ D, fixando v = v0 , obtemos uma curva
diferenciável

C1 : γ(u) = ϕ(u, v0 ) = x(u, v0 ), y(u, v0 ), z(u, v0 )

(Veja a figura a seguir).

109
Superfı́cies Parametrizadas

∂ϕ
∂ϕ (u0 , v0 )
(u0 , v0 ) ∂u
∂v
v z S = ϕ(D)

ϕ(u0 , v0 ) C1
D
ϕ
(u0 , v0 )

C2
u
y

Se
∂ϕ
γ ′ (u0 ) = (u0 , v0 ) = ϕu (u0 , v0 )
∂u
  − →
∂x ∂y ∂z
= (u0 , v0 ) , (u0 , v0 ) , (u0 , v0 ) 6= 0
∂u ∂u ∂u
segue que ϕu (u0 , v0 ) é um vetor tangente a C1 em ϕ(u0 , v0 ).
Analogamente, se
  −

∂ϕ ∂x ∂y ∂z
(u0 , v0 ) = ϕv (u0 , v0 ) = (u0 , v0 ) , (u0 , v0 ) , (u0 , v0 ) 6= 0
∂v ∂v ∂v ∂v
então este vetor é um vetor tangente a C2 em ϕ(u0 , v0 ).
Se o vetor

→ − → ∂ϕ ∂ϕ −

N = N (u0 , v0 ) = (u0 , v0 ) × (u0 , v0 ) = ϕu (u0 , v0 ) × ϕv (u0 , v0 ) 6= 0
∂u ∂v



→ N
então N é um vetor normal a S em ϕ(u0 , v0 ). O vetor −

→ é um vetor
n = −
kN k
normal unitário a S em ϕ(u0 , v0 ).

→ −

Dizemos que S é regular em ϕ(u0 , v0 ) se N (u0 , v0 ) 6= 0 . O plano tangente a
S em ϕ(u0 , v0 ) é dado por
  −→
(x, y, z) − ϕ(u0 , v0 ) · N (u0 , v0 ) = 0 .
Apresentaremos agora, parametrizações das principais superfı́cies:

Plano S


Sejam P0 ∈ S, − →
a e b não paralelos contido no plano S. Seja P ∈ S. Então,
existem escalares u e v, tais que:
−−→ −
→ −

P0 P = u− →
a + v b ⇔ P = P0 + u− →a +v b .

110
Cálculo IV – Aula 11

Então, uma parametrização de S é dada por:




ϕ(u, v) = P0 + u− →a + v b com (u, v) ∈ R2 .

Da Geometria Analı́tica, vê-se que um vetor normal a S em P0 é:



→ − −

N =→ a × b .

S −

N

P0 −

b


a

S = gráfico de z = f (x, y), com (x, y) ∈ D e f (x, y) de classe C 1



N S : z = f (x, y)

y
D

Uma parametrização natural (ou canônica) de S = Gf (gráfico de f ) é dada por



ϕ(x, y) = x, y, f (x, y) , com (x, y) ∈ D .
Um vetor normal é dado por:

→ ∂ϕ ∂ϕ
N = (x, y) × (x, y) = ϕx (x, y) × ϕy (x, y)
∂x ∂y

= − fx (x, y), −fy (x, y), 1 Verifique!

→ −

Como N (x, y) 6= 0 , para todo (x, y) ∈ D, segue que S = Gf é uma superfı́cie
regular.

111
Superfı́cies Parametrizadas

Cilindro x2 + y 2 = a2 , a > 0

Utilizamos as coordenadas cilı́ndricas para parametrizar um cilindro de raio a.


Tem-se: 
 x = a cos θ

y = a sen θ

 z=z

Então: ϕ(θ, z) = (a cos θ, a sen θ, z) com (θ, z) ∈ D : 0 ≤ θ ≤ 2π , z ∈ R é


uma parametrização de S.

→ −

Verifique que N = ϕθ (θ, z)×ϕz (θ, z) = (a cos θ, a sen θ, 0). Logo, N = (x, y, 0)
(x, y, 0)
é um vetor normal exterior a S em cada (x, y, z) ∈ S, donde − →n =
a
é o vetor unitário normal exterior a S.

Esfera x2 + y 2 + z 2 = a2 , a > 0

Utilizamos as coordenadas esféricas para parametrizar a esfera. Tem-se:



 x = a sen φ cos θ

y = a sen φ sen θ

 z = a cos φ

com 0 ≤ φ ≤ π e 0 ≤ θ ≤ 2π. Então:

ϕ(φ, θ) = (a sen φ cos θ, a sen φ sen θ, a cos φ)

com (
0≤φ≤π
(φ, θ) ∈ D :
0 ≤ θ ≤ 2π .

Verifique que:


→ ∂ϕ ∂ϕ 
N = × = a2 sen2 φ cos θ, a2 sen2 φ sen θ, a2 sen φ cos φ
∂φ ∂θ



N = a2 sen φ .

Logo:



→ N ϕ(φ, θ) (x, y, z)
n = −
→ = = ,
N a a

(x, y, z)
isto é, −

n = é o vetor unitário normal exterior à esfera.
a

112
Cálculo IV – Aula 11

Superfı́cie de Revolução S

a) Seja C uma curva no plano yz dada por



 x=0

C: y = y(t)

 z = z(t)

com a ≤ t ≤ b e y(t) ≥ 0 em [a, b].



Ao girar o ponto 0, y(t), z(t) ao
redor do eixo z, na altura z(t), ob-
temos uma circunferência de raio
y(t), parametrizada por

y(t) cos θ, y(t) sen θ, z(t) ,
z

com 0 ≤ θ ≤ 2π. C
Fazendo t variar de a até b,
a circunferência começa a S
` ´
0, y(t), z(t)
de deslocar segundo a altura
(x, y, z)
z = z(t), gerando a superfı́cie
de revolução S da figura ao lado.
z
Tem-se:
x y
 θ
S : ϕ(t, θ) = y(t) cos θ, y(t) sen θ, z(t) y

onde x
(
a≤t≤b
(t, θ) ∈ D :
0 ≤ θ ≤ 2π .

Observe que na superfı́cie S tem-se:

y(t) = raio de uma circunferência transversal

z(t) = altura desta circunferência.

b) Se C é uma curva no plano xz dada por



 x = x(t)

C: y=0

 z = z(t)

com a ≤ t ≤ b, então:

113
Superfı́cies Parametrizadas

x(t) = raio de uma circunferência transversal

z(t) = altura dessa circunferência.

Logo, uma parametrização da superfı́cie de revolução S obtida girando C ao


redor do eixo z é

S : ϕ(t, θ) = x(t) cos θ, x(t) sen θ, z(t)

com (
a≤t≤b
(t, θ) ∈ D :
0 ≤ θ ≤ 2π .

Área de Superfı́cie

Seja S uma superfı́cie parametrizada por ϕ(u, v) = (x(u, v), y(u, v), z(u, v)) ∈ D
onde D é um conjunto compacto que tem área e com ϕ de classe C 2 em um
conjunto aberto contendo D. É necessário também que ϕ seja uma função
injetora exceto possivelmente na fronteira de D e que S seja regular exceto em
um número finito de pontos.
Daqui por diante, até o final do curso, trabalharemos somente com superfı́cies
descritas acima.
Definimos a área de S por
ZZ
∂ϕ ∂ϕ
A(S) = (u, v) × (u, v) dudv .
D ∂u ∂v

v z
S = ϕ(D)

D ϕ

u
y

114
Cálculo IV – Aula 11

OBS.: Se S for o gráfico de uma função de classe C 1 ,


z = f (x, y), (x, y) ∈ D, onde D é um conjunto compacto
que tem área, então:
ZZ q
A(S) = 1 + (fx )2 + (fy )2 dxdy .
D

S = Gf (gráfico de S)

y
D
x

Exemplo 1
Mostre que a área da esfera S : x2 +y 2 +z 2 = a2 , a > 0 é dada por A(S) = 4πa2 .
Solução:
Usando as coordenadas esféricas com ρ = a, para parametrizar a esfera, tem-se

ϕ(φ, θ) = (a sen φ cos θ, a sen φ sen θ, a cos φ)

com
(
0≤φ≤π
(φ, θ) ∈ D : .
0 ≤ θ ≤ 2π

∂ϕ ∂ϕ
Calculemos ϕφ × ϕθ (φ, θ) = × (φ, θ) e seu módulo.
∂φ ∂θ
Tem-se

ϕφ = (a cos φ cos θ, a cos φ sen θ, −a sen φ)

ϕθ = (−a sen φ sen θ, a sen φ cos θ, 0)

115
Superfı́cies Parametrizadas

donde

ϕφ × ϕθ =


→ −
→ −


i j k

= a cos φ cos θ a cos φ sen θ −a sen φ


−a sen φ sen θ a sen φ cos θ 0

= a2 sen2 φ cos θ, a2 sen2 φ sen θ, a2 sen φ cos φ cos2 θ + a2 sen φ cos φ sen2 θ
| {z }
=a2 sen φ cos φ

= a sen φ (a sen φ cos θ, a sen φ sen θ, a cos φ)

= (a sen φ) · ϕ(φ, θ) .

Esta última expressão mostra que o vetor normal em cada ponto da esfera é
radial, isto é, é um múltiplo do vetor posição ϕ (φ, θ).
Tem-se

kϕφ × ϕθ (φ, θ)k = |a sen φ| kϕ (φ, θ)k = a2 |sen φ| = a2 sen φ

pois 0 ≤Z Zφ ≤ π, isto é, kϕφ × ϕθ k = a2 sen φ (memorize este resultado). Como


A(S) = kϕφ × ϕθ k dφdθ então
D

ZZ Z πZ 2π
2 2
A(S) = a sen φ dφdθ = a sen φ dθdφ
D 0 0
Z π
2
= 2πa sen φ dφ
0

 π
= 2πa2 − cos φ 0

= 4πa2 u.a.

Exemplo 2
p
Calcule a área da superfı́cie z = x2 + y 2 , 0 ≤ z ≤ 1.
Solução:
O esboço da superfı́cie S está representado na figura que se sege.

116
Cálculo IV – Aula 11

1 y
1

D
x
p
Temos que S : z = x2 + y 2 , (x, y) ∈ D : x2 + y 2 ≤ 1. Também temos
| {z }
f (x,y)
x y
fx = p e fy = p . Logo,
x2 + y2 x2 + y2
r r
q
2 2 x2 y2 x2 + y 2 √
1 + (fx ) + (fy ) = 1+ 2 2 + 2 2 = 1+ = 2.
x +y x +y x2 + y 2

Como ZZ q
A(S) = 1 + (fx )2 + (fy )2 dxdy
D
então ZZ √ √ ZZ
A(S) = 2 dxdy = 2 dxdy
D D

= 2 · A(D)

= 2 · π · 12

= π 2 u.a.

117
Superfı́cies Parametrizadas

Exercı́cios

1. Esboce e parametrize as superfı́cies abaixo, indicando o domı́nio dos parâmetros:

a) S = {(x, y, z) ∈ R3 ; x2 + y 2 + z 2 = 4, z ≥ 1}.
p
b) S = {(x, y, z) ∈ R3 ; z = 3(x2 + y 2 ), x2 + y 2 + z 2 ≤ 1}.
c) S = {(x, y, z) ∈ R3 ; x + y + z = 2, x2 + y 2 ≤ 1}.
n x yo
d) S = (x, y, z) ∈ R3 ; x2 + y 2 = 4, 0 ≤ z ≤ 2 + − .
4 2
e) S = {(x, y, z) ∈ R3 ; x2 + y 2 = 2y, 0 ≤ z ≤ x2 + y 2 }.

2. Seja S uma superfı́cie parametrizada por



ϕ(u, v) = v cos u, v sen u, 1 − v 2 ,

com 0 ≤ u ≤ 2π e v ≥ 0.
a) Identifique esta superfı́cie.
b) Encontre uma equação da reta normal e a equação do plano tangente
a S em ϕ(0, 1).

3. Seja S a superfı́cie parametrizada por ϕ(u, v) = u, v, 1 − v 2 , u ≥ 0,
v ≥ 0 e u + v ≤ 1.
a) Desenhe S.
b) Determine o plano tangente a S no ponto ϕ (1/2, 1/4).
c) Determine a área de S.
4. Seja S a superfı́cie obtida girando-se a circunferência C : (x−a)2 +z 2 = b2
com 0 < b < a, em torno do eixo z. Essa superfı́cie é dita toro.
a) Parametrize-a.
b) Calcule a sua área.
5. Determine a área do parabolóide z = 2(x2 + y 2 ), abaixo do plano z = 8.

6. Calcule a área da parte da superfı́cie esférica x2 + y 2 + z 2 = 2 que se


encontra dentro do parabolóide z = x2 + y 2 .
p
7. Calcule a área da parte da superfı́cie z = x2 + y 2 compreendida entre
os planos x + y = 1, x + y = 2, x = 0 e y = 0.

8. Seja S a superfı́cie de equação 2z = x2 + y 2 , onde 0 ≤ z ≤ k, k > 0.


a) Dê uma parametrização para S.
14π
b) Sabendo-se que a área de S vale , determine o valor de k.
3

118
Cálculo IV – Aula 11

9. Calcule a área da superfı́ce S parte da superfı́cie x2 + y 2 = 4, limitada por


z = 4 − x2 e z = 0.

10. Calcule a área da superfı́cie do cilindro x2 + y 2 = 2y limitada pelo plano


p
z = 0 e o cone z = x2 + y 2 .

119
Superfı́cies Parametrizadas

120
Cálculo IV – Aula 12

Aula 12
Integral de Superfı́cie de um Campo Es-
calar
Objetivos
• Estudar as integrais de superfı́cie de um campo escalar;
• estudar aplicações como cálculo de massa, centro de massa e momento de
inércia.

Integral de Superfı́cie de um Campo Escalar

Definimos a integral de superfı́cie de um campo escalar contı́nuo f (x, y, z)


sobre uma superfı́cie S, parametrizada por ϕ(u, v), com (u, v) ∈ D, da seguinte
maneira:
ZZ ZZ ZZ

∂ϕ ∂ϕ

f dS = f (x, y, z) dS = f ϕ(u, v) × dudv
S S D | ∂u ∂v
{z }
dS

∂ϕ ∂ϕ
onde dS = × dudv é o elemento de área.
∂u ∂v

OBS.:

a) Temos que, se S é o gráfico da função z = z(x, y),


com (x, y) ∈ D então:
ZZ ZZ
f dS = f (x, y, z) dS
S S
ZZ q
= f (x, y, z(x, y)) 1 + (zx )2 + (zy )2 dxdy
D | {z }
dS
q
onde dS = 1 + (zx )2 + (zy )2 dxdy é o elemento de área.
ZZ n ZZ
X
b) Se S = S1 ∪ S2 ∪ · · · ∪ Sn então f dS = f dS.
S i=1 Si
ZZ ZZ
c) Se f (x, y, z) = 1 em S então 1 dS = dS = A(S).
S S

121
Integral de Superfı́cie de um Campo Escalar

Exemplo 1
ZZ
Calcule xy dS, onde S é parametrizada por ϕ(u, v) = (u−v, u+v, 2u+v+1)
S
com (u, v) ∈ D : 0 ≤ u ≤ 1 e 0 ≤ v ≤ u.
Solução:
∂ϕ ∂ϕ
Temos = (1, 1, 2) e = (−1, 1, 1) donde
∂u ∂v


→ −→ −

i j k
∂ϕ ∂ϕ
× = 1 1 2 = (−1, −3, 2)
∂u ∂v
−1 1 1

e
∂ϕ ∂ϕ √

× = 1 + 9 + 4 = 14 .
∂u ∂v

Logo, dS = 14 dudv.
ZZ ZZ √
xy dS = (u − v)(u + v) 14 dudv
S D
√ ZZ

= 14 u2 − v 2 dudv
D

√ Z 1Z u 2
= 14 (u − v 2 ) dvdu
0 0

√ Z 1 h
v3
iu
= 14 u2 v − du
0 3 0

√ Z 1 h
u3
i
= 14 u3 − du
0 3
√ Z 1
2 14
= u3 du
3 0
√ h i1
14 4
= u
6 0

14
= .
6

Exemplo 2
ZZ
1
Calcule p dS, onde S é a parte da superfı́cie cilı́ndrica
S 1 + 4x2 + 4y 2
z = 1 − x2 − y 2 que se encontra dentro do cilindro x2 + y 2 ≤ 2y.

122
Cálculo IV – Aula 12

Solução:
O esboço de S é dado na figura que se segue:

1
2 y

−3

Temos S : z = 1 − x2 − y 2 , com (x, y) ∈ D : x2 + y 2 ≤ 2y. Como


| {z }
f (x,y)
q p
dS = 1 + (fx )2 + (fy )2 dxdy então dS = 1 + 4x2 + 4y 2 dxdy. Logo
ZZ ZZ p
1 1
p
2 2
dS = p
2 2
· 1 + 4x2 + 4y 2 dxdy
S 1 + 4x + 4y D 1 + 4x + 4y
ZZ
= dxdy
D

= ar (D)

= π · 12

= π.

Aplicações à Fı́sica

Seja S uma chapa delgada formando uma superfı́cie no espaço e seja δ(x, y, z)
sua densidade superficial que supomos contı́nua. Então, a massa M de S é dada
por ZZ
M= δ(x, y, z) dS .
S

123
Integral de Superfı́cie de um Campo Escalar

O momento de inércia de S em relação a um eixo E é dado por


ZZ
IE = r2 (x, y, z)δ(x, y, z) dS
S

onde r(x, y, z) = distância de (x, y, z) ao eixo E.


p
Se o eixo E = eixo z então r(x, y, z) = x2 + y 2 e
ZZ

Iz = x2 + y 2 δ(x, y, z) dS .
S

Se o eixo E = eixo y então r(x, y, z) = x2 + z 2 , donde
ZZ

Iy = x2 + z 2 f (x, y, z) dS .
S
p
Se o eixo E = eixo x então r(x, y, z) = y 2 + z 2 , donde
ZZ

Ix = y 2 + z 2 f (x, y, z) dS .
S

O centro de massa (x, y, z) é dado por


ZZ
xδ(x, y, z) dS
S
x=
ZZ M
yδ(x, y, z) dS
S
y=
ZZ M
zδ(x, y, z) dS
S
z= .
M

Exemplo 1
Calcule a massa da chapa fina S dada por ϕ(u, v) = (u, v, 2u + v), com
(u, v) ∈ D : 0 ≤ u ≤ 1 e 0 ≤ v ≤ u, sendo δ(x, y, z) = x + y + z a den-
sidade superficial.
Solução:
Temos
ZZ ZZ
∂ϕ ∂ϕ
M= (x + y + z) dS = (3u + 2v) × dudv ,
S D ∂u ∂v

onde

→ −
→ −

i j k
∂ϕ ∂ϕ
× = 1 0 2 = (−2, −1, 1)
∂u ∂v
0 1 1

124
Cálculo IV – Aula 12

e
∂ϕ ∂ϕ p √
× = (−2)2 + (−1)2 + 12 = 6 .
∂u ∂v
Logo,
√ ZZ √ Z 1Z u
M= 6 (3u + 2v) dudv = 6 (3u + 2v) dvdu
D 0 0

√ Z 1h iu
= 6 3uv + v 2 du
0 0

√ Z 1h 2 2
i
= 6 3u + u du
0

√ Z 1 2
= 6 4u du
0

4 6
= u.m.
3

Exemplo 2
Calcule o momento de inércia da superfı́cie homogênea de equação z = x2 + y 2 ,
com (x, y) ∈ D : x2 + y 2 ≤ 1, em torno do eixo z.
Solução:
O esboço de S está representado na figura que se segue.

1 y
D
1

A superfı́cie S é descrita por

S : z = f (x, y) = x2 + y 2 , (x, y) ∈ D : x2 + y 2 ≤ 1 .

125
Integral de Superfı́cie de um Campo Escalar

Temos
q p
dS = 1 + (fx )2 + (fy )2 dxdy = 1 + 4x2 + 4y 2 dxdy .

Então: ZZ ZZ
2 2
 
Iz = x +y δ(x, y, z) dS = k x2 + y 2 dS .
S S
pois S é homogênea. Logo:
ZZ
p
Iz = k x2 + y 2 1 + 4x2 + 4y 2 dxdy .
D

Usando coordenadas polares, temos


ZZ √ Z 1
2
1/2
Iz = k 2
r 1 + 4r r drdθ = 2kπ r2 1 + 4r2 r dr
Drθ 0

Fazendo u = 1 + 4r2 , temos r2 = (u − 1)/4 e r dr = du/8. Para r = 0 temos


u = 1 e para r = 1 temos u = 5. Então
Z 5
u − 1 1/2 du
Iz = 2kπ u
1 4 8
Z 5
kπ 
= u3/2 − u1/2 du
16 1
h i5
kπ 2 5/2 2
= u − u3/2
16 5 3 1
 
kπ 2 5/2 2 4
= 5 − 53/2 + .
16 5 3 15

126
Cálculo IV – Aula 12

Exercı́cios
ZZ 
1. Calcule z − x2 + xy 2 − 1 dS, onde S é a superfı́cie
S


→ −
→ −

ϕ(u, v) = u i + v j + (u2 + 1) k ,

com 0 ≤ u ≤ 1 0 ≤ v ≤ 2.
ZZ
2. Calcule x dS, onde S é o triângulo com vértices (1, 0, 0), (0, 1, 0)
S
e (0, 0, 1).
ZZ 
3. Calcule x2 + y 2 dS, onde S é a esfera x2 + y 2 + z 2 = a2 .
S

4. Calcule a massa da superfı́cie S parte do plano z = 2−x dentro do cilindro


x2 + y 2 = 1, sendo a densidade dada por δ(x, y, z) = y 2 .
ZZ
5. Calcule f (x, y, z) dS, onde f (x, y, z) = z e S é a parte da superfı́cie
S
z 2 = x2 + y 2 que se encontra acima do parabolóide 4z = x2 + y 2 + 3.

6. Seja S uma superfı́cie fechada tal que S = S1 ∪ S2 , onde S1 e S2 são as


superfı́cies de revolução obtidas pela rotação em torno do eixo z das curvas
C1 : z = 1 − x, 0 ≤ x ≤ 1 e C2 : z = 0, 0 ≤ x ≤ 1, respectivamente.
p
Se ρ(x, y, z) = x2 + y 2 é a função que fornece a densidade (massa por
unidade de área) em cada ponto (x, y, z) ∈ S, calcule a massa de S.

7. Mostre que o momento de inércia em relação ao eixo z da casca do cone


p
z = x2 + y 2 de altura h que está no primeiro octante com densidade
2
constante é I = M2h , onde M é a massa total.

8. Calcule o momento de inércia da superfı́cie esférica de raio R, homogênea,


de massa M , em torno de qualquer diâmetro.

9. Calcule o momento de inércia da superfı́cie homogênea, de massa M , de


equação x2 + y 2 = R2 , (R > 0), com 0 ≤ z ≤ 1, em torno do eixo z.

10. Mostre que o momento de inércia em relação a um diâmetro que passa


pelo centro de uma casca cilı́ndrica de comprimento L e raio de base a
1 1
com densidade constante é I = M a2 + M L2 , onde M é a massa total.
2 12

127
Integral de Superfı́cie de um Campo Escalar

128
Cálculo IV – Aula 13

Aula 13
Integral de Superfı́cie de um Campo Ve-
torial
Objetivos

• Compreender a noção de superfı́cie orientável;


• estudar as integrais de superfı́cie de campos vetoriais.

Integral de Superfı́cie de um Campo Vetorial

Hoje vamos integrar campos vetoriais sobre superfı́cies. Quando estudamos as


integrais de linha de campos vetoriais, vimos que a definição dependia da ori-
entação da curva, isto é,
Z Z

→ − → −
→ −
F ·dr =− F · d→r .
C− C

Aqui em integral de superfı́cie de um campo vetorial ou fluxo de um campo


vetorial, a definição também depende do conceito de superfı́cie orientada, que
passaremos a definir.
Dizemos que S é uma superfı́cie orientável quando for possı́vel escolher sobre
S um campo de vetores unitários normais a S que varie continuamente sobre
S. Intuitivamente falando, significa que S tem dois lados. Há superfı́cies que
tem um lado só como, por exemplo, a fita de Möbius que pode ser facilmente
construı́da. Peguem uma tira de papel retangular ABCD. Pintem um lado de
vermelho e o outro de azul. Fixem o lado AB e façam uma meia volta com o
lado CD e colem A com C e B com D.

B C B=D

A D A=C

A fita de Möbius tem apenas um lado, pois as duas cores se encontram.

129
Integral de Superfı́cie de um Campo Vetorial

OBS.: Superfı́cies fechadas orientáveis terão duas orientações


“naturais”, determinadas pela normal “exterior” e pela normal “in-
terior”.



11111111111111111111n
00000000000000000000 11111111111111111111
00000000000000000000
00000000000000000000
11111111111111111111
00000000000000000000
11111111111111111111 00000000000000000000
11111111111111111111
00000000000000000000
11111111111111111111
S
00000000000000000000
11111111111111111111 S
00000000000000000000
11111111111111111111
00000000000000000000
11111111111111111111 00000000000000000000
11111111111111111111
00000000000000000000
11111111111111111111
00000000000000000000
11111111111111111111 ou 00000000000000000000
11111111111111111111


00000000000000000000
11111111111111111111
00000000000000000000
11111111111111111111 n
00000000000000000000
11111111111111111111
00000000000000000000
11111111111111111111 00000000000000000000
11111111111111111111
00000000000000000000
11111111111111111111
00000000000000000000
11111111111111111111 00000000000000000000
11111111111111111111
00000000000000000000
11111111111111111111
00000000000000000000
11111111111111111111
00000000000000000000
11111111111111111111 00000000000000000000
11111111111111111111
00000000000000000000
11111111111111111111

Daqui para frente só consideraremos superfı́cies orientáveis (com dois lados).

Definição:
Seja S uma superfı́cie regular orientável. Seja −

n uma orientação de S.


Seja F um campo vetorial contı́nuo definido em um aberto contendo S.

→ −

A integral de superfı́cie de F através de S ou o fluxo φ de F através

→ →
de S é a integral de superfı́cie do campo escalar F · −
n:
ZZ

→ −
Φ= F ·→n dS .
S

OBS.:



1) Se F representa o campo de velocidades de um fluido, essa integral
fornece o volume do fluido que atravessa S em uma unidade de tempo,
na direção de −

n.



n

130
Cálculo IV – Aula 13

2) Se S é parametrizada por ϕ(u, v), (u, v) ∈ D então

ZZ ZZ

→ − −
→ ϕ × ϕv
Φ= F ·→
n dS = F (ϕ(u, v)) · u · kϕu × ϕv k dudv
S D kϕu × ϕv k
ZZ


= F (ϕ(u, v)) · (ϕu × ϕv ) dudv
D

ϕ × ϕv
se −

n = u e
kϕu × ϕv k

ZZ ZZ

→ − → −

Φ= F · n dS = − F (ϕ(u, v)) · (ϕu × ϕv ) dudv
S D

ϕ × ϕv
se −

n =− u .
kϕu × ϕv k
3) Se S é o gráfico da função z = f (x, y), (x, y) ∈ D, então

ZZ ZZ

→ − −

Φ= F ·→
n dS = F (x, y, f (x, y)) · (−fx , −fy , 1) dxdy
S D

(−fx , −fy , 1)
se −

n =p e
1 + (fx ) + (fy )2
2

ZZ ZZ

→ − −

Φ= F ·→
n dS = F (x, y, f (x, y)) · (fx , fy , −1) dxdy
S D

(f , f , −1)
se −

n = p x y2 .
1 + (fx ) + (fy )2
ZZ

→ −
4) Queremos definir F ·→n dS, onde S = S1 ∪ S2 ∪ · · · ∪ Sm .
S

S2

S1

S = S1 ∪ S2

131
Integral de Superfı́cie de um Campo Vetorial

Definição 1:
Seja S uma superfı́cie orientada por um campo de vetores normais
unitários −

n . Dizemos que o bordo de S, ∂S, está orientado positi-
vamente se ao caminhar ao longo de ∂S com a cabeça no sentido de −

n,
tivermos S à nossa esquerda.


n

∂S ∂S
S S
ou


n

OBS.: Uma regra prática para orientar ∂S é a conhecida


“regra da mão direita” com polegar no sentido de −

n.

Definição 2:
Dizemos que S = S1 ∪ S2 ∪ · · · ∪ Sm está orientada se for possı́vel orientar
cada Si de forma que nos bordos comuns a duas superfı́cies, as orientações
resultem opostas.


n 2



n1
S2 S2


n2
S1 ou −
→ S1
n1

Então:
ZZ ZZ ZZ

→ − −
→ − −
→ −
F ·→
n dS = F ·→
n1 dS + · · · + F · n→
m dS .
S S1 Sm

132
Cálculo IV – Aula 13

Exemplo 1

→ −
→ −
→ −

Calcule o fluxo de F (x, y, z) = 2x i + (x + y) j − 2xy k através da superfı́cie

S : ϕ(u, v) = u, v, 1 − u2 − v 2 , com (u, v) ∈ D : 0 ≤ u ≤ 1, 0 ≤ v ≤ 1 e com
ϕ × ϕv
normal −→
n = u .
||ϕu × ϕv ||
Solução:
Temos ϕu = (1, 0, −2u) e ϕv = (0, 1, −2v) donde


→ −
→ −→

i j k
ϕu × ϕv = 1 0 −2u = (2u, 2v, 1) .

0 1 −2v



O fluxo de F é dado por
ZZ ZZ

→ − −
→ ϕ × ϕv
F ·→
n dS = F (ϕ(u, v)) · u · kϕu × ϕv k dudv
S D kϕu × ϕv k
ZZ


= F (ϕ(u, v)) · (ϕu × ϕv ) dudv
D
ZZ
= (2u, u + v, −2uv) · (2u, 2v, 1) dudv
S
ZZ

= 4u2 + 2uv + 2v 2 − 2uv dudv
S
ZZ

= 4u2 + 2v 2 dudv
S
Z 1Z 1 
= 4u2 + 2v 2 dudv
0 0
Z 1h i1
4u3
= + 2uv dv
0 3 0
Z 1  
4
= + 2v dv
0 3
h i1
4
= v + v2
3 0

4
= +1
3
7
= .
3

133
Integral de Superfı́cie de um Campo Vetorial

Exemplo 2

→ −
→ −
→ −

Calcule o fluxo do campo F (x, y, z) = x i + y j + z k através da parte da
superfı́cie esférica x2 + y 2 + z 2 = a2 , com a normal exterior.
Solução:
Lembremos que, no caso da esfera x2 + y 2 + z 2 = a2 (ver Aula 11), temos

→ (x, y, z)
n = . Então o fluxo é dado por
a
ZZ ZZ ZZ ZZ

→ − (x, y, z) x2 + y 2 + z 2 a2
F ·→
n dS = (x, y, z) · dS = dS = dS
S S a S a S a
ZZ
= a dS
S

= aA(S)

= a4πa2

= 4πa3 .

Exemplo 3
ZZ

→ − −
→ −
→ −
→ −

Calcule F ·→
n dS, sendo F (x, y, z) = x i + y j + z k e S a parte do
S
cilindro x2 + y 2 = 4 entre z = 0 e y + z = 3, com a orientação normal que
aponta para o eixo z.
Solução:
O esboço de S está representado na figura que se segue.



n
S

3 y
x
134
Cálculo IV – Aula 13

Lembremos que, no caso do cilindro x2 + y 2 = a2 (ver Aula 11), o vetor unitário


(−x, −y, 0)
normal interior a S é −

n = . Então:
2
ZZ ZZ ZZ 2 2

→ − (−x, −y, 0) (x + y )
F ·→
n dS = (x, y, z) · dS = − dS
S S 2 S 2
ZZ
4
= − dS
S2
ZZ
= −2 dS .
S
ZZ
Para calcular dS, devemos parametrizar S. Logo S : ϕ(t, z) = (2 cos t, 2 sen t, z),
S
com (t, z) ∈ D : 0 ≤ t ≤ 2π e 0 ≤ z ≤ 3 − 2 sen t.
Vimos na Aula 11 que kϕt × ϕz k = k(2 cos t, 2 sen t, 0)k = 2. Como temos que
dS = kϕt × ϕz k dtdz então dS = 2 dtdz. Logo:
ZZ ZZ Z 2πZ 3−2 sen t

→ − →
F · n dS = −2 2 dtdz = −4 dzdt
S D 0 0
Z 2π
= −4 (3 − 2 sen t) dt = −24π .
0

Exemplo 4
ZZ

→ − −
→ −
→ −
→ −

Calcule F ·→
n dS, sendo F (x, y, z) = x i + y j + z k e S a parte do plano
S
y + z = 3, limitada pelo cilindro x2 + y 2 = 4, orientada com a normal −

n tal que

→ −

n · k ≥ 0.
Solução:
O esboço de S é dado a seguir.

S −

n
3

D
2 3
y
x
135
Integral de Superfı́cie de um Campo Vetorial



Se −→n · k ≥ 0 então a componente z de − →n é maior ou igual a zero donde

→n aponta para cima. A superfı́cie S pode ser descrita da seguinte maneira
S : z = 3 − y = f (x, y), com (x, y) ∈ D : x2 + y 2 ≤ 4. Um vetor normal a S


é dado por N = (−fx , −fy , 1) = (0, 1, 1) que aponta para cima. Logo, temos
(0, 1, 1)
que −→n = √ .
2

→ √
Por outro lado, sabemos da Aula 19 que dS = k N k dxdy = 2 dxdy. Portanto:
ZZ ZZ

→ − → (0, 1, 1) √
F · n dS = (x, y, 3 − y) · √ 2 dxdy
S D 2
ZZ
= (y + 3 − y) dxdy
D
ZZ
= 3 dxdy
D

= 3 A(D)

= 3π22

= 12π .

136
Cálculo IV – Aula 13

Exercı́cios

→ −
→ −
→ −

1. Calcular o fluxo do campo rotacional de F = (x − y − 4) i + y j + z k
através da semi-esfera superior de x2 + y 2 + z 2 = 1, com campo de vetores


normais − →
n tal que −→
n · k > 0.
ZZ

→ − −
→ −
→ −
→ −

2. Calcule F ·→
n dS onde F (x, y, z) = xzey i − xzey j + z k e S é
S
a parte do plano x + y + z = 1 no primeiro octante com orientação para
baixo.
ZZ

→ − −
→ −

3. Calcule F ·→
n dS, onde F = −z k e S é a parte da esfera de equação
S
x2 + y 2 + z 2 = 4 fora do cilindro x2 + y 2 = 1, −

n apontando para fora.

→ −
→ −
→ −

4. Calcule o fluxo do campo F = −x i − y j + 3y 2 z k sobre o cilindro
x2 + y 2 = 16, situado no primeiro octante entre z = 0 e z = 5 − y com a
orientação normal que aponta para o eixo z.
ZZ

→ − −
→ −
→ −

5. Calcule F ·→
n dS, sendo F (x, y, z) = z i + yz j e S é a parte do
S
plano z = 2−x, limitada pelo cilindro x2 +y 2 = 9, orientada com o campo

→ −

n tal que −
→n · k ≥ 0.

→ −
→ −
→ −

6. Ache o fluxo de F (x, y, z) = x i + y j + z k através da superfı́cie S do
sólido limitado por z = 0, x2 + y 2 = 4 e x + y + z = 3, com vetor − →n
exterior.

→ 
7. Ache o fluxo de F = yz, −xz, x2 + y 2 através de S superfı́cie de re-
volução obtida girando-se o segmento de reta que liga (1, 0, 1) e (0, 0, 3)
em torno do eixo z, onde o vetor normal −→
n tem componente z não nega-
tiva.
→ −
→ −
→ −

8. Calcule o fluxo do campo F (x, y, z) = x i + y j + z k através de
S: superfı́cie cilı́ndrica x2 + y 2 = a2 , a > 0, limitada superiormente pela

superfı́cie z = x2 e inferiormente pelo plano z = 0, com n apontando para
fora de S.
ZZ

→ − −
→ −
→ −

9. Calcule F ·→n dS, onde F (x, y, z) = −x i + 2z k e S é a fronteira
S
com região limitada por z = 1 e z = x2 + y 2 , com −

n exterior a S.

→ −
→ −
→ −

10. Determine o fluxo de F = x i + y j + 2z k através de S porção do cone
p
z = x2 + y 2 entre os planos z = 1 e z = 2, orientada para cima por
normais unitários.

137
Integral de Superfı́cie de um Campo Vetorial

138
Cálculo IV – Aula 14

Aula 14
Teorema de Gauss
Objetivos
• Estudar um teorema famoso que permite calcular fluxos através de su-
perfı́cies fechadas: o teorema de Gauss.

O Teorema de Gauss

O curso de Cálculo IV, contém alguns teoremas fascinantes como o Teorema de


Green, o Teorema de Stokes e o Teorema de Gauss. Nesta aula apresentamos
o famoso Teorema de Gauss ou Teorema da Divergência. Na próxima aula
apresentaremos o também famoso Teorema de Stokes.
O Teorema de Gauss estabelece uma relação entre uma integral tripla numa
região sólida W de R3 com uma integral de superfı́cie na sua fronteira. Este
teorema é um instrumento poderoso para os modelos matemáticos que descrevem
alguns fenômenos fı́sicos como, fluxos de fluidos, fluxos de campos elétricos ou
magnéticos e fluxos de calor.
Finalmente, enunciaremos o Teorema da Divergência ou de Gauss:

Teorema de Gauss: Seja W ⊂ R3 um sólido, cuja fronteira ∂W = S




está orientada positivamente, com −

n exterior a W . Seja F um campo
vetorial de classe C 1 em um aberto U contendo W . Então:
ZZ ZZZ

→ − → −

F · n dS = div F dxdydz .
S=∂W W

Para enunciar os teoremas de Gauss e de Stokes utilizaremos conceitos definidos


na Aula 7 – Campos Vetoriais: divergente e rotacional. Lembrando aqui que:

→ −
→ ∂P ∂Q ∂R
div F = ∇ · F = + +
∂x ∂y ∂z
e


→ − → −→
i j k

→ −

∂ ∂ ∂
rot F = ∇ × F =
∂x ∂y ∂z

P Q R

∂Q −
→ 
∂R −
→ 
∂P −
→
∂R ∂P ∂Q
= − i + − j + − k.
∂y ∂z ∂z ∂x ∂x ∂y

139
Teorema de Gauss

Exemplo 1

→ −
→ −
→ −

Verifique o teorema de Gauss para F (x, y, z) = xz i +yz j +z 2 k , calculando
as duas integrais do enunciado, e S a esfera x2 + y 2 + z 2 = a2 e ~n a normal
unitária exterior a S.
Solução:
(x, y, z)
Temos que ~n = . Logo
a

ZZ ZZ

→  (x, y, z)
F · ~n dS = xz, yz, z 2 · dS
S a
ZSZ
1 
= x2 z + y 2 z + z 3 dS
a
Z ZS
1 
= z x2 + y 2 + z 2 dS
a
2
Z ZS
a
= z dS
a S
ZZ
= a z dS .
S

Parametrizando S, temos ϕ(φ, θ) = (a sen φ cos θ, a sen φ sen θ, a cos φ), com
D : 0 ≤ φ ≤ π e 0 ≤ θ ≤ 2π. Temos também que dS = a2 sen φ dφ dθ. Então

ZZ ZZ

→ 
F · ~n dS = a (a cos φ) a2 sen φ dφ dθ
S D
Z 2πZ π
= a4 cos φ sen φ dφ dθ
0 0
Z 2π
π
4 sen2 φ
= a dθ
0 2
0
Z 2π
= a4 0 dθ
0

= 0.

Por outro lado, temos


→ ∂P ∂Q ∂R
div F = + + = z + z + 2z = 4z
∂x ∂y ∂z
140
Cálculo IV – Aula 14

e, então:

ZZZ Z 2πZ 1Z π


div F dV = 4ρ3 cos φ sen φ dφ dρ dθ
W 0 0 0
Z 2πZ 1 h iπ
sen2 φ
= 4 ρ3 dρ dθ
0 0 2 0
Z 2πZ 1
= 4 ρ3 · 0 dρ dθ
0 0

= 0.

O teorema de Gauss está, portanto, verificado.

Exemplo 2
ZZ

→ −
→ 
Calcule F · ~n dS, onde F (x, y, z) = x + yez , y + zex , z 2 + xey , S é a
S
fronteira do sólido interior ao cilindro x2 + y 2 = 1, entre os planos z = 0
e z = x + 2 e ~n a normal exterior a S.
Solução:
O esboço de S está representado na figura que se segue.

y


n
2 1

S
D


n
x −1 1 x
y 1
−1
−1 −

n
−2

141
Teorema de Gauss

Seja W o sólido limitado por S. Pelo teorema de Gauss, temos:


ZZ ZZZ

→ −

F · ~n dS = div F dV
S W
ZZZ
= (1 + 1 + 2z) dV
W
ZZ Z x+2
= 2 (1 + z) dz dx dy
D 0
ZZ h
z 2 x+2
i
= 2 z+ dx dy
D 2 0
ZZ h i
(x + 2)2
= 2 x+2+ dx dy
D 2
ZZ

= 6x + 8 + x2 dx dy
D
ZZ ZZ
= 6 x dx dy + 8 A(D) + x2 dx dy .
| {z }
| D {z } 8π
D
= 0

Passando para coordenadas polares, temos:


ZZ Z 2πZ 1
2
x dx dy = r3 cos2 θ dr dθ
D 0 0
Z 2π
1
= cos2 θ dθ
4 0
h i2π
1 1 sen 2θ
= · θ+
4 2 2 0

π
= .
4

Logo,
ZZ

→ π 33π
F · ~n dS = 8π + = .
S 4 4

Exemplo 3
ZZ

→ −
→ −
→ −
→ −

Calcule F · ~n dS, sendo F (x, y, z) = x3 i + y 3 j + z 3 k , ~n a orientação
S
normal exterior a S : x2 + y 2 + z 2 = 1, com z ≥ 0.
Solução:
O esboço de S (aberta) está representado na figura que se segue.

142
Cálculo IV – Aula 14



n

1 y
1

Seja S = S ∪ S1 onde S1 : z = 0, x2 + y 2 ≤ 1, com ~n1 = −~k.



n

S1 y
1
1


n1
x

Seja W o sólido limitado pela superfı́cie fechada S. Como estamos nas condições
do teorema de Gauss, temos

ZZ ZZ ZZZ ZZZ

→ −
→ −
→ 
F ·~n dS+ F ·~n1 dS = div F dV = 3 x2 + y 2 + z 2 dV .
S S1 W W

Passando para coordenadas esféricas, temos que x2 + y 2 + z 2 = ρ2 , também,


dV = ρ2 sen ϕ dρ dφ dθ e


 0≤ρ≤1


Wρφθ : 0 ≤ φ ≤ π/2 .



0 ≤ θ ≤ 2π

143
Teorema de Gauss

Então,
ZZZ ZZZ
2 2 2

3 x + y + z n dV = 3 ρ2 · ρ2 sen φ dρ dφ dθ
W Wρφθ
Z π/2Z 1Z 2π
= 3 ρ4 sen φ dθ dρ dφ
0 0 0
Z π/2Z 1
= 6π ρ4 sen φ dρ dφ
0 0
Z π/2

= sen φ dφ
5 0
h iπ/2

= − cos φ
5 0


= .
5

ZZ


Cálculo de F · ~n1 dS
S1

Temos
ZZ ZZ ZZ

→ 3

3
F · ~n1 dS = x , y , 0 · (0, 0, −1) dS = 0 dS = 0 .
S1 S1 S1

Logo, ZZ

→ 6π
F · ~n dS = .
S 5

144
Cálculo IV – Aula 14

Exercı́cios


1. Verifique o teorema de Gauss para o campo vetorial F (x, y, z) = (x, y, z)
no sólido W limitado pelas superfı́cies z = x2 + y 2 e z = 4.

→ −
→ −

2. Considere o campo vetorial F (x, y, z) = (2x + ez ) i + (3y − zex ) j +


+ (z − 2) k , e seja S a calota esférica x2 + y 2 + z 2 = a2 , com z ≥ 0 e


raio a > 0. Sabendo que o fluxo de F na direção da normal exterior ~n é
igual a 2πa3 , calcule o raio da calota.
ZZ

→ −
→ 
3. Calcule rot F · ~n dS, sendo F (x, y, z) = x2 y + 2, x3 + y 4 , 2xz − 1 ,
S
e S a parte da esfera x2 + y 2 + z 2 − 4z = 0, com z ≤ 1, orientada com ~n
exterior.

4. Seja f : R3 → R de classe C 2 . Seja W um sólido e seja S a fronteira de


W , com normal exterior ~n. Prove que
ZZ ZZZ
∂f
dS = ∇2 f dx dy dz
S ∂~n W

∂f
onde = ∇f · ~n é a derivada direcional de f na direção e sentido do
∂~n
∂2f ∂2f ∂2f
vetor unitário ~n e ∇2 f = 2 + 2 + 2 é o laplaciano de f (estudado
∂x ∂y ∂z
na Aula 7).
∂f 1
5. Seja f : R3 → R de classe C 2 , tal que ∇2 f = x2 + y 2 e (x, y, 1) = ,
ZZ ∂z 3
∂f
para todo (x, y, z) ∈ R3 . Calcule dS, onde S é a lata cilı́ndrica
S ∂~
n
com fundo e sem tampa dada por x2 + y 2 = 1, 0 ≤ z ≤ 1, x2 + y 2 ≤ 1 e
z = 0, com normal ~n apontando para fora de S.

→ −
→ −→ −

6. Seja F (x, y, z) = z arctg y 2 i + z 3 ln x2 + 1 j + z k . Determine o


fluxo de F através da parte do parabolóide x2 + y 2 + z = 2 que está acima
do plano z = 1, sendo − →
n a normal com componente z não negativa.

7. Seja Q uma carga elétrica localizada na origem. Pela Lei de Coulomb, a




força elétrica F (x, y, z) exercida por essa carga sobre uma carga q locali-
zada no ponto (x, y, z) com vetor posição X é

→ εqQ
F (X) = X
kXk3
onde ε é uma constante. Considere a força por unidade de carga

→ 1−
→ εQ εQ(x, y, z)
E (X) = F (X) = 3X = 3/2
q kXk x2 + y2 + z2

145
Teorema de Gauss



que é chamada campo elétrico de Q. Mostre que o fluxo elétrico de E
é igual a 4πεQ, através de qualquer superfı́cie fechada S que contenha a
origem, com normal − →n apontando para fora se S. Esta é a Lei de Gauss
para uma carga simples.
ZZ

→−
8. Calcule F· →
n dS, onde
S


→ −
→ −
→ −

F (x, y, z) = x i + − 2y + ex cos z j + z + x2 k

e S é definida por z = 9 − x2 + y 2 , 0 ≤ z ≤ 5; z = 5, 1 ≤ x2 + y 2 ≤ 4

e z = 8 − 3 x2 + y 2 , x2 + y 2 ≤ 1.

9. Seja a superfı́cie cônica S de vértice (0, 0, h) e de base situada no plano




xy com raio 1 e − →n com a componente k não negativa. Seja

→ ∂f −
→ ∂f −
→ −

F (x, y, z) = (x, y, z) i − (x, y, z) j + 2(z + 1) k ,
∂y ∂x


sendo f (x, y, z) de classe C 2 . Calcule o fluxo de F através de S.

→ x3 y3 z3 
10. Calcule o fluxo do campo F (x, y, z) = +y, , +2 através da su-
3 3 3
perfı́cie S do sólido W definido por W = (x, y, z) ∈ R3 | x2 +y 2 +z 2 ≥ 1 ,
p
x2 + y 2 + (z − 2)2 ≤ 4 , z ≥ x2 + y 2 , com campo de vetores normais
a S apontando para fora de W .

146
Cálculo IV – Aula 15

Aula 15
Teorema de Stokes
Objetivos
• Estudar um teorema famoso que generaliza o teorema de Green para o
espaço.
Seja U um aberto conexo de R3 e

→ −

n
F = (P, Q, R) um campo vetorial de classe
C 1 em U . Seja S ⊂ U , uma superfı́cie regu-
lar por partes, orientada pelo campo normal ∂S
S
unitário −
→n . Seja ∂S o bordo de S, com a
orientação induzida pela de S. Então,
ZZ I

→ − → −
→ −
rot F · n dS = F · d→
r
S ∂S
| {z } | {z }


fluxo do rotacional circulação de F

OBS.: Seja S uma superfı́cie plana contida no plano xy,




orientada com −→
n = k . Então S : z = 0, (x, y) ∈ D.


Logo, dS = dxdy. Seja F (x, y) = (P (x, y), Q(x, y)),
então:

∂P −

→ →
∂Q
rot F = rot(P, Q, 0) = ∂x − ∂y k .



n
y
S
x C = ∂S

Logo, pelo Teorema de Stokes, temos:


I I

→ −
F · d→
r = P dx + Q dy
∂S ∂S
ZZ

→ →
= rot F · −
n dS
S
ZZ  
∂Q ∂P
= 0, 0, − · (0, 0, 1) dxdy
D ∂x ∂y
ZZ  
∂Q ∂P
= − .
D ∂x ∂y

147
Teorema de Stokes

Isto prova que o Teorema de Stokes generaliza o Teorema de Green.


Como consequência do Teorema de Stokes, temos:



“Se U = dom F é um conjunto simplesmente conexo, isto é, U é o R3

→ − →
ou U é o R3 , exceto um número finito de pontos, e se rot F = 0 então


F é conservativo”.


O teorema das quatro equivalências é dado por: Seja F = (P, Q, R) um campo
vetorial de classe C 1 em um conjunto simplesmente conexo do R3 . Então as
seguintes afirmações são equivalentes:
I

→ −
(1) F · d→r = 0 para toda curva fechada C;
C
Z

→ −
(2) F · d→r é independente do caminho;
C

→ −

(3) F é um campo gradiente, isto é, F = ∇ϕ para algum campo escalar ϕ;

→ − →
(4) rot F = 0 .

Exemplo 1
Verifique o teorema de Stokes, calculando as duas integrais do enunciado, para

→ −
→ −
→ −

F (x, y, z) = −y i + x j + z k

e S o parabolóide z = 1 − x2 − y 2 com z ≥ 0 e −

n a normal unitária exterior
a S.
Solução:
O esboço de S está representado na figura que se segue.

z
1



S n

1 y
1
∂S

x
148
Cálculo IV – Aula 15

Pela regra da mão direita, com o polegar no de sentido −


→n e movimentando os
dedos, vemos que a curva bordo de S, ∂S, fica orientada no sentido anti-horário,
quando vista de cima. Então uma parametrização de ∂S é, ∂S : x = cos t,
y = sen t e z = 0, com 0 ≤ t ≤ 2π, donde dx = − sen t dt, dy = cos t dt e
dz = 0. Então:
Z Z

→ − →
F ·dr = −y dx + x dy + z dz
C C
Z 2π  
= (− sen t)(− sen t) + (cos t)(cos t) dt
0
Z 2π 
= sen2 t + cos2 t dt
0

= 2π .

Por outro lado,




→ −
→ −→
i j k

− ∂ ∂ ∂

rot F = = (0, 0, 1 + 1) = (0, 0, 2) .
∂x ∂y ∂z

−y x z

Temos, S : z = 1 − x2 − y 2 = f (x, y), com (x, y) ∈ D : x2 + y 2 ≤ 1. Um




vetor normal a S é dado por N = (−fx , −fy , 1) = (2x, 2y, 1) que é exterior a
S. Logo,


→ (2x, 2y, 1) p
n =p e dS = 1 + 4x2 + 4y 2 dx dy .
21 + 4x + 4y 2

Então ZZ ZZ

→ →
rot F · −
n dS = (0, 0, 2) · (2x, 2y, 1) dx dy
S D
ZZ
= 2 dx dy
D

= 2 A(D)

= 2π · 12

= 2π .

Assim, o teorema de Stokes está verificado para este caso.

149
Teorema de Stokes

Exemplo 2
Z
→ −
− →
Use o teorema de Stokes para calcular F · dr, onde
C



F (x, y, z) = (y − x + sen x, z − x + cos y, x − y + ez )
x z
e C é a interseção do cilindro x2 + y 2 = a2 com o plano + = 1, sendo
a b
a > 0, b > 0, orientada no sentido anti-horário quando vista da parte superior do
eixo z.
Solução:
O esboço de C está representado na figura a seguir.

a a y
x

Para aplicar o teorema de Stokes, precisamos de uma superfı́cie S cujo bordo


x z
seja a curva C. Então consideremos a porção do plano + = 1, limitada
a b
por C.



n

C
S
b

a a y
x

150
Cálculo IV – Aula 15

Pela regra da mão direita, vemos que −→n aponta para cima. Logo, podemos
bx
descrever S por S : z = b − = f (x, y), com (x, y) ∈ D : x2 + y 2 ≤ a2 .
a  

→ b
Um vetor normal a S é N = (−fx , −fy , 1) = , 0, 1 que aponta para cima.
 a


b
a , 0, 1


Então, n = − → e dS = k N k dxdy. Temos:
kN k


→ −
→ −

i j k



∂ ∂ ∂
rot F =
∂x ∂y ∂z


y − x + sen x z − x + cos y x − y + ez

= (−1 − 1, 0 − 1, −1 − 1)
= (−2, −1, −2) .

Do teorema de Stokes, temos


Z ZZ

→ − → −
→ →
F ·dr = rot F · −
n dS
C S
ZZ  
b
= (−2, −1, −2) · , 0, 1 dx dy
D a
  ZZ
−2b
= −2 dx dy
a D
 
b+a
= −2 πab
a

= −2πb(b + a) .

Exemplo 3


Seja F (x, y, z) = (3x2 z + y 3 , 3xy 2 + ez , x3 + yez ).


a) F é conservativo? Por quê?
b) Seja C a curva obtida como interseção da superfı́cie z = x2 + y 2 − 4,
Z

→ →
z ≤ 1 com o plano y = −1. Calcule F · d− r , especificando a orientação
C
escolhida.

Solução:

→ −
→ −

a) Temos rot F = (ez − ez , 3y 2 − 3y 2 , 3y 2 − 3y 2 ) = 0 e dom F = R3 que
é um conjunto simplesmente conexo. Então, pelo teorema das equivalências em


R3 , segue que F é um campo conservativo.

151
Teorema de Stokes

b) Logo, existe uma função potencial ϕ(x, y, z), tal que:


∂ϕ
= 3x2 z + y 3 (1)
∂x
∂ϕ
= 3xy 2 + ez (2)
∂y
∂ϕ
= x3 + yez (3)
∂z
Integrando (1), (2) e (3) em relação a x, y e z, respectivamente, temos:

ϕ(x, y, z) = x3 z + xy 3 + f (y, z) (4)

ϕ(x, y, z) = xy 3 + yez + g(x, z) (5)

ϕ(x, y, z) = x3 z + yez + h(x, y) (6)

Comparando (4), (5) e (6), vemos que f (y, z) = yez , g(x, z) = x3 z e


h(x, y) = xy 3 . Logo, ϕ(x, y, z) = x3 z + xy 3 + yez , (x, y, z) ∈ R3 é uma


função potencial de F . O esboço de C está representado na figura a seguir.

A 1

−1
y
x

−4

Escolhamos a orientação de A = (2, −1, 1) para B = (−2, −1, 1). Então


Z

→ −
F · d→ r = ϕ(B) − ϕ(A)
C

= ϕ(−2, −1, 1) − ϕ(2, −1, 1)

= (−8 + 2 − e) − (8 − 2 − e)

= −12 .

152
Cálculo IV – Aula 15

Exercı́cios

1. Use o teorema de Stokes para mostrar que a integral de linha é igual ao


valor dado, indicando a orientação da curva C.
I √
3 2 πa2
(3y + z) dx + (x + 4y) dy + (2x + y) dz = −
C 4

onde C é a curva obtida como interseção da esfera x2 + y 2 + z 2 = a2 com


o plano y + z = a.

2. Use o Teorema de Stokes para mostrar que a integral de linha é igual ao


valor dado, indicando a orientação da curva.
I  
2 x2
2xy dx + [(1 − y)z + x + x] dy + + ez dz = π
C 2

onde C é a curva obtida como interseção do cilindro x2 + y 2 = 1, z ≥ 0,


com o cone z 2 = x2 + (y − 1)2 .



3. Seja F um campo vetorial cujo rotacional em cada ponto de R3 é dado
por


rot F (x, y, z) = (x, −2y, z) .
I

→ −
Calcule F · d→
r , onde C é o contorno do semifuso esférico, de ângulo
C
π/4, indicado na figura que se segue.

a C

π/4 a y
a

153
Teorema de Stokes



4. Seja F (x, y, z) = (2xz + y 2 , 2xy + 3y 2 , ez + x2 ).


a) F é um campo conservativo em R3 ? Por quê?
b) Seja C a curva obtida como interseção da superfı́cie Zde equação

→ −
z = 9 − x2 − y 2 , z ≥ −4 com o plano y = 2. Calcule F · d→r,
C
especificando a orientação escolhida.
ZZ

→ →
5. Utilizando o Teorema de Stokes, transforme a integral rot F · −
n dS
S

→ −

numa integral de linha e calcule, onde F (x, y, z) = y i , S é a superfı́cie
z = x2 + y 2 com z ≤ 1, e −
→n a normal com componente z positiva.

6. Use o Teorema de Stokes para calcular o fluxo do rotacional do campo



→ −
→ −
→ −

F (x, y, z) = x2 y i + 2y 3 z j + 3z k

→ −
→ −

através da superfı́cie S : ϕ(r, θ) = (r cos θ) i + (r sen θ) j + r k , com
0 ≤ r ≤ 1 e 0 ≤ θ ≤ 2π no sentido do vetor unitário normal exterior − →
n.
Z

→ −
7. Calcule F · d→
r , onde
C


→ 
F (x, y, z) = e−y − ze−x , e−z − xe−y , e−x − ye−z
   
ln(1 + t) πt 1 − et
e C é a curva parametrizada por γ(t) = , sen , ,
ln 2 2 1−e
com 0 ≤ t ≤ 1.
Z

→ −
8. Determine F · d→
r , onde
C


→ 
F (x, y, z) = yz + x3 , xz + 3y 2 , xy + 4

e C é a curva obtida como interseção das superfı́cies de equações


z = 5 − y 2 , z ≥ 1 e x + z = 5, orientada no sentido de crescimento
de y.
Z

→ → −

9. Calcule F · d−

r , onde F (x, y, z) = −2y + esen x , −z + y, x3 + esen z
C
e C é a interseção da superfı́cie z = y 2 com o plano x + z = 1, orientada
no sentido do crescimento de y.
Z    
10. Calcule (z − y)dx + ln 1 + y 2 dy + ln 1 + z 2 + y dz, sendo C dada
C
por γ(t) = (4 cos t, 4 sen t, 4 − 4 cos t), com 0 ≤ t ≤ 2π.

154
Cálculo IV – Aula 16

Aula 16
Preparação para a AP2
Exercı́cios
I

→ −
1. Calcule F · d→
r , ao longo da circunferência x2 + y 2 = 2y, no sentido
C
horário, sendo


→  
3x2 p
F (x, y) = 10x2 − 3y, + y4 + 1 .
2

Z
2. a) A integral I = (sen xy + xy cos xy) dx + x2 cos xy dy é indepen-
C
dente do caminho?

b) Calcule o valor I onde C é dada por σ(t) = t2 −1, t2 +1 , 0 ≤ t ≤ 1.

3. Uma lâmina tem a forma da parte lateral do cilindro x2 + y 2 = 4, entre os


planos z = 0 e z = 3 − x. Determine a massa dessa lâmina se a densidade
no ponto (x, y, z) é dada por f (x, y, z) = x2 .

4. Calcule
Z    2   2 
x2 2 y 2 z 2
I= e +y dx + e − z dy + e − x dz ,

onde C é o contorno da parte do plano x + y + z = 1, que está no primeiro


octante, no sentido anti-horário.

5. Calcule o fluxo de


→ −
→ h  i−
→ −

F = (ez arctg z) i + ez ln x2 + 1 j + z k

através de S parte do parabolóide z = 4 − x2 − y 2 , acima de z = 1, na


direção da normal exterior −

n.

6. Calcule I  
x2 y 2
I= − 2
+ y dx + (x + arctg x)dy ,
C 1+x

onde C é o caminho fechado formado por y = 0, x + 2y = 4 e x = 0,


orientado no sentido anti-horário.

155
Preparação para a AP2

7. Considere a superfı́cie S porção do cilindro x2 + y 2 = 2x, compreendida


entre o plano z = −1 e o parabolóide z = x2 + y 2 . Pede-se.
a) Uma parametrização de S.
b) A área de S.

→ 
c) O fluxo do campo F (x, y, z) = 3x−3−y , 3y+x−1 , esen z através
de S, com o campo de vetores normais apontando para dentro de S.

8. Seja

→ → x2 −
− → √ −

F (x, y, z) = 2y i + j + 1 + z8 k .
2
I

→ →
Calcule F · d−r , onde C é a curva dada pela interseção das superfı́cies
C
z = x2 +y 2 e x2 +(y −1)2 = 1 com um sentido de percuso tal que, quando
projetado no plano z = 0 produz um percurso no sentido anti-horário.

9. Seja um campo de forças dado por



→ 
F (x, y, z) = 3x2 yz + ez , x3 z, x3 y + xez + 3z 2 .



a) F é um campo conservativo? Por quê?


b) Calcule o trabalho realizado por F para mover uma partı́cula ao longo
da curva C interseção da superfı́cie z = 1−x2 com o plano y +z = 1,
orientada no sentido do crescimento de x.
ZZ

→ →
10. Use o teorema de Stokes para transformar a integral rot F · −
n dS em
D

→ 
uma integral de linha e calcule-a sendo F (x, y, z) = yez−1 , −xez−1 , 3ez
e S a superfı́cie x2 + y 2 + (z − 1)2 = 1 com 0 ≤ z ≤ 1 e − →
n a normal
apontando para baixo.

156
Solução dos Exercı́cios

Aula 1

1.
Solução:
Temos ZZ Z 1Z
  1  2
x y2
I= e ye dxdy = e yey dxdy
x
D 0 −1
Z 1  2
= yey [ex ]1−1 dy
0
Z 1
−1
 2
= e−e yey dy .
0
du
Fazendo u = y 2 temos du = 2y dy, donde ydy = . Para y = 0, temos u = 0
2
e para y = 1 temos u = 1. Logo,
Z 1  
e − e−1 u e − e−1  u 1 e − e−1
I= e du = e 0= (e − 1) .
2 0 2 2

2.
Solução:
ZZ
a) Observe que I = f (x, y) dxdy, onde
D

 √
D = (x, y) ∈ R2 | 0 ≤ x ≤ 1 , x3 ≤ y ≤ x .

Portanto, D é do tipo I e está ilustrada na figura a seguir.

y
y = x3

y= x
1

x = y2 D

3
x= x

1 x

157
Descrevendo D como tipo II, temos:
 √
D = (x, y) ∈ R2 | 0 ≤ y ≤ 1 , y 2 ≤ x ≤ 3 y .

Então √
Z 1Z 3 y
f (x, y) dxdy .
0 y2

ZZ
b) Temos que I = f (x, y) dxdy, onde
D

 p √
D = (x, y) ∈ R2 | 0 ≤ y ≤ 1 , − 1 − y 2 ≤ x ≤ 1 − y

é do tipo II.

x2 + y 2 = 1
1

y = 1 − x ⇒ y = (1 − x)2 , 0 ≤ x ≤ 1
D

−1 1 x

Para descrever D como tipo I, devemos dividi-la em duas regiões: D = D1 ∪ D2 .


y= 1 − x2
1

D1

y = (1 − x)2
D2

−1 1 x
y=0 y=0

onde
 √
D1 = (x, y) ∈ R2 | −1 ≤ x ≤ 0 , 0 ≤ y ≤ 1 − x2

D2 = (x, y) ∈ R2 | 0 ≤ x ≤ 1 , 0 ≤ y ≤ 1 − x2 .

158
Então
ZZ ZZ
I = f (x, y) dxdy + f (x, y) dxdy
D1 D2

Z 0 Z 1−x2 Z 1Z (1−x)2
= f (x, y) dydx + f (x, y) dydx .
−1 0 0 0

3.
Solução:
ZZ
2
Temos que I = ex dxdy, onde
D


D = (x, y) ∈ R2 | 0 ≤ y ≤ 1 , y ≤ x ≤ 1

é do tipo II.

y=x
1

D x=1

1 x

Descrição de D como tipo I

Projetando D sobre o eixo x, temos o intervalo [0, 1]. Fixando x ∈ [0, 1], a
ordenada y varia de y = 0 a y = x. Então,
Z 1Z x Z 1 Z 1
x2 x2
 x 2
I= e dydx = e y 0 dx = xex dx .
0 0 0 0

du
Fazendo u = x2 , temos du = 2x dx, donde xdx = . Temos:
2

x=0⇒u=0
x=1⇒u=1

Então Z 1
u du 1 1 e−1
I= e = eu = .
0 2 2 0 2

159
4.
Solução:
O esboço de D está representado na figura que se segue.

y
(−4, 2) 2
2 (6, 2)
x = −y
x=4+y
x = −y 2
D

(−1, −1) (5, −1)


−1

x−y =4

−4

Da figura temos que


D = (x, y) | −1 ≤ y ≤ 2 , −y 2 ≤ x ≤ 4 + y .
ZZ
Como A(D) = dxdy, então
D

Z 2 Z 4+y Z 2 
A(D) = dxdy = 4 + y + y 2 dy
−1 −y 2 −1
h i2
y2 y3
= 4y + +
2 3 −1
   
8 1 1
= 8+2+ − −4 + −
3 2 3
38 23
= +
3 6
33
= u.a.
2

160
5.
Solução:
O esboço de D está representado na figura que se segue.

y y=

x √
y= x
2

4 x
y=0

Enquadrando D como tipo I, temos

(
0≤x≤4
D: √ .
0≤y≤ x

Então:
ZZ Z 4Z √
x
y y
dxdy = dydx
D 1 + x2 0 0 1 + x2
Z 4 h i √x
1 y2
= dx
0 1 + x2 2 0
Z 4
1 x
= dx .
2 0 1 + x2

Temos d(1 + x2 ) = 2x dx, donde x dx = 12 d(1 + x2 ). Logo,

ZZ Z 4
y 1 1 d(1 + x2 )
2
dxdy = ·
D1+x 2 2 0 1 + x2

1 4
= ln(1 + x2 )
4 0

1
= (ln 17 − ln 1)
4
ln 17
= .
4

161
6.
Solução:
ZZ
3
Temos I = ex dxdy, onde D = D1 ∪ D2 com
D
n √
y √ o
D1 = (x, y) ∈ R2 | 0 ≤ y ≤ 1 , ≤x≤ y
2
n √ o
y
D2 = (x, y) ∈ R2 | 1 ≤ y ≤ 4 , ≤x≤1 .
2
Com as informações dadas acima, podemos ilustrar a nossa região D.
y


y x=1
x= ⇒ y = 4x2
2
D2
1 √
D1 x= y ⇒ y = x2

1 x

Enquadrando D como tipo I, temos



D = (x, y) ∈ R2 | 0 ≤ x ≤ 1 , x2 ≤ y ≤ 4x2 .

Então Z 1Z 4x2
3
I = ex dydx
0 x2
Z 1
3 
= ex 4x2 − x2 dx
0
Z 1
3 
= ex 3x2 dx
0

3 1
= ex
0

= e − 1.

7.
Solução:
De y = x2 e y = 4x − x2 , temos 2x2 − 4x = 0, donde x = 0, x = 2. Logo as
interseções são (0, 0) e (2, 4). O esboço da região D está representado na figura
que se segue.

162
y
y = 4x − x2

y = x2
D

2 x

Escrevendo D como tipo I, temos:



 0≤x≤2
D:
 x2 ≤ y ≤ 4x − x2 .
ZZ
Como A(D) = dxdy então:
D

Z 2Z 4x−x2 Z 2 
Z 2 
2 2
A(D) = dydx = 4x − x − x dx = 4x − 2x2 dx
0 x2 0 0
h i2
2x32
= 2x −
3 0

16
= 8−
3
8
= u.a.
3

8.
Solução:
y
2

x+y =2

1
x = y3
x = y3 x=2−y
y
D

1 2 x

163
Vamos enquadrar D como tipo II:


D = (x, y) ∈ R2 | 0 ≤ y ≤ 1 , y 3 ≤ x ≤ 2 − y .

ZZ
Como A(D) = dxdy, então:
D

Z 1Z 2−y Z 1
y4 1
h i
y2 5
A(D) = dxdy = (2 − y − y 3 ) dy = 2y − − = u.a.
0 y3 0 2 4 0 4

9.
Solução:
Para esboçar a superfı́cie z = 4−x2 , no primeiro octante, desenhamos a parábola
z = 4 − x2 no plano xz, com x ≥ 0 e z ≥ 0. Como a equação não contém a
variável y, então devemos traçar paralelas ao eixo y por pontos da parábola. Para
esboçar o plano x + y = 2, no primeiro octante, desenhamos a reta x + y = 2
no plano xy, com x ≥ 0 e y ≥ 0. Como a equação não contém a variável z,
então traçamos paralelas ao eixo z por pontos da reta. Observamos que a reta
x + y = 2 do plano intercepta a superfı́cie z = 4 − x2 no ponto A. Além disso a
reta do plano, paralela ao eixo z, passando por (0, 2, 0), intercepta a superfı́cie
z = 4 − x2 no ponto B. Ligando A a B, encontramos a curva interseção das
duas superfı́cies. Assim, o esboço de W é:

z
4
y
B

W 2
x+y =2

“teto”
y =2−x
D

2 x
A
2 2
y y=0
x D (“piso”)

164
Observe que a equação do “teto” é z = 4 − x2 = f (x, y) e o “piso” D é o
triângulo acima. Então:
ZZ ZZ
V (W ) = f (x, y) dxdy = (4 − x2 ) dxdy
D D
Z 2Z 2−x 
= 4 − x2 dydx
0 0
Z 2
= (2 − x)(4 − x2 ) dx
0
Z 2 
= 8 − 2x2 − 4x + x3 dx
0
h i2
x3 x4
= 8x − 2 − 2x2 +
3 4 0

16
= 16 − −8+4
3
20
= u.v.
3

10.
Solução:
A integral dupla de uma função positiva f (x, y) pode ser interpretada como o
volume do sólido abaixo do gráfico de f (x, y) e acima do domı́nio D de f (x, y),
contido no plano xy. Devemos esboçar o sólido para encontrar a região de
integração.

z = f (x, y) = 9 − y 2

−3

3
9
x 9
D y

165
Do esboço de W , vemos que a região de integração tem a seguinte forma:

y
9

3
D 9
x
x=9−y
x=0
−3

Então
ZZ ZZ

V (W ) = f (x, y) dxdy = 9 − y 2 dxdy .
D D

Descrevendo D como tipo II, temos:

D = {(x, y) | −3 ≤ y ≤ 3 , 0 ≤ x ≤ 9 − y} .

Então:
Z 3 Z 9−y 
V (W ) = 9 − y 2 dxdy
−3 0
Z 3 
= 9 − y 2 (9 − y) dy
−3
Z 3 
= 81 − 9y − 9y 2 + y 3 dy
−3
h i3
9y 2 y4
= 81y − − 3y 3 +
2 4 −3
 
= 81(3 + 3) − 0 − 3 33 − (−3)3 + 0

= 81 · 6 − 3 · 2 · 27

= 6 · 27(3 − 1)

= 324 u.v.

166
Aula 2

1.
Solução:
Temos que:

B′ 2

Du
2

O A′ u

Imagem de O′ A′ pela transformação T

O′ A′ : v = 0, 0 ≤ u ≤ 2 ⇒ OA : x = u, y = u2 , 0 ≤ u ≤ 2

⇒ OA : y = x2 , 0 ≤ x ≤ 2 .

Imagem de A′ B ′ pela transformação T

A′ B ′ : u + v = 2, 0 ≤ u ≤ 2 ⇒ AB : x = 2, y = u2 − (2 − u) , 0 ≤ u ≤ 2

⇒ AB : x = 2, −2 ≤ y ≤ 4 .

Imagem de O′ B ′ pela transformação T

O′ B ′ : u = 0, 0 ≤ v ≤ 2 ⇒ OB : x = v, y = −v, 0 ≤ v ≤ 2

⇒ OB : y = −x, 0 ≤ x ≤ 2 .

167
y

y = x2

Dxy

2 x

−2
y = −x

b) Temos


∂(x, y) 1 1
= = −1 − 2u .
∂(u, v) 2u −1

Usando a fórmula
ZZ ZZ
 ∂(x, y)
f (x, y) dxdy = f x(u, v), y(u, v) dudv ,
Dxy Duv ∂(u, v)

temos
ZZ ZZ
dxdy 1
√ = p |−1 − 2u| dudv
Dxy 1 + 4x + 4y Duv 1 + 4(u + v) + 4(u2 − v)
ZZ
1
= √ (1 + 2u) dudv
Duv 1 + 4u + 4u2
ZZ
1
= p (1 + 2u) dudv
Duv (1 + 2u)2
ZZ
= dudv
Duv

= A(Duv )

1
= ×2×2
2

= 2.

168
2.
Solução:
x−y
Considerando a mudança de variáveis u = , v = x + y temos:
x+y

(
uv = x − y
v =x+y

donde

uv + v
 x=

2
 y = v − uv

2

Assim,

∂x ∂x v 1
∂(x, y) ∂u
∂v

2

2 v v v
J= = =
= + = .
∂(u, v) ∂y ∂y
−v 1 4 4 2

∂u ∂v 2 2

v
Como dxdy = |J| dudv, então dxdy = dudv .
2
Passando para as coordenadas u e v as equações das curvas que limitam D,
temos:

x+y =1 ⇒ v =1
x+y =4 ⇒ v =4
x = 0 ⇒ u = −1
y=0 ⇒ u=1

Portanto Duv pode ser descrito por:


 −1 ≤ u ≤ 1
Duv :
 1≤v≤4

169
Do Teorema da Mudança de Variáveis, temos:
ZZ   ZZ  
π x−y π v
cos · dxdy = cos u dudv
D 2 x+y Duv 2 2
Z 1  Z 4
1 π
= cos u v dvdu
2 −1 2 1
Z 1  h i4
1 π v2
= cos u du
2 −1 2 2 1
Z 1  
15 π
= cos u du
4 −1 2
h  i1
15 2 π
= · sen u
4 π 2 −1
 i
h
15 π π
= sen − sen −
2π 2 2
15
= (1 − (−1))

15
= .
π

3.
Solução:
Façamos x = au e y = bv. Logo,

∂x ∂x
∂(x, y) ∂u
∂v a 0
J= = = = ab .
∂(u, v) ∂y ∂y

0 b
∂u ∂v

x2 y2
O conjunto D : + ≤ 1 é transformado em Duv : u2 + v 2 ≤ 1. Como
ZZ a2 b2
A(D) = dxdy então:
D

ZZ ZZ ZZ
A(D) = |J| dudv = ab dudv = ab dudv
Duv Duv Duv

= abA(Duv )

= ab · π · 12

= πab u.a.

170
4.
Solução:
u
Façamos a mudança de variáveis u = 2x, v = y ou x = , y = v. Logo:
2

1
∂(x, y) 0
2 1
J= = = ,
∂(u, v) 2
0 1
donde,
1
dxdy = |J| dudv = dudv .
2
A região D é limitada pela semi-elipse 4x2 + y 2 = 1, com y ≥ 0 e a reta y = 0.
Passando essas equações para as novas coordenadas temos u2 + v 2 = 1, com
v ≥ 0 e v = 0. Logo, Duv é limitada pela semicircunferência u2 + v 2 = 1, com
v ≥ 0 e o eixo u.
v
1

r=1
Duv

−1 1 u
r=0

Então pelo Teorema da Mudança de Variáveis, temos:


ZZ ZZ
2 2
 1
I= sen 4x + y dxdy = sen u2 + v 2 dudv
D Duv 2
ZZ
1 
= sen u2 + v 2 dudv .
2 Duv

Passando para coordenadas polares, temos





 u = r cos θ

v = r sen θ


 dudv = rdrdθ
 2
u + v 2 = r2
A região Duv descrita em coordenadas polares é:
(
0≤θ≤π
Drθ :
0≤r≤1

171
Logo,
ZZ Z 1 Z π
1 2
 1 
I= sen r r drdθ = sen r2 r dθdr
2 Drθ 2 0 0
Z 1
π 
= sen r2 r dr
2 0

π 1 1
= · − cos r2 0
2 2
π
= (1 − cos 1) .
4

5.
Solução:
O esboço de D está representado na figura que se segue.

2 D

B A

O x

A circunferência x2 + (y − 2)2 = 4 se, e somente se, x2 + y 2 − 4y = 0 se


transforma em r2 − 4r sen θ = 0, isto é, r = 4 sen θ.
A circunferência x2 + y 2 = 4 se transforma em r2 = 4, isto é, r = 2.
−−→ −−→
A região D é “varrida” girando o raio OA, no sentido anti-horário, até o raio OB.
Os pontos A e B são interseções das duas circunferências. Então, resolvendo o
sistema 
 x2 + y 2 − 4y = 0
 x2 + y 2 = 4
temos:

4y = 4 ⇒ y = 1 , x = ± 3 .
Então:
√  √ 
A= 3,1 , B = − 3,1 .

172
O ângulo de inclinação do segmento OA é:
1 π
tg θ = √ ⇒ θ= .
3 6

O ângulo de inclinação do segmento OB é:


π 5π
θ=π− ⇒ θ= .
6 6

Assim, conseguimos transformar a região D em outra região Drθ expressa em


coordenadas polares:

Drθ = (r, θ) | 2 ≤ r ≤ 4 sen θ , π/6 ≤ θ ≤ 5π/6 .
ZZ
Como A(D) = dxdy então:
D

ZZ Z 5π/6Z 4 sen θ
A(D) = r drdθ = r drdθ
Drθ π/6 2
Z 5π/6 h i4 sen θ
1
= r2 dθ
2 π/6 2

Z 5π/6
1 
= 16 sen2 θ − 4 dθ .
2 π/6

Da trigonometria, temos:
1 − cos 2θ
sen2 θ = .
2
Logo,
Z 5π/6
1 
A(D) = 16 sen2 θ − 4 dθ
2 π/6
Z 5π/6  
1 1 − cos 2θ
= 16 − 4 dθ
2 π/6 2
Z 5π/6
1
= (4 − 8 cos 2θ) dθ
2 π/6
h i5π/6
1
= 4θ − 4 sen 2θ
2 π/6
h   i
5π 5π π π
= 2 − sen − − sen
6 3 6 3
 √√ 
2π 3 3
= 2 + +
3 2 2

2π √ 
= 2 + 3 u.a.
3

173
6.
Solução:
a) Temos
Z 4Z √4y−y2 p ZZ p
I= x2 + y2 dxdy = x2 + y 2 dxdy
0 0 D
onde n o
p
2
D = (x, y) | 0 ≤ y ≤ 4 , 0 ≤ x ≤ 4y − y .
p
De x = 4y − y 2 temos x2 + y 2 − 4y = 0, com x ≥ 0 donde x2 + (y − 2)2 = 4,
com x ≥ 0. Então o esboço de D está representado na figura que se segue.
y

r = 4 sen θ

D
2

x
r=0

Passando x2 + y 2 − 4y = 0 para coordenadas polares, temos r2 = 4r sen θ ou


r = 4 sen θ. Assim, a região D pode ser descrita em coordenadas polares por:

 0 ≤ θ ≤ π/2
Drθ : .
 0 ≤ r ≤ 4 sen θ

Então,
ZZ √ ZZ Z π/2Z 4 sen θ
2
I= r2 r drdθ = r drdθ = r2 drdθ
Drθ Drθ 0 0
Z π/2 h 3 i4 sen θ
r
= dθ
0 3 0
Z π/2
64
= sen3 θ dθ
3 0
Z π/2
64 
= 1 − cos2 θ sen θ dθ .
3 0

174
Fazendo u = cos θ temos du = − sen θ dθ. Para θ = 0 temos u = 1 e para
θ = π/2 temos u = 0. Então:
Z 0
u3 0
h i
64  64 64 2 128
I=− 1 − u2 du = − u− = · = .
3 1 3 3 1 3 3 9

b) Temos
Z 0 Z 0 ZZ
2 2
I= √
p dydx = p dxdy
−1 − 1−x2 1 + x2 + y 2 D 1 + x2 + y 2

onde n √ o
2
D = (x, y) | −1 ≤ x ≤ 0 , − 1 − x ≤ y ≤ 0 .

Pela descrição da região D, vemos que ela é limitada inferiormente pela



curva y = − 1 − x2 , se, e somente se, x2 + y 2 = 1, com y ≥ 0 e limi-
tada superiormente pela reta y = 0 (eixo x) e, além disso, a projeção de D
sobre o eixo x é o intervalo [−1, 0]. Com estas informações temos que o esboço
de D é:

−1
x
D r=0

−1
r=1

Em coordenadas polares:



 x = r cos θ

y = r sen θ


 dxdy = rdrdθ
x2 + y 2 r2

=

A região D se transforma em:


(
π ≤ θ ≤ 3π/2
Drθ :
0≤r≤1

A integral I se transforma em:

175
ZZ Z 1Z 3π/2
2 r
I= r drdθ = 2 dθdr
Drθ 1+r 0 π 1+r
Z 1
π r
= 2 dr
2 0 1+r
Z 1
1+r−1
= π dr
0 1+r
Z 1  
1+r 1
= π − dr
0 1+r 1+r
Z 1  
1
= π 1− dr
0 1+r

= π [r − ln(1 + r)]10

= π [(1 − ln 2) − (0 − ln 1)]

= π(1 − ln 2) .
c) Temos
Z 1Z √ ZZ p
1+ 1−y 2 p
I= 2 2
x + y dxdy = x2 + y 2 dxdy
0 y D

onde n o
p
2
D = (x, y) | 0 ≤ y ≤ 1 , y ≤ x ≤ 1 + 1 − y .
Da descrição de D vemos que a sua projeção sobre o eixo y é o intervalo [0, 1]
e sua fronteira lateral da esquerda é a reta x = y e a da direita é a curva
p p
x = 1 + 1 − y 2 (se, e somente se, x − 1 = 1 − y 2 , com x ≥ 1, se e somente
se (x − 1)2 + y 2 = 1, com x ≥ 1). Levando em conta essas informações temos
o esboço de D:

1
r = 2 cos θ

D
π
4
1 2 x
r=0

176
Passando para coordenadas polares, temos:




 x = r cos θ

y = r sen θ
.


 dxdy = rdrdθ
x + y2
2
r2

=

A equação polar da circunferência (x−1)2 +y 2 = 1 ou x2 +y 2 = 2x é r = 2 cos θ.


Logo, a região Drθ é dada por:

(
0 ≤ θ ≤ π/4
Drθ :
0 ≤ r ≤ 2 cos θ

Assim,

ZZ √ Z π/4Z 2 cos θ
I= r2 r drdθ = r2 drdθ
Drθ 0 0
Z π h 3 i2 cos θ
8 r
= 4 dθ
3 0 3 0
Z π/4
8
= cos3 θ dθ
3 0
Z π/4
8 
= 1 − sen2 θ cos θ dθ
3 0
h iπ/4
8 sen3 θ
= sen θ −
3 3 0
"√  √ 3 #
8 2 1 2
= −
3 2 3 2
√  
8 2 1
= · 1−
3 2 6

4 2 5
= ·
3 6

10 2
= .
9

177
7.
Solução:

Passando a circunferência x2 + y 2 − 2y = 0 y
para coordenadas polares temos 2

ρ2 = 2ρ sen θ ⇒ ρ = 2 sen θ .
10
1 B

Da figura ao lado temos


x
B ′ : 0 ≤ θ ≤ π/2 , 0 ≤ ρ ≤ 2 sen θ .

Então: ZZ ZZ
xy dxdy = ρ cos θρ sen θρ dρdθ
B B′
ZZ
= ρ3 cos θ sen θ dρdθ
B′
Z π/2Z 2 sen θ
= ρ3 cos θ sen θ dρdθ
0 0
Z π/2  2 sen θ
ρ4
= cos θ sen θ dθ
0 4
0
Z π/2
= 4 sen4 θ cos θ sen θ dθ
0
Z π/2
= 4 sen5 θ cos θ dθ
0
 π/2
sen6 θ
= 4
6
0

2
= .
3

8.
Solução:
ZZ
1
Temos I = p dxdy, onde D é dado por:
D x + y2
2

(
1≤y≤3
D:
0≤x≤y

178
e cujo esboço está representado na figura a seguir.

y
r = r2 (θ)
3
y=x
(r, θ)

1
r = r1 (θ)

Passando para coordenadas polares, temos




 x= r cos θ

y= r sen θ


 dxdy = rdrdθ
 2
x + y2 = r2

Transformando y = 1 e y = 3 para coordenadas polares, temos r sen θ = 1


1
e r sen θ = 3 ou r = r1 (θ) = = csc θ e r = r2 (θ) = 3 csc θ. Então o
sen θ
conjunto Drθ é dado por:

(
π/4 ≤ θ ≤ π/2
Drθ :
csc θ ≤ r ≤ 3 csc θ

Logo,

179
ZZ
1
I = √ · r drdθ
Drθ r2
ZZ
= drdθ
Drθ
Z π/2Z 3 csc θ
= drdθ
π/4 csc θ
Z π/2
= 2 csc θ dθ
π/4
h iπ/2
= 2 ln csc θ − cotg θ
π/4

 √ 
= 2 ln(1 − 0) − ln( 2 − 1)

= −2 ln( 2 − 1)
 
1
= 2 ln √
2−1

= 2 ln( 2 + 1) .

9.
Solução:
O esboço de W está representado na figura que se segue.

z
2 “teto”

1 1
x y
D (“piso”)

180
Observemos que o “teto” do sólido W é uma porção da esfera x2 + y 2 + z 2 = 4,
p
donde z = 4 − x2 − y 2 = f (x, y). O “piso” de W é o disco D : x2 + y 2 ≤ 1.
Então
ZZ ZZ p
V (W ) = f (x, y) dxdy = 4 − x2 − y 2 dxdy .
D D

Passando para coordenadas polares, temos




 x= r cos θ

y= r sen θ


 dxdy = rdrdθ
 2
x + y2 = r2

O conjunto Drθ é dado por: 0 ≤ r ≤ 1 e 0 ≤ θ ≤ 2π. Então:

ZZ √ Z 1 Z 2π

2 1/2
V (W ) = 4− r2 · r drdθ = 4−r r dθdr
Drθ 0 0
Z 1 1/2
= 2π 4 − r2 r dr .
0

1
Temos d(4 − r2 ) = −2r dr, donde r dr = − d(4 − r2 ). Logo,
2

 Z 1
1 1/2
V (W ) = 2π − 4 − r2 d(4 − r2 )
2 0
h 3/2 i1
2
= −π · · 4 − r2
3 0

2π 3/2 
= − 3 − 43/2
3

 √ 
= 8 − 3 3 u.v.
3

10.
Solução:
Identificando as superfı́cies z = 0 (plano xy), x2 +y 2 = 2y, isto é, x2 +(y − 1)2=1
(superfı́cie cilı́ndrica circular de diretriz circunferência no plano xy e geratrizes
p
paralelas ao eixo z) e z = x2 + y 2 (superfı́cie cônica).

181
z
y
2 2

D
1
W

x
1
2 y
D

Observe na figura acima que o sólido W está abaixo do gráfico de


p
z = f (x, y) = x2 + y 2 e está acima daZ Zregião D limitada pela circun-
ferência x2 + y 2 = 2y. Como V (W ) = f (x, y) dxdy então obtemos
ZZ p D

V (W ) = x2 + y 2 dxdy. Passando a equação x2 + y 2 = 2y para coorde-


D
nadas polares, temos r2 = 2r sen θ ou r = 2 sen θ. Observe que
ZZ ZZ
V (W ) = r · r drdθ = r2 drdθ
Drθ Drθ

onde Drθ = (r, θ) ∈ R2 | 0 ≤ θ ≤ π , 0 ≤ r ≤ 2 sen θ . Então
Z πZ 2 sen θ Z π  2 sen θ
r3
V (W ) = r2 drdθ = dθ
0 0 0 3 0
Z π
8
= sen3 θ dθ
3 0
Z π
8 
= 1 − cos2 θ sen θ dθ
3 0
Z π
8 
= − 1 − cos2 θ d (cos θ)
3 0
 π
8 cos3 θ
= − cos θ −
3 3 0

8
h 1
  1
i
= − −1 + − 1−
3 3 3
8 2
 
= − −2 +
3 3
32
= u.v.
9

182
Aula 3

1.
Solução:
Consideremos o eixo x passando pela base e o eixo y coincidindo com a mediatriz
relativa à base do triângulo.

y
−x + y = 5 x+y =5
5
x=5−y
x = −(5 − y)

D
−5 5
x

Como D é homogênea e é simétrica em relação ao eixo y, então pela observação


5, segue que x = 0. Temos
ZZ
y dxdy
D
y= ,
A(D)
onde
1
× 10 × 5 = 25 u.a.
A(D) =
2

Como D = (x, y) ∈ R2 | 0 ≤ y ≤ 5 , −(5 − y) ≤ x ≤ 5 − y , então
ZZ Z 5Z 5−y Z 5
 5−y
y dxdy = y dxdy = y x −(5−y) dy
D 0 −(5−y) 0
Z 5
= 2 y(5 − y) dy
0
Z 5
= 2 (5y − y 2 ) dy
0
h i5
5y 2 y3
= 2 −
2 3 0

125
= .
3

Logo,
125
3 5
y= = .
25 3
5
Assim, o centro de massa situa-se a cm da base, sobre sua mediatriz.
3

183
2.
Solução:
O esboço de D está representado na figura que se segue.

y
5
y= x
3 (3, 5)
5

3 x

ZZ
Seja (x, y) ∈ D. Então x ≥ 0. Logo, δ(x, y) = x. Como M = δ(x, y) dxdy,
ZZ D

então M = x dxdy.
D

Olhando D como tipo I, tem-se:


 0≤x≤3
D:
 0 ≤ y ≤ 5x
3

Então:

Z 3Z 5 x Z 3 Z 3 h i3
3  5x 5 5 x3
M= x dydx = x y 03 dx = x2 dx = · = 15 u.m.
0 0 0 3 0 3 3 0

O centro de massa (x, y) é dado por:

ZZ ZZ
xδ(x, y) dxdy x2 dxdy
D D
x= =
M M
ZZ ZZ
yδ(x, y) dxdy xy dxdy
D D
y= = .
M M

184
Tem-se: ZZ Z 3Z 5 Z
3
x 3
2 2 5
x dxdy = x dydx = x2 · x dx
D 0 0 0 3
Z 3
5
= x3 dx
3 0
h i3
5 x4
=
3 4 0

135
=
4
e ZZ Z 3Z 5 Z
x 3 h i 35 x
3
y2
xy dxdy = xy dydx = x dx
D 0 0 0 2 0
Z 3
25
= x3 dx
18 0
h i3
25 x4
=
18 4 0

225
=
8
Logo,
135/4 9
x= =
15 4
225/8 15
y= =
  15 8
9 15
donde (x, y) = , .
4 8

3.
Solução:
O esboço de D está representado na figura que se segue.
y

r=2
2

D 1 r=1

θ
1 2 x

p
Como a distância de P = (x, y) à origem é x2 + y 2 , então a densidade em P
p
é dada por δ(x, y) = k x2 + y 2 , onde k é uma constante de proporcionalidade.

185
O centro de massa (x, y) é dado por:
ZZ p
k x x2 + y 2 dA
D
x=
M
ZZ p
k y x2 + y 2 dA
D
y=
M
onde M é a massa de D.
p
Como a função f (x, y) = x x2 + y 2 é ı́mpar na variável x pois temos que
p
f (−x, y) = −xZZ px2 + y 2 = −f (x, y) e como D tem simetria em relação ao
eixo y então x x2 + y 2 dA = 0. Logo, x = 0.
D

Cálculo de M
ZZ ZZ p
M= δ(x, y) dxdy = k x2 + y 2 dA .
D D
Passando para coordenadas polares, temos:



 x = r cos θ

y = r sen θ


 dxdy = rdrdθ
 2
x + y 2 = r2
e Drθ é dado por: (
1≤r≤2
Drθ :
0≤θ≤π
Então,
ZZ √ Z 2 Z π Z 2 h 3 i2
r 7kπ
M =k r · r drdθ = k r2
2 dθdr = kπ 2
r dr = kπ = .
Drθ 1 0 1 3 1 3
ZZ p
Cálculo de y x2 + y 2 dA
D

ZZ p ZZ ZZ
y x2 + y2 dA = (r sen θ)r · r drdθ = r3 sen θ drdθ
D Drθ Drθ
Z 2 Z π
3
= r sen θ dθdr
1 0
Z 2
 π
= − cos θ 0
r3 dr
1
h 4 i2
r
= 2
4 1

15
= .
2
186
Logo,
15k
2 45
y= 7kπ
= .
3
14π
 
45
Assim, (x, y) = 0, .
14π

4.
Solução:
Inicialmente, calculemos os pontos de interseção da parábola y = x2 + 1 com a
reta y = x + 3:

(
y = x2 + 1
⇔ x2 +1 = x+3 ⇔ x2 −x−2 = 0 ⇔ x = 2 ou x = −1 .
y =x+3

Logo, os pontos de interseção são: (−1, 2) e (2, 5). Assim, o esboço de D está
representado na figura que se segue.

5 y =x+3

D
2

y = x2 + 1
1

−1 2 x

Descrevendo D como tipo I, temos:



 −1 ≤ x ≤ 2
D:
 x2 + 1 ≤ y ≤ x + 3 .

Como a distância de (x, y) ∈ D ao eixo x é |y| = y, pois yZ Z≥ 0, então a densidade


em (x, y) é dada por δ(x, y) = ky, k > 0. Como M = δ(x, y) dxdy então
D

187
ZZ Z 2 Z x+3
M =k y dxdy = k y dydx
D −1 x2 +1
Z 2 h ix+3
y2
= k dx
−1 2 x2 +1
Z 2 h
k 2
2 i 2
= (x + 3) − x + 1 dx
2 −1
Z 2
k 
= −x4 − x2 + 6x + 8 dx
2 −1
h i2
k x5 x3
= − − + 3x2 + 8x
2 5 3 −1
h   i
k 32 8 1 1
= − − + 12 + 16 − + +3−8
2 5 3 5 3
 
k 32 8 1 1
= − − + 28 − − + 5
2 5 3 5 3
117k
= u.m.
10
ZZ ZZ
xδ(x, y) dxdy k xy dxdy
D D
b) Sabemos que x = = . Temos que:
M M
ZZ Z 2Z x+3
xy dxdy = xy dydx
D −1 x2 +1
Z 2 h ix+3
y2
= x dx
−1 2 x2 +1
Z 2
1 
= x −x4 − x2 + 6x + 8 dx
2 −1
Z 2
1 
= −x5 − x3 + 6x2 + 8x dx
2 −1
h i2
1 x6 x4
= − − + 2x3 + 4x2
2 6 4 −1
h   i
1 64 1 1
= − − 4 + 16 + 16 − − − − 2 + 4
2 6 6 4
 
1 64 1 1
= − + 28 + + − 2
2 6 6 4
63
= .
8
Logo,
63k
8 35
x= 117k
= .
52
10
188
5.
Solução:
Considere o disco D de raio 5 cm centrado na origem como na figura a seguir.

D
5 x

Se D é homogêneo, então a densidade é constante, isto é, f (x, y) = k para


todo (x, y) ∈ D.
a) Temos

ZZ ZZ
2 2
 
I0 = x +y f (x, y) dxdy = k x2 + y 2 dxdy .
D D

Passando para coordenadas polares, temos

ZZ ZZ
2
I0 = k r · r drdθ = k r3 drdθ
Drθ Drθ

onde

 0 ≤ θ ≤ 2π
Drθ :
 0 ≤ r ≤ 5.

Então:

Z 5Z 2π Z 5 h 4 i5
3 r 625kπ
I0 = k r dθdr = 2kπ r3 dr = 2kπ = .
0 0 0 4 0 2

189
b) Considere o diâmetro contido no eixo x. Então
ZZ ZZ
2
Ix = y f (x, y) dxdy = k y 2 dxdy
D D
ZZ
= k r2 sen2 θ · r drdθ
Drθ
Z 5 Z 2π
3
= k r sen2 θ dθdr
0 0
h i2π Z 5
1 sen 2θ
= k· θ− r3 dr
2 2 0 0
h 4 i5
r
= kπ
4 0

625kπ
= .
4

6.
Solução:
O esboço da lâmina D é a figura que se segue.

D
2

1 x

A distância de um ponto (x, y) ∈ D ao ponto (1, 2) é dada pela expressão


p
(x − 1)2 + (y − 2)2 . Portanto a densidade Z Z é dada pela equação
p
δ(x, y) = k (x − 1)2 + (y − 2)2 . Como M = δ(x, y) dxdy, então temos
ZZ p D

que M = k (x − 1)2 + (y − 2)2 dxdy. Para calcular essa integral, devemos


D
usar a seguinte mudança de variáveis:

( (
u=x−1 x=u+1
ou .
v =y−1 y =v+1

190
Logo,

∂x ∂x
∂(x, y) ∂u
∂v 1 0
J= = = = 1,
∂(u, v) ∂y ∂y 0 1



∂u ∂v
donde dxdy = |J| dudv = dudv.
O disco D : (x − 1)2 + (y − 2)2 ≤ 1 é transformado em Duv : u2 + v 2 ≤ 1.
Aplicando a fórmula da mudança de variáveis, temos:
ZZ √
M =k u2 + v 2 dudv .
Duv

Passando para coordenadas polares, temos





 u = r cos θ

v = r sen θ


 dudv = rdrdθ
 2
u + v2 = r2

e Drθ é dado por:



 0≤r≤1
Drθ :
 0 ≤ θ ≤ 2π .

Assim,
ZZ √ ZZ Z 2πZ 1
2
M =k r2 · r drdθ = k r drdθ = k r2 drdθ
Drθ Drθ 0 0
Z 2π h 3 i1
r
= k dθ
0 3 0
Z 2π
k
= dθ
3 0

2kπ
= u.a.
3

7.
Solução:
É conveniente considerar um sistema de coordenadas como o da figura que se
segue, com o vértice do ângulo reto na origem e os catetos sobre os eixos coor-
denados, e a hipotenusa sobre a reta x + y = a.

191
y

a
x+y =a

y =a−x
D

a x
y=0


Podemos definir D por D = (x, y) ∈ R2 | 0 ≤ x ≤ a , 0 ≤ y ≤ a − x . Calcu-
lemos o momento de inércia em relação ao cateto que está sobre o eixo y : Iy .
Temos: ZZ
Iy = x2 δ(x, y) dA
D

onde δ(x, y) = c , c ∈ R. Portanto:


ZZ Z a Z a−x Z a
2 2
Iy = c x dA = c x dydx = c x2 (a − x) dx
D 0 0 0
Z a
= c (ax2 − x3 ) dx
0
h ia
ax3 x4
= c −
3 4 0
 4
a4 a
= c −
3 4
ca4
= .
12

8.
Solução:
Primeiro vamos encontrar as interseções das curvas, isto é, as interseções de
y = x com as circunferências.
(
y = x
2 2
⇔ 2x2 = 2x ⇔ x2 − x = 0 ⇔ x = 0 ou x = 1 .
x + y = 2x

Logo, as interseções são (0, 0) e (1, 1).


(
y = x
⇔ 2x2 − 4x = 0 ⇔ 2x(x − 2) = 0 ⇔ x = 0 ou x = 2 .
x2 + y 2 = 4x

192
Logo, as interseções são (0, 0) e (2, 2).
 √
 y = 3x

3 x2
⇔ x2 + = 2x
 x2 + y 2 = 2x
 3

⇔ 4x2 = 6x
⇔ 2x(2x − 3) = 0
3
⇔ x = 0 ou x = .
2
 √ 
3 3
Logo, as interseções são (0, 0) e 2
, 2
.
 √
 3
 y= x x2
3 ⇔ x2 + = 4x
 x2 + y 2 = 4x
 3

⇔ 4x2 = 12x
⇔ 4x(x − 3) = 0
⇔ x = 0 ou x = 3 .
√ 
Logo, as interseções são (0, 0) e 3, 3 .
De x2 + y 2 = 2x e x2 + y 2 = 4x, temos respectivamente, (x − 1)2 + y 2 = 1
e (x − 2)2 + y 2 = 4. Levando em conta essas informações, esboçamos a região
D na figura que se segue.

y y=x

3
y= x
D 3

1 2 3 4 x

p
A distância de (x, y) ∈ D à origem é x2 + y 2 . Como a densidade é inver-
k
samente proporcional a distância de (x, y) à origem, então δ(x, y) = p 2 2 ,
ZZ x +y
onde k > 0 é a constante de proporcionalidade. Como M = δ(x, y) dA,
ZZ D
1
então M = k p dA.
D x2 + y 2

193
Passando para coordenadas polares, temos




 x = r cos θ

y = r sen θ


 dxdy = rdrdθ
x2 + y 2 r2

=

Temos
r6=0
x2 + y 2 = 2x ⇒ r2 = 2r cos θ ⇒ r = 2 cos θ
r6=0
x2 + y 2 = 4x ⇒ r2 = 4r cos θ ⇒ r = 4 cos θ
π
y=x ⇒ θ=
√ 4
3x π
y= ⇒ θ=
3 6

Então o conjunto Drθ é dado por:



π/6 ≤ θ ≤ π/4
Drθ :
 2 cos θ ≤ r ≤ 4 cos θ

Logo,
ZZ
1
M = k √ r drdθ
Drθ r2
ZZ
= k drdθ
Drθ
Z π/4Z 4 cos θ
= k drdθ
π/6 2 cos θ
Z π/4
= 2k cos θ dθ
π/6
h iπ/4
= 2k sen θ
π/6
√ 
2 1
= 2k −
2 2
√ 
= k 2 − 1 u.m.

194
9.
Solução:
O esboço da placa D é a figura a seguir.

y
y=4

D
y =4−x

4 x

Observemos que D pode ser definido por:



 0≤x≤4
D: .
 4−x≤y ≤4

Como D é homogênea então a densidade é constante, δ(x, y) = k, e portanto,


1
a massa de D é igual a M = kA(D) = k · · 4 · 4 = 8k u.m.. Sabemos que o
2
momento de inércia em relação ao eixo y é dado por:
ZZ ZZ
2
Iy = x δ(x, y) dxdy = k x2 dxdy
D D

ou Z 4Z 4
Iy = k x2 dydx
0 4−x
Z 4
= k x2 (4 − 4 + x) dx
0
Z 4
= k x3 dx
0
h i4
x4
= k
4 0

= 64k

= 8(8k)

= 8M .

195
10.
Solução:
a) Completando quadrado em x2 + y 2 = 2x e x2 + y 2 = 4x, otemos a equação
(x − 1)2 + y 2 = 1 e (x − 2)2 + y 2 = 4. Logo, o esboço de D está representado
na figura que se segue.
y
y=x

B
P = (r, θ)
A
1 2 4 x
D

y = −x

A massa M é dada por:


ZZ
1
M= p dxdy .
D x + y2
2

Vamos usar coordenadas polares para descrever a região D. Temos:





 x = r cos θ

y = r sen θ


 dxdy = rdrdθ
 2
x + y 2 = r2

Logo, x2 + y 2 = 2x e x2 + y 2 = 4x acarretam em r2 = 2r cos θ e r2 = 4r cos θ, e


para r 6= 0, obtemos r = 2 cos θ e r = 4 cos θ. Para descrever D, consideramos
um ponto P = (x, y) = (r, θ) ∈ D. A semireta OP intercepta a fronteira de D
em A = (r1 (θ), θ) e B = (r2 (θ), θ), onde r1 (θ) = 2 cos θ e r2 (θ) = 4 cos θ.
Logo, 2 cos θ ≤ r ≤ 4 cos θ.
As retas y = x e y = −x acarretam em r sen θ = r cos θ e r sen θ = −r cos θ
ou tg θ = 1 e tg θ = −1, donde θ = π/4 e θ = −π/4, respectivamente. Logo,
−π/4 ≤ θ ≤ π/4. Assim,

D = (r, θ) ∈ R2 | −π/4 ≤ θ ≤ π/4 , 2 cos θ ≤ r ≤ 4 cos θ .
Então:
196
ZZ ZZ Z π/4 Z 4 cos θ
1
M= √ · r drdθ = drdθ = drdθ
2
Drθ r Drθ −π/4 2 cos θ
Z π/4
= 2 cos θ dθ
−π/4
h iπ/4
= 2 sen θ
−π/4

2
= 4·
2

= 2 2 u.m.
b) Temos:
ZZ ZZ
2 2 (x2 + y 2 )
I0 = (x + y )δ(x, y) dA = p dA
2 2
D D x +y
ZZ p
= x2 + y 2 dA
D
ZZ √
= r2 · r drdθ
Drθ
Z π/4 Z 4 cos θ
= r2 drdθ
−π/4 2 cos θ
Z π/4  4 cos θ
r3
= dθ
−π/4 3 2 cos θ
Z π/4
1 
= 64 cos3 θ − 8 cos3 θ dθ
3 −π/4
Z π/4
56
= cos3 θ dθ
3 −π/4
Z π/4
56
= (1 − sen2 θ) cos θ dθ
3 −π/4
 π/4
56 sen3 θ
= sen θ −
3 3 −π/4
"√  √ 3 #
56 2 1 2
= ·2 −
3 2 3 2
√ √ 
112 2 2
= −
3 2 12

112 5 2
= ·
3 12

140 2
= .
9
197
Aula 4

1.
Solução:
O esboço de W está representado na figura que se segue.
z
y
1
1

y =1−x
z =1−x−y
W
Dxy
1
y 1 x
1
z=0 y=0
x

Podemos descrever W por W = (x, y, z) | (x, y) ∈ Dxy e 0 ≤ z ≤ 1 − x − y
(
0≤x≤1
onde Dxy = projxOy
W : . Então:
0≤y ≤1−x
ZZZ ZZ Z 1−x−y 
x dxdydz = x dz dxdy
W Dxy 0
ZZ
= x(1 − x − y) dxdy
Dxy
Z 1Z 1−x
= x(1 − x − y) dydx
0 0
Z  1 1−x
y2
= x y − xy − dx
0 2 0
Z 1  
(1 − x)2
= x 1 − x − x(1 − x) − dx
0 2
Z
1 1 
= x 2 − 2x − 2x + 2x2 − 1 + 2x − x2 dx
2 0
Z
1 1 
= x − 2x2 + x3 dx
2 0
 1
1 x2 x3 x4
= −2 +
2 2 3 4 0
 
1 1 2 1
= − +
2 2 3 4
1
= .
24

198
2.
Solução:
a) Para esboçar a superfı́cie z + x2 = 4 (dita cilindro parabólico), traçamos no
plano y = 0, a parábola z = 4 − x2 . Como a equação não contém a variável y,
então por pontos da parábola (por exemplo A = (0, 0, 4), (2, 0, 0) e (−2, 0, 0))
traçamos paralelas ao eixo y. No plano x = 0, traçamos a reta y + z = 4, que
intercepta o cilindro em A = (0, 0, 4).
Para esboçar o plano, devemos traçar paralelas ao eixo x, por pontos da reta.
Em particular, por (0, 4, 0). Esta paralela intercepta o cilindro nos pontos B e
C. A curva que passa por B, A e C representa a curva interseção do plano com
o cilindro. Considerando os planos y = 0 e z = 0, temos o esboço de W .
z

A = (0, 0, 4)

W y =4−z

y=0
(−2, 0, 0) C

(2, 0, 0) (0, 4, 0) y
B
x
ZZZ
b) Temos V (W ) = dxdydz, onde W pode ser descrito por
 W
W = (x, y, z) | (x, z) ∈ Dxz e 0 ≤ y ≤ 4 − z de onde temos

que Dxz = (x, z) ∈ R2 | −2 ≤ x ≤ 2 , 0 ≤ z ≤ 4 − x2 .

z
4
z = 4 − x2

−2 2 x
z=0

199
Então: ZZ Z 
4−z
V (W ) = dy dxdz
Dxz 0
ZZ
= (4 − z) dxdz
Dxz
Z 2 Z 4−x2
= (4 − z) dzdx
−2 0
Z 2 h i4−x2
z2
= 4z − dx
−2 2 0
Z 2
1 
= 32 − 8x2 − 16 + 8x2 − x4 dx
2 −2
Z 2
1 
= 16 − x4 dx
2 −2
h i2
1 x5
= 16x −
2 5 −2
 
1 32
= · 2 32 −
2 5
128
= u.v.
5

3.
Solução:
O sólido W e sua projeção sobre o plano xy acham-se ilustrados nas figuras a
seguir.

z z

√ 2 y2 + z2 = 2
2
y2 + z2 = 2 y
√ y = 2x
2
D
W

2
√ x=0 x = y/2
2 D y
y y = 2x x
D
y = 2x
x x

200
n p o
Podemos descrever W como W = (x, y, z) ∈ R3 | 0 ≤ z ≤ 2 − y 2 , (x, y) ∈ D
onde D é a projeção de W sobre o plano xy. Temos:

ZZZ
M= f (x, y, z)dxdydz
W

onde f (x, y, z) é a densidade dada por f (x, y, z) = kz, com k constante de


proporcionalidade. Então:

ZZZ Z Z "Z √2−y2 #


M =k z dxdydz = k z dz dxdy
W D 0
ZZ
k 
= 2 − y 2 dxdy
2 D
Z √ Z
2 y/2
k 
= 2 − y 2 dxdy
2 0 0
Z √
2
k y 
= 2 − y 2 dy
2 0 2
Z √
2
k 
= 2y − y 3 dy
4 0
h i√ 2
k y4
= y2 −
4 4 0

k
= (2 − 1)
4
k
= u.m.
4

4.
Solução:
a) Para esboçar a superfı́cie y 2 + z = 4, 0 ≤ x ≤ 2 ou z = 4 − y 2 , 0 ≤ x ≤ 2,
esboçamos inicialmente a parábola z = 4 − y 2 no plano yz e em seguida, por
pontos da parábola traçamos paralelas ao eixo x (pois a variável x não está
presente na equação da parábola).

201
z

−2

2 y
2

Para esboçar a porção do plano y + z = 2, 0 ≤ x ≤ 2, traçamos a reta y + z = 2


no plano yz e por pontos da reta traçamos paralelas ao eixo x (pois a variável
não está presente na equação do plano).

2
y
2
x

Juntando as duas superfı́cies, temos o esboço do sólido W na figura que se segue.

202
z

−2

2
2 y

Resolvendo o sistema
(
z = 4 − y2
⇒ 4 − y 2 = 2 − y ⇒ y 2 − y − 2 = 0 ⇒ y = −1 ou y = 2 .
z =2−y
Portanto, projetando W sobre o plano xy, encontramos o retângulo

 −1 ≤ y ≤ 2
Dxy : .
 0≤x≤2

Assim, descrevemos o sólido W por



D = (x, y, z) ∈ R3 | (x, y) ∈ Dxy e 2 − y ≤ z ≤ 4 − y 2 .
Logo,
ZZZ ZZ Z 4−y 2 ZZ
ax ax
e dV = e dzdxdy = eax (2 − y 2 + y) dxdy
W Dxy 2−y Dxy
Z 2 Z 2
ax
= e (2 − y 2 + y) dydx
0 −1
Z 2
y2 2
h i
y3
= eax 2y − + dx
0 3 2 −1
Z 2
9
= eax dx
2 0
h i2
9 eax
=
2 a 0

9 
= e2a − 1 .
2a

203
5.
Solução:
a) Inicialmente, encontremos os pontos de interseção das duas parábolas:

(
z = y2
2
⇔ y 2 = 2 − y 2 ⇔ 2y 2 = 2 ⇔ y = ±1 .
z =2−y

Por pontos das parábolas traçamos paralelas ao eixo x (por exemplo, (0, 0, 2),
(0, 0, 0), (0, −1, 1) e (0, 1, 1)).
No plano xz, traçamos a reta x + z = 4, que intercepta as superfı́cies anteriores
em A e B. Por (0, 0, 1), traçamos uma paralela ao eixo x, que intercepta a reta
em C. Por C, traçamos uma paralela ao eixo y, que intercepta as superfı́cies
anteriores em D e E. Ligando A, E, B e D por uma curva fechada, obtemos o
sólido W na figura a seguir.

z
4

(0, 0, 2)
x=0
A
W
(0, 0, 1)

D C
E

y
B = (4, 0, 0) x=4−z
x

204
b) Projetando W sobre o plano yz encontramos Dyz na figura a seguir.

z
2

z = 2 − y2

z = y2

y

Então descrevemos W por W = (x, y, z) | (y, z) ∈ Dyz e 0 ≤ x ≤ 4 − z .
Temos,
ZZZ
V (W ) = dxdydz
W
ZZ Z 4−z 
= dx dydz
Dyz 0
ZZ
= (4 − z) dydz
Dyz
Z 1 Z 2−y 2
= (4 − z) dzdy
−1 y2
Z 1 h i2−y2
z2
= 4z − dy
−1 2 y2
Z 1h i2−y2
1
= 8z − z 2 dy
2 −1 y2
Z 1h
1  i
= 16 − 8y 2 − 4 + 4y 2 − y 4 − 8y 2 − y 4 dy
2 −1
Z 1
1
= (12 − 12y 2 ) dy
2 −1
h i1
1
= 12y − 4y 3
2 −1

1
= · 2(12 − 4)
2

= 8 u.v.

205
6.
Solução:
A figura a seguir mostra o sólido dado.
z

x=2−y

x 6
2

x = 6 − 2y 3

Projetando W sobre o plano yz encontramos a região D ilustrada na figura que


se segue.
z
z
2 2 y2 + z2 = 4

D
D

x 6 2 y
2

Então W pode ser descrito por



W = (x, y, z) ∈ R3 | 2 − y ≤ x ≤ 6 − 2y , (y, z) ∈ D .
Assim,
ZZZ Z Z Z 6−2y  ZZ
V (W ) = dxdydz = dx dydz = (4 − y) dydz .
W D 2−y D

206
Passando para coordenadas polares temos:

(

 y = r cos θ
0≤r≤2
z = r sen θ com .

 dydz = rdrdθ 0 ≤ θ ≤ π/2

Então
Z π/2Z 2 Z π/2Z 2 
V (W ) = (4 − r sen θ) rdrdθ = 4r − r2 sen θ drdθ
0 0 0 0
Z π/2 h i2
r3
= 2r2 − sen θ dθ
0 3 0
Z π/2  
8
= 8 − sen θ dθ
0 3
h iπ/2
8
= 8θ + cos θ
3 0

4
= (3π − 2) u.v.
3

7.
Solução:
Inicialmente encontremos as interseções das curvas z = 3x2 e z = 4 − x2 .

3x2 = 4 − x2 ⇒ 4x2 = 4 ⇒ x2 = 1 ⇒ x = ±1 e z = 3 .

Esboçando os cilindros parabólicos z = 3x2 , 0 ≤ z ≤ 3, z = 4 − x2 , 0 ≤ z ≤ 3


e o plano y + z = 6, podemos visualizar o nosso sólido W na figura que se segue.
z
6

4
3

W y =6−z

y=0

x 6 y

207
Projetando W sobre o plano xz encontramos a seguinte região D na figura a
seguir.

4 z = 4 − x2

z = 3x2

−1 1 x

Portanto:
ZZZ Z Z Z 6−z 
V (W ) = dxdydz = dy dxdz
W D 0
ZZ
= (6 − z) dxdz
D
Z 1 Z 4−x2
= (6 − z) dzdx
−1 3x2
Z 1h i4−x2
z2
= 6z −
−1 2 3x2
Z 1 
= 4 4 − 5x2 + x4 dx
−1

x5 1
h i
5x3
= 4 4x − +
3 5 −1

304
= u.v.
15

8.
Solução:
a) Inicialmente, traçamos o cilindro x2 + y 2 = 1, em seguida, traçamos no plano
yz, a reta y+z = 2, que intercepta o cilindro em A e B. Pelo ponto C = (0, 0, 2)
da reta, traçamos uma paralela ao eixo x, que intercepta o cilindro em D e E.
Ligando os pontos A, B, D e E, obtemos a curva interseção do cilindro com o
plano. Assim, temos o sólido W na figura a seguir.

208
z

E
2 z =2−y
D C

W A
−1

1 1
x 2 y
Dxy
z=0


b) Temos W = (x, y, z) | (x, y) ∈ Dxy , 0 ≤ z ≤ 2 − y onde Dxy é dado por
Dxy : x2 + y 2 ≤ 1. Temos:

ZZZ
M = δ(x, y, z) dV
W
ZZZ
= z dV
W
ZZ Z 2−y 
= z dz dxdy
Dxy 0
ZZ h 2 i2−y
z
= dxdy
Dxy 2 0
ZZ
1
= (4 − 4y + y 2 ) dy .
2 Dxy

Aplicando coordenadas polares, temos:



 x = r cos θ
y = r sen θ

 dxdy = rdrdθ

e
(
0≤r≤1
Drθ : .
0 ≤ θ ≤ 2π

209
Então:
ZZ
1 
M = 4 − 4r sen θ + r2 sen2 θ r drdθ
2 Drθ
Z 2πZ 1
1 
= 4r − 4r2 sen θ + r3 sen2 θ drdθ
2 0 0
Z 2π h i1
1 r3 r4
= 2r2 − 4 sen θ + sen2 θ dθ
2 0 3 4 0
Z 2π h i
1 4 1
= 2− sen θ + sen2 θ dθ
2 0 3 4
h  i
1 4 1 1 sen 2θ 2π
= 2θ + cos θ + · θ−
2 3 4 2 2 0
 
1 π
= 4π +
2 4
17π
= u.m.
8

9.
Solução:
Consideramos um sistema de coordenadas conforme a figura a seguir.

a y
a
x

210
Como a densidade em (x, y, z) é proporcional à distância z da base então
δ(x, y, z) = kz, onde k > 0 é uma constante de proporcionalidade. A massa do
sólido é

ZZZ ZZZ ZZZ


M= δ(x, y, z) dV = kz dV = k z dV
W W W
Z Z Z h 
= z dz dxdy
D 0
Z Z h 2 ih
z
= k dxdy
D 2 0
ZZ
kh2
= dxdy
2 D

kh2
= A(D)
2
kh2 2
= πa
2
kπh2 a2
= .
2

10.
Solução:
As figuras a seguir mostram o sólido W e a sua projeção sobre o plano xz.

y = −4 4

W
y=z

−4
−2

x 2 y

211
z
z
4
4
z = 4 − x2

D
D
−4
−2

x 2 y −2 2 x

O sólido W pode ser definido por


W = (x, y, z) ∈ R3 | −4 ≤ y ≤ z , (x, z) ∈ D .

onde

D = (x, z) ∈ R2 | −2 ≤ x ≤ 2 , 0 ≤ z ≤ 4 − x2 .

Como W é homogêneo, então a densidade é constante, isto é, f (x, y, z) = k.


Então:
ZZZ Z Z Z z 
M =k dxdydz = k dy dxdz
W D −4
ZZ
= k (z + 4) dxdz
D
Z 2 Z 4−x2
= k (z + 4) dzdx
−2 0
Z 2 h i4−x2
z2
= k + 4z dx
−2 2 0
Z 2
k 
= 48 − 16x2 + x4 dx
2 −2
h i2
k 16x3 x5
= 48x − +
2 3 5 −2

896k
= u.m.
15

212
Aula 5

1.
Solução:
O esboço de W está representado na figura que se segue.

z=4
4

z = x2 + y 2

2 y
2
D
x

A massa de W é dada por:


ZZZ ZZZ
1/2
M= δ(x, y, z) dV = x2 + y 2 dxdydz .
W W

Passando para coordenadas cilı́ndricas, tem-se:




 x = r cos θ




 y = r sen θ
z = z




 dxdydz = r drdθdz
x2 + y 2 r2

 =

Como W é dado por


(
(x, y) ∈ D
W :
x2 + y 2 ≤ z ≤ 4

então Wrθz é dado por



 0≤r≤2

Wrθz : 0 ≤ θ ≤ 2π

 r2 ≤ z ≤ 4

213
Assim,
ZZZ ZZZ
2

2 1/2
1/2
M= x +y dxdydz = r2 r drdθdz
W Wrθz
ZZZ
= r2 drdθdz
Wrθz
Z 2 Z 4Z 2π
2
= r dθdzdr
0 r2 0
Z 2 Z 4
2
= 2π r dzdr
0 r2
Z 2 
= 2π r2 4 − r2 dr
0
Z 2 
= 2π 4r2 − r4 dr
0
h i2
4r3 r5
= 2π −
3 5 0

128π
= u.m.
5
2.
Solução:
p 2
Sendo f (x, y, z) a densidade em (x, y, z), então f (x, y, z) = k x2 + y 2 =

= k x2 + y 2 , onde k é a constante de proporcionalidade.
Temos que
z
ZZZ
M = f (x, y, z) dxdydz 1
W
ZZZ W

= k x2 + y 2 dxdydz ,
W

onde D
n p o −1 1 y
W = (x, y, z) ∈ R3 | x2 + y 2 ≤ z ≤ 1 , (x, y) ∈ D ,

sendo D o disco dado por x2 + y 2 ≤ 1.


x
Passando para coordenadas cilı́ndricas, temos



 x = r cos θ

y = r sen θ
.


 z = z

dxdydz = r drdθdz

214
Temos que W é a imagem do conjunto


Q = (r, θ, z) ∈ R3 | 0 ≤ r ≤ 1 , 0 ≤ θ ≤ 2π , r ≤ z ≤ 1 .

Então
ZZZ ZZZ
2
M =k r r drdθdz = k r3 drdθdz
Q Q
Z 1Z 2πZ 1
= k r3 dzdθdr
0 0 r
Z 1Z 2π
= k r3 (1 − r) dθdr
0 0
Z 1 
= 2kπ r3 − r4 dr
0
h i1
r4 r5
= 2kπ −
4 5 0


= u.m.
10

3.
Solução:
q 
De x2 + y 2 + z 2 = 4 e z = 3 x2 + y 2 , tem-se x2 + y 2 = 1 que é a projeção,
no plano xy, da curva interseção das duas superfı́cies. A projeção do sólido W
é o disco D : x2 + y 2 ≤ 1.
O sólido W e sua projeção D são mostrados a seguir:

z
2
1

W 1 π
⇒ ⇒ sen φ = ⇒φ=
φ 2 2 6

D 1 2
1
2
x

215
Passando para coordenadas esféricas, tem-se:


 x = ρ sen φ cos θ




 y = ρ sen φ sen θ
z = ρ cos φ




 dxdydz = ρ2 sen φ dρdφdθ
 x 2 + y 2 + z 2 = ρ2

A equação da esfera x2 + y 2 + z 2 = 4 fica em coordenadas esféricas ρ2 = 4


ou ρ = 2. Então:

Wρφθ = (ρ, φ, θ) | 0 ≤ ρ ≤ 2 , 0 ≤ φ ≤ π/6 , 0 ≤ θ ≤ 2π .
Logo:
ZZZ p ZZZ
x2 + y2 + z2 dxdydz = ρ · ρ2 sen φ dρdφdθ
W Wρφθ
Z π/6Z 2Z 2π
= ρ3 sen φ dθdρdφ
0 0 0
Z π/6 Z 2
= 2π sen φ ρ3 dρdφ
0 0
h 4 i2 Z π/6
ρ
= 2π sen φ dφ
4 0 0
h iπ/6
= 8π − cos φ
0
 
π
= 8π 1 − cos
6
 √ 
3
= 8π 1−
2

= 4π(2 − 3) .

4.
Solução:
a) Inicialmente, passaremos de coordenadas cilı́ndricas a coordenadas cartesianas:



 x = r cos θ

y = r sen θ
.


 z = z

dxdydz = r dzdrdθ
Logo,
r2 sen θ dzdrdθ = r sen θ rdzdrdθ = y dxdydz .

216
√ p
A equação z = 4 − r2 em coordenadas cartesianas é z = 4 − x2 − y 2
ou x2 + y 2 + z 2 = 4, z ≥ 0. Logo, a imagem do conjunto Q dado por

 0≤θ≤π

Q: 0≤r≤2
 0 ≤ z ≤ √4 − r 2

é o sólido W dado por


n p o
W = (x, y, z) ∈ R3 | 0 ≤ z ≤ 4 − x2 − y 2 , (x, y) ∈ D ,

onde D é dado a seguir.

z y
2

W D

2 y −2 2 x
2

Então ZZZ
I= y dxdydz .
W

Passando para coordenadas esféricas:





 x = ρ sen ϕ cos θ

y = ρ sen ϕ sen θ
.


 z = ρ cos ϕ
ρ2 sen ϕ dρdϕdθ

dxdydz =

Temos que W é a imagem de



 0≤ρ≤2

Q: 0 ≤ ϕ ≤ π/2 .

 0≤θ≤π

217
Então

ZZZ
I = ρ sen ϕ sen θρ2 sen ϕ dρdϕdθ
Q
ZZZ
= ρ3 sen2 ϕ sen θ dρdϕdθ
Q
Z 2Z π/2Z π
= ρ3 sen2 ϕ sen θ dθdϕdρ
0 0 0

b)

Z 2Z π/2 h iπ
I = ρ3 sen2 ϕ − cos θ dϕdρ
0 0 0
Z 2Z π/2
= 2 ρ3 sen2 ϕ dϕdρ
0 0
Z 2 h iπ/2
1 sen 2ϕ
3
= 2 ρ · ϕ− dρ
0 2 2 0
Z 2
π
= ρ3 dρ
2 0
h i2
π ρ4
=
2 4 0

= 2π .

5.
Solução:
r
2 2 2 2 2 2 1 2
a) De x + y + z = 4z, temos x + y + (z − 2) = 4. De z = (x + y 2 ) ,
3
temos x2 + y 2 = 3z 2 . Logo, substituindo em x2 + y 2 + z 2 = 4z, temos
3z 2 + z 2 − 4z = 0 ou 4z 2 − 4z = 0, donde z = 0 ou z = 1.
Se z = 1, temos x2 + y 2 = 3. Assim, o cone intercepta a esfera segundo uma

circunferência de raio 3 , contida no plano z = 1. Portanto, o esboço de W
está representado na figura que se segue.

218
z
4

W
2

Usando coordenadas esféricas, temos



 x= ρ sen φ cos θ




 y= ρ sen φ sen θ
z= ρ cos φ .

 2


 dxdydz = ρ sen φdρdφdθ
x + y2 + z2 =
2
ρ2

Para encontrar o conjunto Wρφθ , devemos descrever as superfı́cies que compõem


a fronteira W .
De x2r
+ y 2 + z 2 = 4z, temos ρ2 = 4ρ cos φ ou ρ = 4 cos φ, se ρ 6= 0. De
1 2
z = (x + y 2 ) ou 3z 2 = x2 + y 2 , temos 3ρ2 cos2 φ = ρ2 sen2 φ, donde
3

tg2 φ = 3, se ρ 6= 0 ou tg φ = 3 , se ρ 6= 0. Logo, φ = π/3. Assim, Wρφθ
é dado por:


 0 ≤ θ ≤ 2π

Wρφθ : 0 ≤ φ ≤ π/3 .

 0 ≤ ρ ≤ 4 cos φ

ZZZ
Como V (W ) = dxdydz então:
W

219
ZZZ
V (W ) = ρ2 sen φ dρdφdθ
Wρφθ
Z 2πZ π/3 Z 4 cos φ
= sen φ ρ2 dρdφdθ
0 0 0
Z 2πZ π/3 h 3 i4 cos φ
ρ
= sen φ dφdθ
0 0 3 0
Z 2πZ π/3
64
= sen φ cos3 φ dφdθ
3 0 0
Z 2π h iπ/3
64 cos4 φ
= − dθ
3 0 4 0
Z 2π   
16 4
1
= − − 1 dθ
3 0 2
 
16 1
= 1− · 2π
3 16

= 10π u.v.

6.
Solução:
Na figura a seguir, representamos o conjunto W .

z
p
x2 + y 2 + (z − 1)2 = 1 ⇒ z = 1 + 1 − x2 − y 2

1 z = x2 + y 2

−1 1 y
1
D

220
Temos ZZZ
V (W ) = dxdydz
W
onde
n p o
W = (x, y, z) ∈ R3 | x2 + y 2 ≤ z ≤ 1 + 1 − x2 − y 2 , (x, y) ∈ D ,

sendo D a projeção de W sobre o plano xy.


As superfı́cies se interceptam segundo a curva x2 + y 2 = 1 e z = 1. Logo,

D = (x, y) ∈ R2 | x2 + y 2 ≤ 1 .

Passando para coordenadas cilı́ndricas, temos:



 x =
 r cos θ
y = r sen θ .

 z = z

Temos que W é a imagem do conjunto


n √ o
Q = (r, θ, z) ∈ R3 | 0 ≤ r ≤ 1 , 0 ≤ θ ≤ 2π , r2 ≤ z ≤ 1 + 1 − r2 .

Portanto, usando a fórmula


ZZZ ZZZ
f (x, y, z) dxdydz = f (r cos θ, r sen θ, z)r drdθdz
W Q

com f (x, y, z) = 1, temos


ZZZ
V (W ) = r drdθdz
Q
Z 1Z 2πZ √
1+ 1−r 2
= r dzdθdr
0 0 r2
Z 1Z 2π √ 
= r 1+ 1 − r2 − r2 dθdz
0 0
Z 1 √ 
= 2π r + r 1 − r2 − r3 dr
0
h i1
r2 1 2 3/2 r4
= 2π − · · 1 − r2 −
2 2 3 4 0
 
1 1 1
= 2π + −
2 3 4

= u.v.
6

221
7.
Solução:
O esboço de W está representado na figura que se segue.
z

ρ=2

1 P

ρ=1
π/4
y

Descrição de W em coordenadas esféricas

Consideremos um ponto P = (x, y, z) qualquer em W ; observemos que o


raio OP intercepta a superfı́cie do sólido (ou a fronteira do sólido) inicial-
mente em ρ = 1 e depois em ρ = 2. Logo, 1 ≤ ρ ≤ 2. O ângulo φ va-
ria de 0 (eixo z positivo) até π/4 (parede do cone); a variação do ângulo θ
é encontrada na projeção de W no plano xy : 0 ≤ θ ≤ 2π. Logo, Wρφθ é dado
por: 
 1≤ρ≤2

Wρφθ : 0 ≤ φ ≤ π/4

 0 ≤ θ ≤ 2π .

Como x2 + y 2 + z 2 = ρ2 e dV = ρ2 sen φ dρdφdθ, então


ZZZ ZZZ
1 1
2 2 2
dV = 2
· ρ2 sen φ dρdφdθ
W x +y +z Wρφθ ρ
Z π/4 Z 2 Z 2π
= sen φ dφ dρ dθ
0 1 0

 2  π/4
= 2π ρ 1 − cos φ 0
 √ 
− 2
= 2π +1
2
 √ 
= π 2− 2 .
222
8.
Solução:
Primeiramente, calculemos a interseção das duas superfı́cies.
 
 x2 + y 2 + (z − 1)2 = 1  x2 + y 2 + z 2 = 2z
z2
⇒ ⇒ + z 2 = 2z
 z = p3(x2 + y 2 )  z 2 = 3(x2 + y 2 ) 3

⇒ 4z 2 − 6z = 0
3
⇒ z = 0 ou z = .
2

Logo, a interseção se dá no plano z = 3/2, e a sua projeção no plano xy é a


circunferência x2 + y 2 = 3/4. Assim, o esboço de W é:

z
2

3/2 √
z= 3y
W 1

π/6


√ 3/2 y
3/2

x
√ p
Como o ângulo da reta z = 3 y (corte do cone z = 3(x2 + y 2 ) , considerando
x = 0) é o ângulo π/6, então φ varia de 0 (eixo z positivo) a π/2 − π/6 = π/3.
Transformando a equação x2 + y 2 + (z − 1)2 = 1 ou x2 + y 2 + z 2 = 2z para
coordenadas esféricas temos ρ2 = 2ρ cos φ donde ρ = 0 ou ρ = 2 cos φ. Isto
significa que ρ varia de 0 a 2 cos φ. A variação de θ é encontrada na projeção de
W no plano xy. Logo, 0 ≤ θ ≤ 2π. Assim, Wρφθ é dado por:

 0 ≤ θ ≤ 2π

Wρφθ : 0 ≤ φ ≤ π/3

 1 ≤ ρ ≤ 2 cos φ .

Como a distância de (x, y, z) ao plano z = 0 é |z| então δ(x, y, z) = |z|2 = z 2 .

223
A massa de W é:

ZZZ ZZZ
M= δ(x, y, z) dV = z 2 dV
W W
ZZZ
= (ρ cos φ)2 ρ2 sen φ dρdφdθ
Wρφθ
ZZZ
= ρ4 cos2 φ sen φ dρdφdθ
Wρφθ
Z 2πZ π/3 Z 2 cos φ
2
= cos φ sen φ ρ4 dρdφdθ
0 0 0
Z 2πZ π/3 h i2 cos φ
ρ5
= cos2 φ sen φ dφdθ
0 0 5 0
Z 2πZ π/3
32
= cos7 φ sen φ dφdθ
5 0 0
h iπ/3 Z 2π
32 − cos8 φ
= dθ
5 8 0 0
  8 
8π 1
= 1−
5 2

8π 28 − 1
= · 8
5 2
51π
= u.m.
32

9.
Solução:
ZZZ p
a) Temos que I = z x2 + y 2 dV , onde
W

n p o
W = (x, y, z) ∈ R3 | (x, y) ∈ Dxy e 0 ≤ z ≤ 4 − x2 − y 2

sendo Dxy dado por:


(
0≤x≤√ 2
Dxy :
0 ≤ y ≤ 4 − x2

cujo esboço está representado na figura a seguir.

224
y

Dxy

2 x

p
De z = 4 − x2 − y 2 , temos x2 + y 2 + z 2 = 4, z ≥ 0. Como Dxy é a projeção
de W sobre o plano xy, concluı́mos que o esboço de W é:

z
2

p
z= 4 − x2 − y 2

W
P = (x, y, z)

2
2 y
x
z=0

Vamos decrever W em coordenadas cilı́ndricas:





 x= r cos θ

y= r sen θ
.


 z= z

dV = r drdθdz

Seja P = (x, y, z) ∈ W . Então, a reta que passa por P , paralela ao eixo z,


p √
intercepta a fronteira de W em z = 0 e z = 4 − x2 − y 2 = 4 − r2 . Logo,

0 ≤ z ≤ 4 − r2 . As variações de r e θ são encontradas examinando Dxy .
Logo, 0 ≤ r ≤ 2 e 0 ≤ θ ≤ π/2. Então Wrθz é dado por:

 0≤r≤2

Wrθz : 0 ≤ θ ≤ π/2 .
 0 ≤ z ≤ √4 − r 2

225
Assim,
ZZZ Z Z √
√ 2 π/2Z 4−r 2
2
I= z r2 · r drdθdz = r z dzdθdr
Wrθz 0 0 0
Z 2 Z π/2 h 2 i√4−r2
2 z
= r dθdr
0 0 2 0
Z 2 Z π/2
1 2 2
= r (4 − r ) dθdr
2 0 0
Z 2
π 
= 4r2 − r4 dr
4 0
h i2
π 4r3 r5
= −
4 3 5 0

π 5 · 25 − 3 · 25
= ·
4 15
16π
= .
15

OBS.: Podemos resolver o exercı́cio usando coordenadas


esféricas. O conjunto Wρφθ é dado por:

 0≤ρ≤2

Wρφθ : 0 ≤ φ ≤ π/2 .

 0 ≤ θ ≤ π/2

Como 


 x= ρ sen φ cos θ

y= ρ sen φ sen θ
,


 z= ρ cos φ
ρ2 sen φ dρdφdθ

dV =
temos
p p
x2 + y 2 = ρ2 sen2 φ cos2 θ + ρ2 sen2 φ sen2 θ
p
= ρ2 sen2 φ

= ρ |sen φ|

= ρ sen φ

226
pois 0 ≤ φ ≤ π/2. Então,
ZZZ

I = (ρ cos φ)(ρ sen φ) ρ2 sen φ dρdφdθ
Wρφθ
Z 2 Z π/2 Z π/2
4 2
= ρ cos φ sen φ dθdφdρ
0 0 0
h iπ/2 Z 2
π sen3 φ
= · ρ4 dρ
2 3 0 0
h 5 i2
π ρ
=
6 5 0

16π
= .
15

10.
Solução:
p
O esboço do sólido W , limitado superiormente pelo cone z = x2 + y 2 , in-
feriormente pelo plano z = 0 e lateralmente pelo cilindro x2 + y 2 = 2y ou
x2 + (y − 1)2 = 1 está representado na figura que se segue.

2
p
z= x2 + y 2
W

P = (x, y, z)

1 2 y

z=0

227
Passando para coordenadas cilı́ndricas, temos:



 x= r cos θ




 y= r sen θ
z= z .




 dV = r drdθdz
x + y2 =
2
r2

Seja P = (x, y, z) ∈ W . A reta passando por P e paralela ao eixo z intercepta


p
a fronteira de W em z = 0 e z = x2 + y 2 = r. As variações de r e θ são
olhadas na projeção de W no plano xy : x2 + (y − 1)2 ≤ 1 ou x2 + y 2 ≤ 2y.

y
2

(x, y) r = 2 sen θ
1

x
r=0

De x2 + y 2 = 2y, temos r2 = 2r sen θ ou r = 2 sen θ se r 6= 0. Então:

(
0≤θ≤π
.
0 ≤ r ≤ 2 sen θ

Logo, Wrθz é dado por:


 0≤θ≤π

Wrθz : 0 ≤ r ≤ 2 sen θ .

 0≤z≤r

O momento de inércia em relação ao eixo z é:

ZZZ
Iz = (x2 + y 2 ) · δ(x, y, z) dV ,
W

228
onde δ(x, y, z) = k. Logo,:
ZZZ ZZZ

Iz = k x2 + y 2 dV = k r2 · r drdθdz
W Wrθz
Z πZ 2 sen θ Z r
3
= k r dzdrdθ
0 0 0
Z πZ 2 sen θ
= k r4 drdθ
0 0
Z π h 5 i2 sen θ
r
= k dθ
0 5 0
Z π
32k
= sen5 θ dθ
5 0
Z π
32k 2
= 1 − cos2 θ sen θ dθ
5 0
Z π
32k 
= 1 − 2 cos2 θ + cos4 θ sen θ dθ
5 0
h iπ
−32k 2 cos3 θ cos5 θ
= cos θ − +
5 3 5 0
 
64k 2 1
= 1− +
5 3 5
512
= k.
75

Aula 6

1.
Solução:
Se γ(t) = (cos t, sen t, t), então γ ′ (t) = (− sen t, cos t, 1), donde
√ √
kσ ′ (t)k = sen2 t + cos2 t + 1 = 2 .

Como ds = kσ ′ (t)k dt, então ds = 2 dt. Temos:
Z Z 2π √ √ Z 2π
1 1 1
2 2 2
ds = 2 2 2
· 2 dt = 2 dt
C x +y +z 0 cos t + sen t + t 0 1 + t2
√ 2π

= 2 arctg t
0

= 2 arctg 2π .

229
2.
Solução:
Por definição de integral de linha de campo escalar, temos

Z Z b
f (x, y, z) ds = f (γ(t)) kγ ′ (t)k dt .
C a

Se γ(t) = (4 cos t, 4 sen t, 3t) então γ ′ (t) = (−4 sen t, 4 cos t, 3) donde

p
kγ ′ (t)k = (−4 sen t)2 + (4 cos t)2 + 32

= 16 sen2 t + 16 cos2 t + 9
p
= 16 (sen2 t + cos2 t) + 9

= 16 + 9

= 25

= 5.

Logo,
Z p Z 2π p
x2 + y2 ds = (4 cos t)2 + (4 sen t)2 · 5 dt
C −2π
Z 2π p
= 5 16 (cos t2 + sen t2 ) dt
−2π
Z 2π
= 20 ds
−2π

= 20(2π + 2π)

= 80π .

3.
Solução:
a) Observe a figura a seguir.

230
y

P = (x, y)

a C
t y

−a O x Q a x

Seja P = (x, y) ∈ C. Seja t o ângulo (em radianos) formado pelo semi-eixo


positivo Ox e a semireta OP . Então
(
x = a cos t
y = a sen t,
com 0 ≤ t ≤ π.
Assim, uma parametrização para C é dada por σ(t) = (a cos t, a sen t), com
0 ≤ t ≤ π. Observe que com esta parametrização, C está sendo percorrida no
sentido anti-horário.
b) Temos σ ′ (t) = (−a sen t, a cos t). Donde
√ p
kσ ′ (t)k = a2 sen2 t + a2 cos2 t = a2 (sen2 t + cos2 t)
p
= a2 (1)

= a2

= |a|

= a.
Z Z π
Como f ds = f (σ(t)) kσ ′ (t)k dt, então:
C 0
Z Z π
2 2
 
x − y ds = a2 cos2 t − a2 sen2 t a dt
C 0
Z π
3

= a cos2 t − sen2 t dt
0
Z π
= a3 cos 2t dt
0
 π
3 sen 2t
= a
2 0

= 0.

231
4.
Solução:
a) O segmento AB pode ser parametrizado por σ(t) = A + t(B − A), com
0 ≤ t ≤ 1, donde

 
σ(t) = (1, 2, 3) + t (4, 5, 6) − (1, 2, 3)

= (1, 2, 3) + t(3, 3, 3)

= (1 + 3t, 2 + 3t, 3 + 3t),

com, 0 ≤ t ≤ 1.

b) Temos que σ ′ (t) = (3, 3, 3) o que implica kσ ′ (t)k = 3 3 . Como

Z Z 1
f ds = f (σ(t)) kσ ′ (t)k dt
C 0

então
Z Z 1 √
f ds = (1 + 3t + 2 + 3t + 3 + 3t)3 3 dt
C 0
√ Z 1
= 3 3 (6 + 9t) dt
0
√ Z 1
= 9 3 (2 + 3t) dt
0
√ h 3t2 1
i
= 9 3 2t +
2 0

63 3
= .
2

5.
Solução:
 
a 2 a2
Tem-se C : x2 +y 2 = ax ou C : x − +y 2 = . Logo, uma parametrização
 2 4
a a a
de C é γ(t) = + cos t , sen t , com 0 ≤ t ≤ 2π. Assim:
2 2 2

 
a a
γ ′ (t) = − sen t , cos t
2 2

γ (t) = a sen2 t + cos2 t = a .

2 2

232
a
Como ds = γ ′ (t) dt, então ds = dt. Assim:
2
Z Z 2π  
a a a a
(x − y) ds = + cos t − sen t dt
C 0 2 2 2 2
Z 2π
a2
= (1 + cos t − sen t) dt
4 0
h i2π
a2
= t + sen t + cos t
4 0

πa2
= .
2

6.
Solução:
Parametrizando C : x2 + y 2 = 2x, com y ≥ 0 ou C : (x − 1)2 + y 2 = 1,
com y ≥ 0, temos x = 1 + cos t e y = sen t. Como y ≥ 0 então sen t ≥ 0,
donde 0 ≤ t ≤ π. Então, uma parametrização de C é γ(t) = (1 + cos t, sen t),
com 0 ≤ t ≤ π. Logo, γ ′ (t) = (− sen t, cos t), donde kγ ′ (t)k = 1 e, portanto,
ds = kγ ′ (t)k dt = dt. Então
Z Z π Z π h i
2 2 1 − cos 2t 1 sen 2t π π
y ds = sen t dt = dt = t− = .
C 0 0 2 2 2 0 2

7.
Solução:
O esboço de C está representado na figura que se segue.

y
(1, 2)

C2

(1, 1)

C1

1 x

Como C = C1 ∪ C2 , por propriedade de integral de linha, temos


Z Z Z
8x ds = 8x ds + 8x ds .
C C1 C2

233
Z
Cálculo de 8x ds
C1

Uma parametrização de C1 é γ1 (t) = t, t2 , com 0 ≤ t ≤ 1. Logo, temos que

γ1′ (t) = (1, 2t), donde ds = kγ ′ (t)k dt = 1 + 4t2 dt. Então,
Z Z 1 √
8x ds = 8t 1 + 4t2 dt .
C1 0

Fazendo u = 1 + 4t2 , temos du = 8t dt. Para t = 0, temos u = 1 e para t = 1,


temos u = 5. Então
Z Z 5
2 √
h i5
2 
8x ds = u1/2 du = u3/2 = 5 5 − 1 .
C1 1 3 1 3

Z
Cálculo de 8x ds
C2

Uma parametrização de C2 é γ2 (t) = (1, t), com 1 ≤ t ≤ 2. Logo, γ2′ (t) = (0, 1),
de onde ds = kγ ′ (t)k dt = dt. Então,
Z Z 2 h i2
8x ds = 8 dt = 8t = 8 .
C2 1 1

Assim, Z √ √
10 5 − 2 10 5 + 22
8x ds = 3
+8= 3
.
C

8.
Solução:
a) A figura que se segue mostra um esboço da curva C.
z z
2 2

B C B C

1 1 2 y 1 2 y
1
2 A y=x A
2 y=x
x x
b) Seja (x, y, z) ∈ C. Então x = y e x2 +y 2 +z 2 = 4, z ≥ 0. Logo, 2y 2 +z 2 = 4
y2 z2
ou + = 1, z ≥ 0 (semi-elipse) é a equação da projeção de C sobre o
2 4
plano yz.

234
Temos
 √
 y = 2 cos t

z = 2 sen t
 x = y = √2 cos t .

√ √ √
Se A = ( 2 , 2 , 0), então t = 0. Se B = (1, 1, 2 ) então t = π/4.
√ √
Logo, uma parametrização de C é dada por σ(t) = ( 2 cos t, 2 cos t, 2 sen t),
com 0 ≤ t ≤ π/4.
c)
√ √
σ ′ (t) = (− 2 sen t, − 2 sen t, 2 cos t) ⇒ kσ ′ (t)k

= 2 sen2 t + 2 sen2 t + 4 cos2 t

= 4 sen2 t + 4 cos2 t

= 2.

Logo, ds = kσ ′ (t)k dt = 2 dt. Então:

Z Z π/4 √ Z π/4
2 2
x ds = ( 2 cos t) · 2 dt = 4 cos2 t dt
C 0 0
 π/4
1 sen 2t
= 4· t+
2 2 0
 
π 1
= 2 +
4 2
π+2
= .
2

9.
Solução:

Uma parametrização de C é dada por γ(t) = t , t2 , t , com 0 ≤ t ≤ 1. Então:

γ ′ (t) = (1, 2t, 1)


′ √ √
γ (t) = 1 + 4t2 + 1 = 2 + 4t2

ds = γ ′ (t) = 2 + 4t2 dt

235
Logo: Z Z 1 1/2
x ds = t 2 + 4t2 dt
C 0
Z 1
1 1/2 
= 2 + 4t2 d 2 + 4t2
8 0
h  i1
1 2 2 3/2
= · · 2 + 4t
8 3 0

1 
= 63/2 − 23/2
12
√ √
6 2
= − .
2 6
10.
Solução:
De (
x2 + y 2 + z 2 = 8 − 2(x + y)
,z≥0
x+y =2
temos x2 + (2 − x)2 + z 2 = 8 − 4, z ≥ 0 ou 2x2 − 4x + z 2 = 0, z ≥ 0 ou
z2
2(x − 1)2 + z 2 = 2, z ≥ 0 ou (x − 1)2 + = 1, z ≥ 0 que é a semi-elipse no
2
plano xz e representa a projeção de C nesse plano. Portanto,

 x = 1 + cos t

y = 2 − x = 1 − cos t
 z = √2 sen t

Como z ≥ 0 então 0 ≤ t ≤ π. Logo, uma parametrização para C é


 √ 
γ(t) = 1 + cos t, 1 − cos t, 2 sen t , 0 ≤ t ≤ π .
 √ 
Temos que γ ′ (t) = − sen t, sen t, 2 cos t . Logo, temos que
√ √ √
kγ ′ (t)k = sen2 t + sen2 t + 2 cos2 t = 2 donde ds = kγ ′ (t)k dt = 2 dt.
Portanto:
Z Z π
 √ √
2 2
(x + y )z ds = (1 + cos t)2 + (1 − cos t)2 2 sen t 2 dt
C 0
Z π 
= 2 2 + 2 cos2 t sen t dt
0
Z π 
= 4 1 + cos2 t sen t dt .
0
Como d(cos t) = − sen t dt então
Z
cos3 t π
h i  
1 32
(x2 + y 2 )z ds = −4 cos t + = −8 −1 − = .
C 3 0 3 3

236
Aula 7

1.
Solução:
Seja (x, y) ∈ C. Então x2 + y 2 /4 = 1, com x ≥ 0 e y ≥ 0. Logo,
(
x = cos t
y = 2 sen t
com 0 ≤ t ≤ π/2. Então σ(t) = (cos t, 2 sen t), com 0 ≤ t ≤ π/2 é
uma parametrização de C. Temos σ ′ (t) = (− sen t, 2 cos t), donde temos que
√ √
kσ ′ (t)k = sen2 t + 4 cos2 t = 1 + 3 cos2 t .
√ Z Z
Logo, ds = kσ ′ (t)k dt = 1 + 3 cos2 t dt. Como M = δ(x, y) ds = xy ds,
C C
então Z π/2 √
M= 2 cos t sen t 1 + 3 cos2 t dt .
0
2

Temos d 1 + 3 cos t = −6 cos t sen t dt, donde
1 
2 cos t sen t dt = − d 1 + 3 cos2 t .
3
Logo,
Z π/2
1 1/2 
M = − 1 + 3 cos2 t d 1 + 3 cos2 t
3 0
π/2
1 2 3/2
= − · 1 + 3 cos2 t
3 3
0

2 
= − 1 − (1 + 3)3/2
9
2
= − (1 − 8)
9
14
= u.m.
9
2.
Solução:
 √ 

2 2 5/2 t4
De γ(t) = t, t , temos que γ ′ (t) = (1, 2 t3/2 , t3 ), donde obtemos
5 4
√ q

kγ (t)k = 1+ 2t3
+ t6
= (1 + t3 )2 = 1 + t3 . Logo, obtemos
′ 3

ds = kγ (t)k dt = 1 + t dt. A primeira coordenada do centro de massa
do fio homogêneo é dada por
Z
x ds
C
x=
L

237
onde L é o comprimento de C, isto é,
Z b Z 2
t4 2
 h i

L= kγ (t)k dt = 1 + t3 dt = t + = 6.
a 0 4 0

Por outro lado,


Z Z 2 Z 2 h i2
3
 4
 t2 t5 32 42
x ds = t 1 + t dt = t+t dt = + =2+ = .
C 0 0 2 5 0 5 5

Logo:
42
5 7
x= = .
6 5
3.
Solução:
O esboço de C está representado abaixo.

z z
1 1
C C

1 y 1 y
1 1
x x

z
1
C

1 y
1
x

b) Seja (x, y, z) ∈ C. Então (x, y, z) satisfaz x2 + y 2 + z 2 = 1, y + z = 1 e


x ≥ 0. De y + z = 1, temos z = 1 − y. Substituindo em x2 + y 2 + z 2 = 1,
temos x2 + y 2 + (1 − y)2 = 1 ou x2 + 2y 2 − 2y = 0. Completando quadrado,

238
2
x2 (y − 1/2)
temos x2 + 2 (y − 1/2)2 = 1/2 ou + = 1 (elipse) que representa
1/2 1/4

2 1 1
a projeção de C sobre o plano xy. Então x = cos t, y = + sen t. Como
√ 2 2 2
2
x ≥ 0, então cos t ≥ 0, donde −π/2 ≤ t ≤ π/2. Como z = 1 − y, então
2
1 1
z = − sen t. Assim, uma parametrização diferenciável para C é dada por
2 2

√ 
2 1 1 1 1 π π
γ(t) = cos t , + sen t , − sen t ,− ≤t≤ .
2 2 2 2 2 2 2

 √ 
′ 2 1 1
c) Temos γ (t) = − sen t , cos t , − cos t , donde
2 2 2

r r √
′ 1 1 1 1 1 2
kγ (t)k = sen2 t + cos2 t + cos2 t = 2 2
sen t + cos t = .
2 4 4 2 2 2


′ 2
Como ds = kγ (t)k dt, então ds = dt. Temos
2
Z
M = δ(x, y, z) ds
C
Z
= yz ds
C
Z π/2   √
1 1 1 1 2
= + sen t − sen t dt
−π/2 2 2 2 2 2

√ Z π/2  
2 1 1
= − sen2 t dt
2 −π/2 4 4

√ Z π/2
2 
= 1 − sen2 t dt
8 −π/2

√ Z π/2
2
= cos2 t dt
8 −π/2
√ h i
2 1 sen 2t π/2
= · t+
8 2 2 −π/2

2 π
= ·2·
16 2


= u.m.
16

239
4.
Solução:

z = 2 − x − y = f (x, y)

S y

2
x

C (x, y, 0) 2 C
2 y
x

Seja S a superfı́cie lateral de base C contida no plano xy e altura


f (x, y) = 2 − x − y em cada (x, y) ∈ C. Então
Z Z
A(S) = f (x, y) ds = (2 − x − y) ds
C C

π
onde C é parametrizada por σ(t) = (2 cos t, 2 sen t), ≤ π ≤ 2π.
2
Logo, σ ′ (t) = (−2 sen t, 2 cos t), donde ds = kσ ′ (t)k dt = 2 dt. Então,

Z 2π
A(S) = (2 − 2 cos t − 2 sen t)2 dt
π/2
h i2π
= 4 t − sen t + cos t
π/2
h i 
π
= 4 (2π − 0 + 1) − −1+0
2
 

= 4 +2
2

= 6π + 8 u.a.

Logo, o preço da peça é igual a (6π + 8)M reais.

240
5.
Solução:
Esboçando o cilindro circular x2 + y 2 = 1 e o cilindro parabólico z = 1 − x2 ,
vemos que A1 , A2 , A3 e A4 são pontos de interseção.

z z
A2
1

A4

A3

1
1
A1 y y
x x

Ligando estes pontos por uma curva fechada temos a interseção das superfı́cies.
O esboço de S é:

z
1

1
1 y
x
C
Z Z 
Da teoria, temos que A(S) = f (x, y) ds = 1 − x2 ds. Uma para-
C C
metrização de C é dada por γ(t) = (cos t, sen t), com 0 ≤ t ≤ 2π, donde
γ ′ (t) = (− sen t, cos t).
p
Logo, ds = kγ ′ (t)k dt = (− sen t)2 + (cos t)2 dt = dt. Portanto:
Z 2π Z 2π h i
2
 2 1 sen 2t 2π
A(S) = 1 − cos t dt = sen t dt = t− = π u.a.
0 0 2 2 0

241
6.
Solução:
O esboço da cerca S está representado na figura que se segue.
z

S
f (x, y) = y

20 (x, y) 20
x y
C

Apresentemos uma parametrização para C. Fazendo u = x1/3 , v = y 1/3 e


a = 201/3 e substituindo na equação de C, temos u2 + v 2 = a2 , com u ≥ 0
e v ≥ 0, donde, u = a cos t e v = a sen t, com 0 ≤ t ≤ π/2. Como x = u3 ,
y = v 3 , então
x = a3 cos3 t = 20 cos3 t
.
y = a3 sen3 t = 20 sen3 t

Logo, γ(t) = 20 cos3 t, 20 sen3 t , com 0 ≤ t ≤ π/2 é uma parametrização

para C. Temos que γ ′ (t) = −60 cos2 t sen t, 60 sen2 t cos t , então:

kγ ′ (t)k = 60 cos4 t sen2 t + sen4 t cos2 t
p
= 60 cos2 t sen2 t (cos2 t + sen2 t)

= 60 |cos t sen t| = 60 cos t sen t .


pois 0 ≤ t ≤ π/2. Logo, ds = kσ ′ (t)k dt = 60 cos t sen t dt.
A área de um lado da superfı́cie é dada por
Z Z Z π/2 
A(S) = f (x, y) ds = y ds = 20 sen3 t 60 cos t sen t dt
C C 0
Z π/2
= 1200 sen4 t cos t dt
0
π/2
sen5 t
= 1200 ·
5
0

= 240 m2 .

Portanto, o pintor receberá 2 × 240 R = 480 R reais.

242
7.
Solução:
y
Consideremos o fio C sobre o eixo
x com a extremidade da esquerda
na origem e da direita em x = L. C

Então o eixo perpendicular ao fio O L x


passando pela origem é o eixo y.

Uma parametrização do fio C é dada por σ(t) = (t, 0), com 0 ≤ t ≤ L. Como
σ ′ (t) = (1, 0) então kσ ′ (t)k = 1. Logo, ds = kσ ′ (t)k dt = dt.
O momento de inércia em relação ao eixo y é:
Z Z
Iy = x δ(x, y) ds = k x2 ds
2
C C
pois o fio é homogêneo. Logo,
Z L
L
2 t3 kL3
Iy = k t dt = k = .
0 3 3
0

Como δ(x, y) = k , k ∈ R então M = kL. Logo,


M L2
Iy = .
3

8.
Solução:
Uma parametrização de C é γ(t) = (t, t), com 1 ≤ t ≤ 2, donde γ ′ (t) = (1, 1)

e ds = kγ ′ (t)k dt = 2 dt. Como a distância de (x, y) ∈ C ao eixo y é |x| = x
então a densidade é δ(x, y) = kx, com k ∈ R. Como a distância de (x, y) ∈ C
ao eixo E: y = −1 é r(x, y) = y + 1, então o momento de inércia é:
Z Z
2
IE = r (x, y)δ(x, y) ds = k (y + 1)2 x ds
C C
Z 2 √
= k (t + 1)2 t 2 dt
1

√ Z 2 
= k 2 t3 + 2t2 + t dt
1

√ h t4 2t3 t2 i2
= k 2 + +
4 3 2 1
√ h 16
 
1 2 1
i
= k 2 4+ +2 − + +
3 4 3 2

119 2 k
= .
12

243
9.
Solução:
a) Temos:  
x
− 0−y· √
∂P x2 +y 2 xy
= =
∂x x2 + y 2 (x2 + y2 )
3/2

e y
−x · √
∂Q x2 +y 2 −xy
= =
∂y (x2 + y 2 ) (x2 + y2 )
3/2


→ ∂P ∂Q
Logo, div F = + = 0.
∂x ∂y
→ 
− ∂Q ∂P

Por outro lado, rot F = 0, 0, − , onde
∂x ∂y
p x
x2 + y 2 − x · √
∂Q x2 +y 2 x2 + y 2 − x2 y2
= = =
∂x x2 + y 2 (x2 + y 2 )3/2 (x2 + y 2 )3/2
e
p y
x2 + y 2 − y · √
∂P x2 +y 2 x2 + y 2 − y 2 x2
=− =− =−
∂y x2 + y 2 (x2 + y 2 )3/2 (x2 + y 2 )3/2
Logo,    

→ y 2 + x2 1
rot F = 0, 0, 3/2
= 0, 0, p .
(x2 + y 2 ) x2 + y 2

b) Temos

→ ∂ (yez ) ∂ (xez ) ∂ (xyez )
div F = + + = 0 + 0 + xyez = xyez .
∂x ∂y ∂z

Temos

→ −
→ →


i j k



∂ ∂ ∂
rot F =
∂x ∂y ∂z


yez xez xye z


∂ ∂ ∂ ∂ ∂ ∂
−→ −→ −
∂y ∂z ∂z ∂x ∂x ∂y →
= i + j + k
z z
xe xyez xyez yez ye xez


→ −
→ −
→ − →
= (xez − xez ) i + (yez − yez ) j + (ez − ez ) k = 0 .

244
10.
Solução:
p
Se −→r = (x, y, z), então r = k−
→ 3/2
rk = x2 + y 2 + z 2 = x2 + y 2 + z 2 .
Então


 
r (x, y, z) x y z
3
= = , , .
r 3/2
(x2 + y 2 + z 2 )
3/2 3/2
(x2 + y 2 + z 2 )
3/2
(x2 + y 2 + z 2 ) (x2 + y 2 + z 2 )

Logo


r
div =
r3


r
=∇· 3
r
     
∂ x ∂ y ∂ z
= + + .
∂x (x2 + y 2 + z 2 )
3/2 ∂y (x2 + y 2 + z 2 )
3/2 ∂z (x2 + y 2 + z 2 )
3/2

Temos:
  3/2 3
1/2
∂ x x2 + y 2 + z 2 −x· 2
2 x +y
2
+ z2 · 2x
= 3
∂x (x2 + y 2 + z2)
3/2
(x2 + y2 + z2 )
1/2
(x2 + y 2 + z 2 ) (x2 + y 2 + z 2 − 3x2 )
=
(x2 + y 2 + z 2 )3
−2x2 + y 2 + z 2
= .
(x2 + y 2 + z 2 )5/2

Analogamente,
 
∂ y −2y 2 + x2 + z 2
=
∂y (x2 + y 2 + z2)
3/2
(x2 + y 2 + z 2 )
5/2

 
∂ z −2z 2 + x2 + y 2
= .
∂z (x2 + y 2 + z 2 )
3/2
(x2 + y 2 + z 2 )
5/2

Logo,


r
div =0
r3
e, portanto, −

r /r3 é solenoidal. Por outro lado,


→ −
→ −


i j k



→r −
→r

∂/∂x ∂/∂y ∂/∂z
rot 3 = ∇ × 3 =
r r


x y z

(x2 + y 2 + z 2 )3/2 (x2 + y 2 + z2)
3/2
(x2 + y 2 + z 2 )
3/2

245
Desenvolvendo o “determinante” em relação à primeira linha, temos:



    
r ∂ z ∂ y −

rot 3 = − i+
r ∂y (x2 + y 2 + z2)
3/2 ∂z (x2 + y 2 + z2)
3/2

    
∂ x ∂ z −

+ − j+
∂z (x2 + y2 + z2)
3/2 ∂x (x2 + y2 + z2)
3/2

    
∂ y ∂ x −

+ − k =
∂x (x2 + y2 + z2)
3/2 ∂y (x2 + y2 + z2)
3/2

 
−2yz 2yz −

= 5/2
+ 5/2
i+
(x2 + y 2 + z2) (x2 + y 2 + z2)
 
−2xz 2xz −

+ 5/2
+ 5/2
j+
(x2 + y 2 + z2) (x2 + y 2 + z2)
 
−2xy 2xy −

+ 5/2
+ 5/2
k =
(x2 + y2 + z2) (x2 + y2 + z2)



= 0 .

Logo, −

r /r3 é um campo irrotacional.

Aula 8

1.
Solução:
 

 −1 ≤ y ≤ 0 
 0≤y≤1
a) Temos que D = D1 ∪ D2 onde D1 : e D2 : .

 −y ≤ x ≤ 1 
 y≤x≤1

y
y

1 x 1
D1

D2
−1
1 x

246
Logo, o esboço de D está representado na figura que se segue.

y=x

D
1 x

y = −x

−1

b) Descrevendo D como tipo I, temos



D = (x, y) ∈ R2 | 0 ≤ x ≤ 1 , −x ≤ y ≤ x .

Então Z 1Z x
I= f (x, y) dydx .
0 −x

c) Temos
Z 1Z x Z 1 Z 1
1 2x −1/2
I= √ dydx = √ dx = 2 x 1 + x2 dx
0 −x 1 + x2 0 1 + x2 0

Como d 1 + x2 = 2x dx, então

Z 1 1/2 1 √

2 −1/2 2
 (1 + x2 ) 
I= 1+x d 1+x = =2 2 − 1 .
0 1/2
0

2.
Solução:
a) O esboço de D está representado na figura que se segue.

247
y

1
y=x

D
1 x

−1
y = −x

Como D é homogênea, então o centro de massa (x, y) é dado por


ZZ ZZ
x dA y dA
D D
x= , y=
A(D) A(D)
onde
1 π
A(D) = · π · 12 = .
4 4
Como f (x, y) = ZyZé uma função ı́mpar na variável y e D tem simetria em relação
ao eixo x então y dA = 0, donde y = 0.
D
ZZ
Calculemos x dA, usando coordenadas polares. Temos:
D

(

 x = r cos θ
−π/4 ≤ θ ≤ π/4
y = r sen θ e Drθ : .

 dA = rdrdθ 0≤r≤1

Temos,
ZZ ZZ ZZ
x dA = (r cos θ)r drdθ = r2 cos θ drdθ
D Drθ Drθ
Z π/4 Z 1
= cos θ r2 drdθ
−π/4 0

h 3 i1 Z π/4
r
= cos θ dθ
3 0 −π/4
h iπ/4
1
= sen θ
3 −π/4

2 2
= ·
3 2

2
= .
3

248
Logo,

2 √
3 4 2
x= π = ,

4

 √ 
4 2
donde o centro de massa está localizado em ,0 .

b) O momento de inércia em relação ao eixo x é dado por:

ZZ ZZ
2
Ix = y δ(x, y) dA = k y 2 dA
D D
ZZ
= k r3 sen2 θ drdθ
Drθ
Z π/4 Z 1
2
= k sen θ r3 drdθ
−π/4 0
Z π/4
k
= sen2 θ dθ
4 −π/4
h iπ/4
k 1 sen 2θ
= · θ−
4 2 2 −π/4
h   i
k π sen π/2 π sen(−π/2)
= − − − −
8 4 2 4 2
 
k π 1 π 1
= − + −
8 4 2 4 2
 
k π
= −1 .
8 2

3.
Solução:
 
a 2 a2
De x2 + y 2 = ax tem-se x − + y 2 = . O esboço de D está representado
2 4
na figura a seguir.

a/2 a x

249
Passando para coordenadas polares, tem-se:



 x = r cos θ

y = r sen θ


 dxdy = rdrdθ
 2
x + y 2 = r2

A circunferência x2 + y 2 = ax em coordenadas polares é r2 = ar cos θ, ou


r = a cos θ. Logo, Drθ é dado por:

 0 ≤ θ ≤ π/2
Drθ : .
 0 ≤ r ≤ a cos θ

Então: ZZ √
I = a2 − r2 · r drdθ
Drθ
Z π/2Z a cos θ 1/2
= a2 − r 2 r drdθ
0 0
Z π/2Z a cos θ
1 1/2 
= − a2 − r 2 d a2 − r2 dθ
2 0 0
Z π/2
a cos θ
1 2 3/2
= − · a2 − r 2 dθ
2 0 3
0
Z π/2 h
1 3/2 3/2 i
= − a2 − a2 cos2 θ − a2 dθ
3 0
Z π/2 h i
1 3/2
= − a2 sen2 θ − a3 dθ
3 0
Z π/2
1 
= − a3 sen3 θ − a3 dθ
3 0
Z !
π/2
a3 π
= − sen3 θ dθ − .
3 0 2

Da trigonometria, tem-se sen3 θ = sen2 θ · sen θ = 1 − cos2 θ sen θ. Então:
Z π/2 Z π/2
cos3 θ π/2
 h i
3 2
sen θ dθ = − 1 − cos θ d(cos θ) = − cos θ −
0 0 3 0

cos3 0
= cos 0 −
3
1
= 1−
3
2
=
3
 
a3 π 2
Logo I = − .
3 2 3
250
4.
Solução:
a) Temos que

Z 4Z √4y−y2 ZZ
I= dxdy = dxdy ,
0 0 D

n p o p
onde D = (x, y) | 0 ≤ y ≤ 4 , 0 ≤ x ≤ 4y − y2 . De x = 4y − y 2 , temos
x + y = 4y com x ≥ 0 ou x + (y − 2)2 = 4, com x ≥ 0.
2 2 2

Como 0 ≤ y ≤ 1, então o esboço de D está representado abaixo.

D
2

Passando para coordenadas polares, temos




 x = r cos θ

y = r sen θ
.


 dxdy = r drdθ
x2 + y 2 r2

=

A equação x2 + y 2 = 4y em coordenadas polares é: r2 = 4r sen θ ou r = 4 sen θ.


Portanto, a região D em coordenadas polares é:

(
0 ≤ θ ≤ π/2
Drθ : .
0 ≤ r ≤ 4 sen θ

251
Logo,
ZZ
I = r drdθ
Drθ
Z π/2Z 4 sen θ
= r drdθ
0 0
Z π/2 h 2 i4 sen θ
r
= dθ
0 2 0
Z π/2
= 8 sen2 θ dθ
0
Z π/2
1 − cos 2θ
= 8 dθ
0 2
h iπ/2
1 sen 2θ
= 8· θ−
2 2 0

= 2π .

b) Temos que

Z 2Z √4−y2 ZZ
2 2
 
I= x +y dxdy = x2 + y 2 dxdy
0 y D

n p o p
onde D = (x, y) | 0 ≤ y ≤ 2 , y ≤ x ≤ 4− y2 . De x = 4 − y 2 , temos
2 2
x + y = 4, com x ≥ 0. Da definição de D, vemos que D é limitado supe-
riormente por um arco da circunferência x2 + y 2 = 4 e inferiormente pela reta
y = x e a projeção de D sobre o eixo y é o intervalo [0, 2]. Logo, o esboço de
D está representado na figura que se segue.

π/4

2 x

252
Passando para coordenadas polares, temos:



 x = r cos θ

y = r sen θ
.


 dxdy = r drdθ
 2
x + y 2 = r2

A descrição de D em coordenadas polares é:



 π/4 ≤ θ ≤ π/2
Drθ : .
 0 ≤r≤ 2

Então,
ZZ ZZ Z 2 Z π/2
2 3 3
I= r · r drdθ = r drdθ = r dθdr
Drθ Drθ 0 π/4
Z 2
π
= r3 dr
4 0
h 4 i2
π r
=
4 4 0

= π.

5.
Solução:
Tem-se que ZZZ p
I= x2 + y 2 + z 2 dxdydz
W
n p o
com W = (x, y, z) | 0 ≤ z ≤ 1 − x2 − y 2 , (x, y) ∈ D onde D é a projeção
de W sobre o plano xy e é dada por:

 0≤x≤1
D: .
 0 ≤ y ≤ √1 − x2

253
y
1 p
y= 1 − x2 ⇒ x2 + y 2 = 1

1 x

p
De z = 1 − x2 − y 2 tem-se x2 + y 2 + z 2 = 1. Logo, W é a porção da esfera
x2 + y 2 + z 2 = 1, no primeiro octante, que se projeta no plano xy, segundo a
região D. O esboço de W está na figura que se segue.

z
1

1
1 y
x

Passando para coordenadas esféricas, tem-se:



 x = ρ sen φ cos θ




 y = ρ sen φ sen θ
z = ρ cos φ




 dxdydz = ρ2 sen φ dρdφdθ
x + y2 + z2
2
ρ2

 =

O conjunto Wρφθ é dado por:


 0≤ρ≤1

Wρφθ : 0 ≤ θ ≤ π/2

 0 ≤ φ ≤ π/2

254
Então: ZZZ
I = ρ · ρ2 sen φ dρdφdθ
Wρφθ
ZZZ
= ρ3 sen φ dρdφdθ
Wρφθ
Z π/2 Z 1 Z π/2
= sen φ ρ3 dθdρdφ
0 0 0
Z π/2 Z 1
π
= sen φ ρ3 dρdφ
2 0 0
h i1 Z π/2
π ρ4
= sen φ dφ
2 4 0 0
h iπ/2
π
= − cos φ
8 0

π
= .
8

6.
Solução:
O esboço do sólido W está na figura abaixo.

1 y
1

Da figura vemos que W pode ser descrito por:



W = (x, y, z) | (x, y) ∈ D e 0 ≤ z ≤ x2 + y 2

255
onde D é o disco x2 + y 2 ≤ 1. Como W é homogêneo, então o momento de
inércia em relação ao eixo z é:
ZZZ ZZZ
2 2
 
Iz = x + y δ(x, y, z) dV = k x2 + y 2 dV .
W W
Passando para coordenadas cilı́ndricas, temos


 x = r cos θ




 y = r sen θ
z = z .




 dV = r drdθdz
 x2 + y 2 = r 2

Como 0 ≤ z ≤ x2 + y 2 então 0 ≤ z ≤ r2 , donde Wrθz é dado por:



 0≤r≤1

Wrθz : 0 ≤ θ ≤ 2π .

 0 ≤ z ≤ r2

Então,
ZZZ ZZZ
2
Iz = k r · r drdθdz = k r3 drdθdz
Wrθz Wrθz
Z 2πZ 1Z r2
= k r3 dzdrdθ
0 0 0
Z 2πZ 1
= k r3 · r2 drdθ
0 0
Z 2πZ 1
= k r5 drdθ
0 0
Z 2π h 6 i1
r
= k dθ
0 6 0

k
= · 2π
6

= .
3
7.
Solução:
Como W é homogêneo então a densidade é constante, isto é, δ(x, y, z) = k. Da
definição de momento de inércia em relação ao eixo z, temos
ZZZ ZZZ
2 2
 
Iz = x + y δ(x, y, z) dV = k x2 + y 2 dV
W W
 2 2 2

onde W = (x, y, z) | x + y + z ≤ 4 .

256
Passando para coordenadas esféricas, temos:



 x = ρ sen φ cos θ

y = ρ sen φ sen θ
.


 z = ρ cos φ
dV = ρ2 sen φ dρdφdθ

O conjunto W é descrito em coordenadas esféricas por:



 0 ≤ θ ≤ 2π

Wρφθ : 0≤φ≤π .

 0≤ρ≤2

Então: ZZZ

Iz = k ρ2 sen2 φ ρ2 sen φ dρdφdθ
Wρφθ
ZZZ
= k ρ4 sen3 φ dρ dφ dθ
Wρφθ
Z π Z 2 Z 2π
3 4
= k sen φ ρ dθdρdφ
0 0 0
Z π h i2
ρ5 3
= 2kπ sen φ dφ
0 5 0
Z π
64kπ
= sen2 φ sen φ dφ
5 0
Z π
64kπ 
= 1 − cos2 φ sen φ dφ .
5 0
Fazendo u = cos φ temos du = − sen φ dφ. Para θ = 0, temos u = 1 e para
θ = π, temos u = −1. Então:
Z 1 Z 1
64kπ 2
 64kπ 
Iz = 1 − u (−du) = 1 − u2 du
5 −1 5 −1
h i1
64kπ u3
= u−
5 3 −1

256kπ
= .
15

8.
Solução:
a) Temos γ ′ (t) = (−4 sen t, 4 cos t, 3), donde
√ √ √
kγ ′ (t)k = 16 sen2 t + 16 cos2 t + 9 = 16 + 9 = 25 = 5 .

257
Como ds = kγ ′ (t)k dt então ds = 5 dt. A massa M é dada por
Z
M = δ(x, y, z)ds
C
Z

= x2 + y 2 ds
C
Z 4π 
= 16 cos2 t + 16 sen2 t 5 dt
0
Z 4π
= 80 dt = 320π .
0

b) O momento de inércia em relação ao eixo z é:


Z

Iz = x2 + y 2 δ(x, y, z) ds
C
Z
2
= x2 + y 2 ds
C
Z 4π 
= 16 cos2 t + 16 sen2 t 5 dt
0

= 5120π

= 16M .

9.
Solução:
De x2 + y 2 + z 2 = 2(x + y) temos (x − 1)2 + (y − 1)2 + z 2 = 2 (que é

uma esfera de centro (1, 1, 0) e raio 2 ). Substituindo y = 2 − x em
(x−1)2 +(y−1)2 +z 2 = 2, temos (x−1)2 +(1−x)2 +z 2 = 2 ou 2(x−1)2 +z 2 = 2
donde (x − 1)2 + z 2 /2 = 1 (elipse) representa a projeção de C no plano xz.
Então, se (x, y, z) ∈ C, x e z satisfazem à equação (x − 1)2 + z 2 /2 = 1. Logo,

x = 1 + cos t e z = 2 sen t, com 0 ≤ t ≤ 2π. Como y = 2 − x então
y = 1 − cos t. Assim, uma parametrização de C é

γ(t) = (1 + cos t, 1 − cos t, 2 sen t) , 0 ≤ t ≤ 2π .

Temos γ ′ (t) = (− sen t, sen t, 2 cos t), donde
√ √
kγ ′ (t)k = sen2 t + sen2 t + 2 cos2 t = 2.

258

Logo, ds = kγ ′ (t)kdt = 2 dt. Então,

Z Z 2π √ √ Z 2π 
xy ds = (1 + cos t)(1 − cos t) 2 dt = 2 1 − cos2 t dt
C 0 0

√ Z 2π
= 2 sen2 t dt
0
√ 1
h
sen 2t
i2π
= 2· t−
2 2 0

= 2 π.

10.
Solução:
 √ 
2 2 3/2
De γ(t) = t cos t , t sen t , 3
t temos


 √ 1/2 
γ (t) = cos t − t sen t , sen t + t cos t , 2 t

donde

kγ ′ (t)k =

= cos2 t − 2t sen t cos t + t2 sen2 t + sen2 t + 2t sen t cos t + t2 cos2 t + 2t
p
= 1 + t2 (sen2 t + cos2 t) + 2t

= 1 + t2 + 2t
p
= (1 + t)2

= |1 + t|

=1+t

pois 1 + t > 0. Logo, ds = kγ ′ (t)kdt = (1 + t)dt. Temos:


Z Z
2 2
 
Iz = x + y δ(x, y, z) ds = k x2 + y 2 ds =
C C
Z 1 Z 1 Z 1
 
=k t2 cos2 t + t2 sen2 t (1+t) dt = k 2
t (1+t) dt = k t2 + t3 dt =
0 0 0
h 3 4 i1  
t t 1 1 7k
=k + =k + = .
3 4 0 3 4 12

259
Aula 9

1.
Solução:
De 4x2 + 25y 2 = 100, temos x2 /25 + y 2 /4 = 1. Então, γ(t) = (5 cos t, 2 sen t),
com 0 ≤ t ≤ 2π é uma parametrização da elipse no sentido anti-horário. O
trabalho é dado por

Z Z

→ −
W = F · d→
r = − 3y dx + 3x dy
C C
Z 2π  
= (−6 sen t)(−5 sen t) + (15 cos t)(2 cos t) dt
0
Z 2π 
= 30 sen2 t + 30 cos2 t dt
0
Z 2π
= 30 dt
0

= 60π .

2.
Solução:
Z
→ →
Temos que W = F · d r . Para calcular a integral, dividimos C em quatro
C
caminhos C1 , C2 , C3 e C4 , conforme a figura a seguir.

C3
a

C4 C2

a x
C1

260
O caminho C1 tem equação vetorial (ou parametrização) σ(t) = (t, 0), com
t ∈ [0, a]. Portanto,
Z Z a Z a
→ → → 

F · d r= F (σ(t)) · σ (t) dt = t2 , 0 · (1, 0) dt
C1 0 0
Z a
= t2 dt
0
h 3 ia
t
=
3 0
3
a
= .
3
O caminho C2 tem equação vetorial σ(t) = (a, t), com t ∈ [0, a]. Portanto,
Z Z a Z a
→ → → 

F · d r= F (σ(t)) · σ (t) dt = a2 − t2 , 2at · (0, 1) dt
C2 0 0
Z a
= 2at dt
0
h ia
= at2
0

= a3 .
O caminho C3 tem equação vetorial σ(t) = (a − t, a), com t ∈ [0, a]. Portanto,
Z Z a
→ → →

F ·d r = F (σ(t)) · σ (t) dt
C3 0
Z a 
= (a − t)2 − a2 , 2a(a − t) · (−1, 0) dt
0
Z a 
= 2at − t2 dt
0
h ia
2t3
= at −
3 0

2a3
= .
3
O caminho C4 tem equação vetorial σ(t) = (0, a − t), com t ∈ [0, a]. Portanto,
Z Z a
→ → →

F · dr = F (σ(t)) · σ (t) dt
C4 0
Z a 
= − (a − t)2 , 0 · (0, −1) dt
0
Z a
= 0 dt
0
= 0.

261
Logo,
Z
→ →
W = F · dr
C
Z Z Z Z
→ → → → → → → →
= F · dr + F · dr + F · dr + F · dr
C1 C2 C3 C4

a3 2a3
= + a3 −
3 3

= 2a3 u.ω.

3.
y
Solução:
C3 (1, 1)
Temos C = C1 ∪ C2 ∪ C3 . Então
I Z Z Z C2

→ − → −
→ − → −
→ − → −
→ −
F ·d r = F ·d r + F ·d r + F ·d→
r .
C+ C1 C2 C3
(0, 0) (1, 0) x
C1

Z

→ −
Cálculo de F · d→
r
C1

Temos C1 : y = 0, com 0 ≤ x ≤ 1. Logo, dy = 0. Então


Z Z Z Z 1 h i1

→ − x3 1
F · d→
r = 2
x dx + (x + y) dy = 2
x dx = x2 dx = = .
C1 C1 C1 0 3 0 3

Z

→ −
Cálculo de F · d→
r
C2

Temos C2 : x = 1, com 0 ≤ y ≤ 1. Logo, dx = 0. Então


Z Z Z Z 1  1

→ − y2 3
F ·d→
r = 2
x dx+(x+y) dy = (1+y) dy = (1+y) dy = y + = .
C2 C2 C2 0 2 0
2

Z

→ −
Cálculo de F · d→
r
C3

Temos que C3− é a curva C3 percorrida no sentido contrário. Logo, C3− : y = x,


com 0 ≤ x ≤ 1, donde, dy = dx. Logo

262
Z Z Z

→ − −
→ −
F · d→
r =− F · d→
r = − x2 dx + (x + y) dy
C3 C3− C3−
Z
= − x2 dx + (x + x) dx
C3−
Z
= − (x2 + 2x) dx
C3−
Z 1
= − (x2 + 2x) dx
0
 1
x3
= − + x2
3 0
h1 i 4
= − +1 =− .
3 3
Portanto Z

→ − 1 3 4 1
F · d→
r = + − = .
C 3 2 3 2

4.
Solução:
A figura que se segue mostra um esboço da curva C.

z
z
C 8
8 C

(x, y, z)

(x, y, z) 4 4

8 y
x
y x

Para parametrizá-la, observamos que x, y e z satisfazem

 x2 + y 2 + (4 − y)2 = 16 ⇒ x2 + 2y 2 − 8y = 0
 x2 + y 2 + (z − 4)2 = 16
⇔ ⇒ x2 + 2(y − 2)2 = 8
 z =8−y
x2 (y − 2)2
⇒ + =1
8 4
263

que é uma elipse de centro (0, 2) e semi-eixos 2 2 e 2. Esta elipse é a projeção
de C sobre o plano xy.
Temos  √
 x = 2 2 cos t

y = 2 + 2 sen t

 z = 8 − (2 + 2 sen t) = 6 − 2 sen t

com 0 ≤ t ≤ 2π, donde:


 √
 dx = −2 2 sen t dt

dy = 2 cos t dt .

 dz = −2 cos t dt

Então
Z
zdx + ydy − xdz =
C
Z 2π h √ √ i
= (6 − 2 sen t)(−2 2 sen t) + (2 + 2 sen t)2 cos t − 2 2 cos t(−2 cos t) dt
0
Z 2π √ √ √ 
= − 12 2 sen t + 4 2 sen2 t + 4 cos t + 4 sen t cos t + 4 2 cos2 t dt
0
Z 2π √ √ 
= 4 2 − 12 2 sen t + 4 cos t + 4 sen t cos t dt
0
h √ √ i2π
= 4 2 t + 12 2 cos t + 4 sen t + 2 sen2 t
0

= 8 2π .

5.
Solução:
Ao interceptar a esfera x2 + y 2 + z 2 = 2(x + y) com o plano x + y = 2, obtém-se:

(2 − y)2 + y 2 + z 2 = 4 ⇒ 2y 2 − 4y + z 2 = 0

⇒ 2(y − 1)2 + z 2 = 2
z2
⇒ (y − 1)2 + =1
2

que é uma elipse no plano yz.


A elipse representa a projeção da curva C sobre o plano yz. Como C está
orientada no sentido anti-horário quando vista da origem, então a elipse está
orientada no sentido horário. O esboço de C está ilustrado a seguir.

264
z z

C C

2 y 2 y
2 2
x x

Devemos parametrizar C − . Tem-se:





 y=√ 1 + cos t

z = 2 sen t


 x = 2 − y = 2 − (1 + cos t) = 1 − cos t

0 ≤ t ≤ 2π

Portanto:


 √ 
C : γ(t) = 1 − cos t, 1 + cos t, 2 sen t , 0 ≤ t ≤ 2π
 √ 
γ ′ (t) = sen t, − sen t, 2 cos t .

Tem-se:
Z

→ −
W = F · d→
r
C
Z

→ −
= − F · d→
r
C−
Z 2π


= − F (γ(t)) · γ ′ (t) dt
0
Z 2π  √   √ 
= − 1 + cos t, 2 sen t, 1 − cos t · sen t, − sen t, 2 cos t dt
0
Z 2π  √ √ √ 
= − sen t + sen t cos t − 2 sen2 t + 2 cos t − 2 cos2 t dt
0
 √ 
= − − 2 · 2π

= 2 2π u.ω.

265
6.
Solução:
Esboçando os dois cilindros, vemos que A1 = (1, 0, 0), A2 = (−1, 0, 0)
e A3 = (0, 1/2, 1) são pontos de interseção. Ligando-os encontramos C.

z z
1 1 C
A3

1/2
−1 −1 A2

A1
1/2 1/2
x 1 y x 1 y

y Projeção de C no plano xy
1/2
x2 + 4y 2 = 1

−1 1 x

Se (x, y, z) ∈ C então (x, y, z) satisfaz


2
 x2 + y = 1 com y ≥ 0

1/4

 x2 + z 2 = 1 com z ≥ 0


então x = cos t e y = (1/2) sen t, com 0 ≤ t ≤ π. Como z = 1 − x2
√ √
então temos que z = 1 − cos2 t = sen2 t = sen t. Logo, obtemos que
γ(t) = (cos t, (1/2) sen t, sen t), com 0 ≤ t ≤ π é uma parametrização de
C, orientada de A1 para A2 . Temos dx = − sen t dt, dy = (1/2) cos t dt e
dz = cos t dt. Então

266
Z
− 2y dx + 3z dy + x dz =
C
Z π  
1 1 
= −2 · sen t (− sen t) + (3 sen t) · cos t + (cos t)(cos t) dt
0 2 2
Z π  
2 3 2
= sen t + sen t cos t + cos t dt
0 2
Z π  
3
= 1 + sen t cos t dt
0 2

3 sen2 t π
h i
= t+ ·
2 2 0

= π.

7.
Solução:
Temos 
 2 − x se x ≤ 2
|2 − x| =
 −2 + x se x > 2

Então 

 2 − (2 − x) = x se 0 ≤ x ≤ 2
C: y=

 2 − (−2 + x) = 4 − x se 2 < x ≤ 4 .

y
2

2 4 x


Pela expressão de F , vemos que calcular diretamente a integral é uma tarefa


penosa. Surge então uma pergunta natural: F é conservativo? Ou existirá


alguma função potencial de F ?
Observe que
∂Q ∂P
− = (sen xy − xy cos xy − 2y) − (sen xy − xy cos xy − 2y) = 0 .
∂x ∂y
→  ∂Q ∂P  −
− → − → −

Logo, rot F = − k = 0 . Isto significa que F pode ser conservativo.
∂x ∂y

267
Então, tentemos resolver

∂ϕ
= y sen xy − y 2 − 3 (1)
∂x
∂ϕ
= x sen xy − 2xy (2)
∂y
Integrando (1) e (2) em relação a x e y respectivamente, temos

ϕ(x, y) = − cos(xy) − xy 2 − 3x + f (y) (3)

ϕ(x, y) = − cos(xy) − xy 2 + g(x) (4)

De (3) e (4), vemos que se f (y) = 0 e g(x) = −3x então temos que


ϕ(x, y) = − cos(xy) − xy 2 − 3x é uma função potencial de F . Logo, o trabalho
é dado por
Z

→ −
W = F · d→ r = ϕ(0, 0) − ϕ(4, 0)
C

= (−1 − 0 − 0) − (−1 − 0 − 12)

= −1 + 13 = 12 u.w.

8.
Solução:
a) Devemos resolver
∂ϕ
= −4xe3y + zexz (1)
∂x
∂ϕ
= −6x2 e3y + 4y 2 (2)
∂y
∂ϕ
= xexz + cos(z) (3)
∂z
Integrando (1), (2) e (3) em relação a x, y e z, respectivamente, temos

ϕ(x, y, z) = −2x2 e3y + exz + f (y, z)


4y 3
ϕ(x, y, z) = −2x2 e3y + + g(x, z)
3
ϕ(x, y, z) = exz + sen(z) + h(x, y)

Devemos tomar f (y, z) = (4/3)y 3 + sen z, g(x, z) = exz + sen z e, também


h(x, y) = −2x2 e3y + (4/3)y 3 .

268
Então temos que ϕ(x, y, z) = −2x2 e3y + exz + (4/3)y 3 + sen z é uma função


potencial de F .
b) Então
Z

→ −
F · d→
r = ϕ (γ(1)) − ϕ (γ(0))
C
 
π
= ϕ 0, 1, − ϕ(2, 0, 0)
2
 
4
= 0 + 1 + + 1 − (−8 + 1 + 0 + 0)
3
10
= +7
3
31
= .
3

9.
Solução:

→  −

Seja F (x, y) = 2xy, x2 , com (x, y) ∈ R2 . Se provarmos que F é um campo


conservativo, então por propriedade, temos que a integral de linha de F é in-


dependente do caminho. Para isso, basta provar que F admite uma função

→ −

ϕ(x, y), tal que ∇ϕ = F em R2 , dita função potencial de F . Então, devemos


resolver o sistema ∇ϕ = F em R2 , isto é,

∂ϕ

 = 2xy (1)
 ∂x

 ∂ϕ = x2

(2)
∂y

Integrando (1) e (2) em relação a x e y, respectivamente, temos

ϕ(x, y) = x2 y + f (y)
ϕ(x, y) = x2 y + g(x)

onde f (y) e g(x) são “constantes” de integração. Tomando f (y) = 0 e g(x) = 0,




temos que ϕ(x, y) = x2 y é uma função potencial de F .
Z


Logo, F · d~r é independente do caminho e:
C

Z


F · d~r = ϕ(1, 4) − ϕ(2, −1) = 12 · 4 − 22 (−1) = 4 + 4 = 8 .
C

269
10.
Solução:
Como é extremamente difı́cil calcular a integral via definição, procuremos saber

→ −

se o campo F é conservativo, isto é, se existe ϕ(x, y, z) tal que ∇ϕ = F em


R3 = dom F . Devemos resolver o seguinte sistema:

∂ϕ


 = 2x sen z (1)

 ∂x

 ∂ϕ
= z 3 − ey (2)
 ∂y


 ∂ϕ
= x2 cos z + 3yz 2


 (3)
∂z

Integrando (1), (2) e (3) em relação a x, y e z, respectivamente, temos:



 ϕ(x, y, z) = x2 sen z + A(y, z)

 (4)




ϕ(x, y, z) = yz 3 − ey + B(x, z) (5)




 2 3
 ϕ(x, y, z) = x sen z + yz + C(x, y)
 (6)

Comparando as equações (4), (5) e (6), devemos tomar A(y, z) = yz 3 − ey ,


B(x, z) = x2 sen z e C(x, y) = −ey . Logo,

ϕ(x, y, z) = x2 sen z + y z 3 − ey

→ −

é uma função potencial de F e, portanto F é conservativo. Logo,
Z


F · d~r = ϕ(γ(1)) − ϕ(γ(0))
C

= ϕ(1, 0, 2) − ϕ(0, 0, 1)
 
= 12 sen 2 + 0 − e0 − 0 + 0 − e0

= sen 2 − 1 + 1

= sen 2 .

270
Aula 10

1.
Solução:
Devemos verificar que
I ZZ  
→ → ∂Q ∂P
F · dr = ∂x

∂y
dxdy
C+ D

onde
→   2 2
2 3 2
F = (P, Q) = xy − x y , xy ,
3

(x, y) ∈ R2 e C = ∂D é conforme figura abaixo. Logo, C = C1 ∪ C2 ∪ C3 . Por


propriedade, temos
I Z Z Z
→ → → → → → → →
F · dr = F · dr + F · dr + F · dr .
C+ C1 C2 C3

1
y=x

C3 C2
D

C1 1 x

Z
→ →
Cálculo de F · dr
C1

Parametrizando C1 , temos σ(t) = (t, 0), com 0 ≤ t ≤ 1. Da definição de


integral de linha de campo vetorial, temos:

Z Z 1 → Z 1 Z 1
→ → ′
F · dr = F (σ(t)) · σ (t) dt = (0, 0) · (1, 0) dt = 0 dt = 0 .
C1 0 0 0

271
Z
→ →
Cálculo de F · dr
C2

Temos C2 : σ(t) = (1, t), com 0 ≤ t ≤ 1. Logo,

Z Z 1 →
→ → ′
F · dr = F (σ(t)) · σ (t) dt
C2 0
Z 1  
2 3 
= t − t , t2 · (0, 1) dt
0 3
Z 1
= t2 dt
0
h 3 i1
t
=
3 0

1
= .
3

Z
→ →
Cálculo de F · dr
C3

Temos C3 : σ(t) = (1 − t, 1 − t), com 0 ≤ t ≤ 1. Logo,

Z Z 1  
→ → 2 2
F · dr = (1 − t)4 − (1 − t)3 , (1 − t)4 · (−1, −1) dt
C3 0 3 3
Z 1  
5 2
= − (1 − t)4 − (1 − t)3 dt
0 3 3
h i1
5 (1 − t)5 2 (1 − t)4
= · − ·
3 5 3 4 0

1 1
= − +
3 6
1
= − .
6

Portanto,

I
→ → 1 1 1
F · dr = 0 + 3 − 6 = 6 (1)
C+

272
Por outro lado,
ZZ   ZZ h i
∂Q ∂P 2 
− dxdy = 2xy 2 − 3xy 2 − x2 dxdy
D ∂x ∂y D 3
ZZ  
2
= 2xy − 2xy + x2
2 2
dxdy
D 3
ZZ
2
= x2 dxdy
3 D
Z 1Z x
2
= x2 dydx
3 0 0
Z 1
2
= x3 dx
3 0
h i1
2 x4
=
3 4 0

1
= .
6

ou ZZ  
∂Q ∂P 1
− dxdy = (2)
D ∂x ∂y 6

De (1) e (2) vemos que o teorema está verificado.


2.
Solução:
A região D, limitada por C está ilustrada na figura a seguir.

x = y4 + 1

D C = ∂D

1 2 x

−1

273
→  y 
e
Como F = (P, Q) = , ey ln x + 2x é de classe C 1 no conjunto aberto
 x

U = (x, y) ∈ R2 | x > 0 contendo D e C = ∂D está orientada positivamente,
então podemos aplicar o teorema de Green. Temos, então que:
I ZZ  
→ → ∂Q ∂P
F · d r = − dxdy
C+ ∂x
D ∂y
ZZ  
ey ey
= +2− dxdy
D x x
ZZ
= 2 dxdy .
D

Descrevendo D como tipo II, temos:



D = (x, y) ∈ R2 | −1 ≤ y ≤ 1 , y 4 + 1 ≤ x ≤ 2 .

Então: I Z Z
→ 1 2

F · dr = 2 dxdy
C+ −1 y 4 +1
Z 1 
= 2 1 − y 4 dy
−1

y5 1
h i
= 2 y−
5 −1
 
2
= 2 2−
5
16
= .
5

3.
Solução:
O esboço da região D está representado na figura que se segue.

2
D

−2 −1 2 x

274
Façamos a interseção das curvas y = x2 e y = x + 2:
(
y = x2
⇒ x2 = x + 2 ⇒ x2 − x − 2 = 0 ⇒ x = −1 ou x = 2 .
y =x+2


→ p 
Como F = (P, Q) = y − x + arctg x , 2x − y + 1 + y 2 é de classe C 1
em R2 e ∂D = C está orientada positivamente, podemos aplicar o Teorema de
Green. Como
∂Q ∂P
− = 2 − 1 = 1,
∂x ∂y
então
I ZZ   ZZ
∂Q ∂P
I= P dx + Q dy = − dxdy = dxdy
C+ D ∂x ∂y D

onde D pode ser descrita como tipo I:


(
−1 ≤ x ≤ 2
D:
x2 ≤ y ≤ x + 2 .

Então: Z Z Z
2 x+2 2 
I= dydx = x + 2 − x2 dx
−1 x2 −1
h i2
x2 x3
= + 2x −
2 3 −1
   
8 1 1
= 2+4− − −2+
3 2 3
9
= .
2

4.
Solução:
O esboço de D está representado na figura que se segue.

275
y

3
1111111111111111111111111
0000000000000000000000000
0000000000000000000000000
1111111111111111111111111
0000000000000000000000000
1111111111111111111111111
2
0000000000000000000000000
1111111111111111111111111
0000000000000000000000000
1111111111111111111111111
D
0000000000000000000000000
1111111111111111111111111
0000000000000000000000000
1111111111111111111111111
−5 1111111111111111111111111
0000000000000000000000000
2 5 x
0000000000000000000000000
1111111111111111111111111
0000000000000000000000000
1111111111111111111111111
0000000000000000000000000
1111111111111111111111111
−2
0000000000000000000000000
1111111111111111111111111
0000000000000000000000000
1111111111111111111111111
−3


→ 2 
Como F = (P, Q) = 2xy + ex , x2 + 2x + cos y 2 é um campo de classe C 1
em R2 e ∂D está orientada positivamente, podemos aplicar o Teorema de Green.
Tem-se
∂Q
= 2x + 2
∂x
∂P
= 2x
∂y
donde
∂Q ∂P
− = 2.
∂x ∂y
Então, pelo Teorema de Green, tem-se:
ZZ  
∂Q ∂P
I = − dxdy
D ∂x ∂y
ZZ
= 2 dxdy
D

= 2A(D) = 2 · (área da elipse - área do disco)



= 2 πab − πr2

= 2 π · 5 · 3 − π · 22

= 2 15π − 4π

= 22π .

276
5.
Solução:
A curva C está ilustrada na figura que se segue.
y

(0, 2)

(0, 0)
C1
(4, 0) x

Não é fácil calcular a integral pela definição. Então procuremos uma alternativa
para resolvê-la. Como o teorema de Green só se aplica em curvas fechadas,
consideremos a curva fechada C = C ∪ C1 , onde C1 é o segmento de reta que
liga (0, 0) a (4, 0) parametrizada por σ(t) = (t, 0), com 0 ≤ t ≤ 4.
Seja D e região limitada por C.
y

D C

C1 4 x

Do teorema de Green, temos:


I Z Z
→ → → → → →
+
F · dr = F · dr + F · dr
C C C1
ZZ  
∂Q ∂P
= − dxdy
D ∂x ∂y
ZZ  
1 x
= + dxdy
D 1+x 1+x
ZZ
1+x
= dxdy
D 1+x
ZZ
= dxdy
D

= A(D)

1
= ×4×2
2

= 4.
277
Mas Z Z
→ 4 →
→ ′
F · dr = F (σ(t)) · σ (t) dt
C1 0
Z 4
= (0 , ln(1 + t)) · (1, 0) dt
0
Z 4
= 0 dt = 0 .
0

Logo: Z
→ →
F · dr = 4 .
C

6.
Solução:

a) Observe que F é de classe C 1 no aberto U = R2 − {(0, 0)}. Como C1 envolve
(0, 1), vemos que a região limitada por C1 não está contida um U , conforme a
figura a seguir.

(0, 2)

(0, 1)

Portanto, o teorema de Green não se aplica. Então parametrizando C1 , temos


que σ(t) = (cos t, 1 + sen t), com 0 ≤ t ≤ 2π, donde σ ′ (t) = (− sen t, cos t).
Então:
I Z 2π
→ → →

F · dr = F (σ(t)) · σ (t) dt
C1+ 0
Z 2π  
1 + sen t − 1 − cos t
= − (1 + sen t), · (− sen t, cos t) dt
0 1 1
Z 2π 
= sen t − cos2 t dt
0
h  i
1 sen 2t 2π
= − cos t − t+
2 2 0

= −π .

278
b) Como C2 também envolve (0, 1) não podemos aplicar o teorema de Green.
Calcular a integral por definição é uma tarefa difı́cil, portanto usemos o seguinte
procedimento: isolamos o ponto (0, 1) com a curva C1 do item (a) e consi-
deramos a região D que não contém (0, 1), limitada por C1 e C2 . Orientamos
positivamente ∂D = C1 ∪C2 (C2 no sentido anti-horário e C1 no sentido horário)
e, finalmente, usamos o teorema de Green.
y
4

2
C2
1 C1
D

4 x

Temos então
I I ZZ  
→ → → → ∂Q ∂P
F · dr + F · dr = − dxdy
C2+ C1− D ∂x ∂y
ZZ  
x2 − (y − 1)2 x2 − (y − 1)2
= 2 − 2 + 1 dxdy
D (x2 + (y − 1)2 ) (x2 + (y − 1)2 )
ZZ
= dxdy
D

= A(D)

= 16π − π

= 15π

ou I I
→ → → →
F · dr − F · d r = 15π .
C2+ C1+

Do item (a), temos que


I
→ →
F · d r −(−π) = 15π
C2+

donde I
→ →
F · d r = 14π .
C2+

279
7.
Solução:
a) Observe que calcular a integral por definição é complicado. Por outro lado,
tem-se:
∂Q 2xy ∂P
=− 2 = .
∂x 2
x +y 2 ∂y


Porém, o dom F = R2 − {(0, 0)} não é simplesmente conexo. Logo, não pode-
mos usar o teorema das equivalências. Então, tentemos encontrar uma função
ϕ(x, y) tal que 
∂ϕ x
 ∂x = x2 + y 2 (1)




∇ϕ = F ⇔
 ∂ϕ = y


2 2
(2)
∂y x +y

em R2 − {(0, 0)}. Integrando (1) em relação a x, e integrando (2) em relação a


y, obtém-se: 
 ∂ϕ 1 2 2

 ∂x = 2 ln x + y + A(y)

 ∂ϕ = 1 ln x2 + y 2 + B(x)

 
∂y 2

Devemos tomar A(y) = 0 e B(x) = 0. Como o domı́nio de ϕ é também


R2 − {(0, 0)}, então concluı́mos que

1 
ϕ(x, y) = ln x2 + y 2
2


é uma função potencial de F . Então,
Z

→ −
F · d→
 
r = ϕ γ(π) − ϕ γ(0)
C

= ϕ eπ , 0 − ϕ(1, 0)

1 1
= ln e2π − ln 1
2 2

= ln e
2

= π.


→ 
b) Seja F = (P, Q) = 7x6 y, x7 . Então,

∂Q ∂P
= 7x6 = .
∂x ∂y

280


Como dom F = R2 é simplesmente conexo então, do teorema das equivalências,


F é conservativo com uma função potencial dada por ϕ(x, y) = x7 y (por
inspeção). Logo:
Z
 
7x6 y dx + x7 dy = ϕ γ(1) − ϕ γ(0)
C

= ϕ(1, 1) − ϕ(0, e−1 )

= 1−0

= 1.

8.
Solução:

→ 
Pondo F (x, y) = 2y/x3 , −1/x2 , vemos que o seu domı́nio é D = R2 − eixo y.


Vemos também que ∂Q/∂x = −(−2x)/x4 = 2/x3 = ∂P/∂y, donde rot F =


0 . Mas D não é um conjunto simplesmente conexo. Logo, não podemos usar
o teorema das quatro equivalências. Então, tentemos encontrar uma função


potencial de F . Devemos ter
∂ϕ 2y
= 3 (1)
∂x x
∂ϕ 1
=− 2 (2)
∂y x
Integrando (1) e (2) em relação a x e y respectivamente, temos

y
ϕ(x, y) = − + f (y)
x2
y
ϕ(x, y) = − + g(x)
x2

Tomando f (y) = 0 e g(x) = 0, temos que ϕ(x, y) = −y/x2 , para todo



→ −

(x, y) ∈ D é uma função potencial de F . Logo, F é conservativo e, portanto,
a integral independe do caminho. Temos,
4 −(−2) 4 22
I = ϕ(3, 4) − ϕ(1, −2) = − − =− −2=− .
9 1 9 9

9.
Solução:

→ 
Pondo F (x, y) = 3 + 2xy, x2 − 3y 2 , vemos que o seu domı́nio é o R2 , que
é um conjunto simplesmente conexo. Vemos que ∂Q/∂x = 2x = ∂P/∂y,

281

→ −

donde rot F = 0 . Logo, pelo teorema das quatro equivalências, segue que a
integral é independente do caminho. Também do teorema das equivalências


temos que F é um campo conservativo com uma função potencial
ϕ(x, y) = 3x + x2 y − y 3 (Verifique!) Logo,

I = ϕ (σ(π)) − ϕ (σ(0)) = ϕ(0, −eπ ) − ϕ(0, 1) = −e3π − (−1) = 1 − e3π .

10.
Solução:
Z

→ −
Sabemos que o trabalho é dado por W = F · d→
r . Mas é impossı́vel calcular
C
diretamente a integral pois não conhecemos a equação de C. Como nós temos
∂Q ∂P −

que − = 2 − 3xy 2 6= 0, então F não é conservativo. Assim, só nos resta
∂x ∂y
aplicar o Teorema de Green. Para isso, devemos fechar a curva por um segmento
de reta sobre o eixo x, de (−4, 0) a (4, 0).
y

C
D

(−4, 0) (4, 0) x
C1


Seja D a região limitada por C = C ∪ C1 . Como F é de classe C 1 em R2 e C é
a fronteira de D e está contida em R2 e está orientada no sentido anti-horário,
podemos aplicar o Teorema de Green. Então, temos:
I I  2 

→ − → x 3

F ·dr = + xy dx + 2x + arctg y dy
C
+
C
+
|4 {z } | {z
Q
}
P
ZZ  
∂Q ∂P
= − dxdy
D ∂x ∂y
ZZ

= 2 − 3xy 2 dxdy
D
ZZ ZZ
= 2 dxdy − 3xy 2 dxdy .
D D

Como f (x, y) = 3xy 2 é uma função ı́mpar na variável x e D tem simetria em


relação ao eixo y, então ZZ
3xy 2 dxdy = 0 .
D

282
Assim, Z Z

→ − −
→ −
F · d→
r + F · d→
r = 2 A(D) = 2 · 16 = 32 .
C C1

Z

→ −
Cálculo de F · d→
r
C1

Temos C1 : y = 0, com −4 ≤ x ≤ 4, donde dy = 0. Então


Z Z  2 

→ − → x
F ·dr = + xy 3 dx + (2x + arctg y) dy
C1 C1 4
Z
x2
= dx
C1 4
Z 4
x2
= dx
−4 4
h i4
x3
=
12 −4

2 · 43
=
12
32
= .
3

Logo, Z

→ − 32 64
W = F · d→
r = 32 − = .
C 3 3

Aula 11

1.
Solução:
a) A superfı́cie S está ilustrada na figura a seguir.

z
2

S
1

π/3 3
−2 2 y
2

x
283
Usando φ e θ como parâmetros, temos

S : ϕ(φ, θ) = (2 sen φ cos θ, 2 sen φ sen θ, 2 cos φ)



 0 ≤ φ ≤ π/3
com . Também podemos definir S usando as coordenadas retan-
 0 ≤ θ ≤ 2π
gulares x e y. Temos
 p 
S : ϕ(x, y) = x, y, 4 − x2 − y 2

com (x, y) ∈ D : x2 + y 2 ≤ 3.
Uma outra forma de definir S é usando as coordenadas r e θ. Temos:
 √ 
S : ϕ(r, θ) = r cos θ, r sen θ, 4 − r2

 0 ≤ r ≤ √3
com .
 0 ≤ θ ≤ 2π

b) Encontremos a interseção das duas superfı́cies:


 p
 z = 3(x2 + y 2 )
⇒ x2 + y 2 + 3(x2 + y 2 ) = 1
 x2 + y 2 + z 2 = 1
1
⇒ x2 + y 2 =
4

3
⇒ z= .
2

Elas se interceptam segundo uma circunferência contida no plano horizontal


√  √ 
z = 3/2 , de centro 0, 0, 3/2 e raio 1/2.


3/2

−1/2 1/2
1 y
1
x
284
Usando as coordenadas x e y para definir S, temos:
 p 
ϕ(x, y) = x, y, 3(x2 + y 2 )

com (x, y) ∈ D : x2 + y 2 ≤ 1/4. Outra parametrização:



S : ϕ(r, θ) = (r cos θ, r sen θ, 3 r)

 0 ≤ r ≤ 1/2
com .
 0 ≤ θ ≤ 2π

c) O esboço de S pode ser visto na figura que se segue.

1 2
1 y
2
x
Uma parametrização é dada por

S : ϕ(x, y) = (x, y, 2 − x − y)

com (x, y) ∈ D : x2 + y 2 ≤ 1.
Outra parametrização:

S : ϕ(r, θ) = (r cos θ, r sen θ, 2 − r cos θ − r sen θ)



 0≤r≤1
com .
 0 ≤ θ ≤ 2π

d) O esboço de S está na figura a seguir.

285
z

S
2

−8 x
2
2
4

Adotando θ e z como parâmetros, definimos S por

ϕ(θ, z) = (2 cos θ, 2 sen θ, z)

com 

0 ≤ θ ≤ 2π
cos θ
 0≤z ≤2+ − sen θ
2
e) A superfı́cie S está ilustrada na figura a seguir.
z

S
y
2
1

x
Seja (x, y, z) ∈ S. Então x e y satisfazem x2 + y 2 = 2y ou x2 + (y − 1)2 = 1.
Logo, 
 x = cos t
 y = 1 + sen t

com t ∈ [0, 2π].


286
Adotando t e z como parâmetros, temos a seguinte parametrização para S:

ϕ(t, z) = (cos t, 1 + sen t, z)

com 
 0 ≤ t ≤ 2π
 0 ≤ z ≤ 2(1 + sen t)

2.
Solução:
a) As equações paramétricas de S são:

 x = v cos u

y = v sen u

 z = 1 − v2

com 0 ≤ u ≤ 2π e v ≥ 0. Eliminando os parâmetros u e v, temos que


x2 + y 2 = v 2 = 1 − z ou z = 1 − x2 − y 2 (parabolóide circular).
b) Um vetor normal de S em ϕ(0, 1) = (1, 0, 0) é:


→ ∂ϕ ∂ϕ
N (0, 1) = (0, 1) × (0, 1)
∂u ∂v


= (−v sen u, v cos u, 0) × (cos u, sen u, −2v)
(0,1)

= (0, 1, 0) × (1, 0, −2)


− →
→ − → −
i j k

= 0 1 0

1 0 −2

= (−2, 0, −1)

Equação do plano tangente a S em ϕ(0, 1) = (1, 0, 0)




Da fórmula [(x, y, z) − ϕ(0, 1)] · N (0, 1) = 0 temos:

[(x, y, z) − (1, 0, 0)] · (−2, 0, −1) = 0 ⇒ (x − 1, y, z) · (−2, 0, −1) = 0

⇒ −2(x − 1) − z = 0

⇒ 2x + z − 2 = 0 .

287
Equação da reta normal a S em ϕ(0, 1) = (1, 0, 0)


Da fórmula (x, y, z) − ϕ(0, 1) = λ N (0, 1), com λ ∈ R temos:
(x, y, z) − (1, 0, 0) = λ(−2, 0, −1) ,
com λ ∈ R, que é a equação vetorial da reta normal ou

 x = 1 − 2λ

y=0

 z = −λ

com λ ∈ R, que são equações paramétricas da reta normal.


3.
Solução:
a) Temos 
 x=u

S: y=v

 z = 1 − v2

com (u, v) ∈ D : u + v ≤ 1, u ≥ 0 e v ≥ 0 donde, eliminando os parâmetros,


temos que S : z = 1 − y 2 com (x, y) ∈ Dxy : x + y ≤ 1, x ≥ 0 e y ≥ 0. Logo,
o esboço de S está representado na figura que se segue.
z

1 1
x y
D

b) Temos
∂ϕ ∂ϕ
ϕ (1/2, 1/4) = (1/2, 1/4, 15/16) , (u, v) = (1, 0, 0), (u, v) = (0, 1, −2v)
∂u ∂v
donde,

→ −
→ −

i j k
∂ϕ ∂ϕ
× = 1 0 0 = (0, 2v, 1) .
∂u ∂v
0 1 −2v

288
∂ϕ ∂ϕ
Logo, × (1/2, 1/4) = (0, 1/2, 1) é um vetor normal a S em ϕ (1/2, 1/4).
∂u ∂v
Portanto, uma equação do plano tangente a S em ϕ (1/2, 1/4) é dada por
h  i    
1 1 ∂ϕ 1 1 ∂ϕ 1 1
(x, y, z) − ϕ , · , × , =0
2 4 ∂u 2 4 ∂v 2 4
   
ou x − 21 , y − 14 , z − 15
16
· 0, 1
2
, 1 = 0 ou, y + 2z = 17
8
.
c) Temos
ZZ ZZ √
∂ϕ ∂ϕ
A(S) = × dudv = 1 + 4v 2 dudv
D ∂u ∂v D

u=1−v

1 u

Enquadrando D como tipo II, temos


(
0≤v≤1
D: .
0≤u≤1−v

Logo,
Z 1Z 1−v √ Z 1 √
A(S) = 1+ 4v 2 dudv = (1 − v) 1 + 4v 2 dv
0 0 0
Z 1 √ 1 √ Z
= 1 + 4v 2 dv − v 1 + 4v 2 dv .
|0 {z } | 0
{z }
I1 I2

Cálculo de I1

Fazendo 2v = tg θ, temos dv = 12 sec2 θ dθ. Para


( (
v=0 θ=0 .
temos
v=1 θ = arctg 2
Então
Z arctg 2 p Z arctg 2
1 1
I1 = 1 + tg θ · sec2 θ dθ =
2
sec3 θ dθ .
0 2 2 0

289
Do Cálculo II, temos
Z
1 1
sec3 θ dθ = sec θ tg θ + ln(sec θ + tg θ) + C (Verifique!)
2 2

Logo, h iarctg 2
1 1 1
I1 = sec θ tg θ + ln(sec θ + tg θ) .
2 2 2 0

Fazendo u = arctg 2 temos tg u = 2. Então sec2 u = 1 + tg2 u = 5, donde


√ √
sec u = 5 ou sec(arctg 2) = 5 . Então,
√  √ 
5 1
I1 = + ln 2 + 5 .
2 4

Cálculo de I2

Temos que:
Z 1 √ Z 1 1/2
I2 = v 1 + 4v 2 dv = v 1 + 4v 2 dv
0 0


Como d 1 + 4v 2 = 8v dv então
Z 1
1 1/2 
I2 = 1 + 4v 2 d 1 + 4v 2
8 0
h  i1
1 2 2 3/2
= · 1 + 4v
8 3 0
 √ 
1
= 5 5−1 .
12

Assim,
A(S) = I1 − I2
√  √  √
5 1 5 5 1
= + ln 2 + 5 − +
2 4 12 12
√  √ 
5 1 1
= + ln 2 + 5 + u.a.
12 4 12

290
4.
Solução:
a) Parametrizando a curva C contida no plano xz, temos C : x(t) = a + b cos t,
y(t) = 0 e z(t) = b sen t, com 0 ≤ t ≤ 2π. Sabemos que uma parametrização
de S é dada por

ϕ(t, θ) = (x(t) cos θ, x(t) sen θ, z(t))

com (t, θ) ∈ D : 0 ≤ t ≤ 2π e 0 ≤ θ ≤ 2π ou

ϕ(t, θ) = ((a + b cos t) cos θ, (a + b cos t) sen θ, b sen t)

com (t, θ) ∈ D : 0 ≤ t ≤ 2π e 0 ≤ θ ≤ 2π.


b) Temos

ϕt = (−b sen t cos θ, −b sen t sen θ, b cos t)

ϕθ = (−(a + b cos t) sen θ, (a + b cos t) cos θ, 0)


→ −
→ −


i j k


ϕt × ϕθ = −b sen t cos θ −b sen t sen θ b cos t =


−(a + b cos t) sen θ (a + b cos t) cos θ 0

= (−b(a + b cos t) cos θ cos t, −b(a + b cos t) sen θ cos t, −b(a + b cos t) sen t)

donde


kϕt × ϕθ k = b(a + b cos t) cos2 θ cos2 t + sen2 θ cos2 t + sen2 t
p
= b(a + b cos t) cos2 t (cos2 θ + sen2 θ) + sen2 t

= b(a + b cos t) .

ZZ
Como A(S) = kϕt × ϕθ k dtdθ, então:
D

291
ZZ
A(S) = b(a + b cos t) dtdθ
D
Z 2πZ 2π
= b (a + b cos t) dθdt
0 0
Z 2π
= 2πb (a + b cos t) dt
0

 2π
= 2πb at + b sen t 0

= 2πb(2πa)

= 4π 2 ab u.a.

5.
Solução:
O esboço da superfı́cie S pode ser visto na figura a seguir.
z

2 y
2
D
x

Definimos S da seguinte maneira:

S : z = 2(x2 + y 2 ) = f (x, y) ,
com (x, y) ∈ D : x2 + y 2 ≤ 4. Como
s  2  2
ZZ
∂f ∂f
A(S) = 1+ + dx dy
D ∂x ∂y
então
ZZ p ZZ p
A(S) = 2 2
1 + (4x) + (4y) dx dy = 1 + 16x2 + 16y 2 dx dy .
D D

292
Em coordenadas polares, temos
ZZ √
A(S) = 1 + 16r2 r dr dθ
Drθ
Z 2Z 2π 1/2
= (1 + 16r2 ) r dθdr
0 0
Z 2
2π 1/2 
= (1 + 16r2 ) d 1 + 16r2
32 0
h i2
π 2 3/2
= · (1 + 16r2 )
16 3 0

π √
= (65 65 − 1) u.a.
24

6.
Solução:
Fazendo a interseção das superfı́cies temos:
(
x2 + y 2 + z 2 = 2
2 2
⇔ z 2 + z − 2 = 0 ⇔ z = 1 pois z ≥ 0 .
z =x +y

Logo, a interseção é uma circunferência de raio 1, contida no plano z = 1.


z

2 S

φ

2
1 y
1 π
tg φ = =1⇒φ= .
D 1 4

Parametrizando S, adotando φ e θ (coordenadas esféricas) como parâmetros:


 √
 x = √ 2 sen φ cos θ

y = 2 sen φ sen θ
 z = √2 cos φ

293
com
(
0 ≤ θ ≤ 2π
(φ, θ) ∈ D :
0 ≤ φ ≤ π/4

Da aula 11, temos que dS = ρ2 sen φ dφdθ = 2 sen φ dφdθ. Portanto:

ZZ ZZ Z π Z 2π
A(S) = dS = 2 sen φ dφdθ = 2 sen φ dθdφ
S S 0 0
Z π
= 4π sen φ dφ
0

 π
= 4π − cos φ 0

= 8π u.a.

7.
Solução:
O esboço da superfı́cie S está representado na figura que se segue.

1
1
2
2
y
x
D

294
p
Podemos definir S por S : z = x2 + y 2 = f (x, y), com (x, y) ∈ D onde D é
dada pela figura que se segue

2 E

1 B D

A C
O 1 2 x

Portanto:
ZZ q
A(S) = 1 + (fx )2 + (fy )2 dxdy
D
ZZ r  2  2
x y
= 1+ + dxdy
D x2 + y 2 x2 + y 2
ZZ r
x2 + y 2
= 1 + 2 2 dxdy
D x +y
ZZ √
= 2 dxdy
D

= 2 A(D)

= 2 (área ∆OCE − área ∆OAB)
√ 1 1

= 2 ·2·2− ·1·1
2 2

3 2
= u.a.
2

8.
Solução:
a) Adotando x e y como parâmetros, temos:
 
x2 + y 2
S : ϕ(x, y) = x, y, ,
2 }
| {z
f (x,y)

com (x, y) ∈ D : x2 + y 2 ≤ 2k.

295
Observe a figura a seguir.

k
S

D √
2k y

b) Como
ZZ r  2  2
∂f ∂f
A(S) = 1+ + dxdy ,
D ∂x ∂y

então:
ZZ p
A(S) = 1 + x2 + y 2 dxdy
D
ZZ √
= 1 + r2 r drdθ (coordenadas polares)
Drθ
Z 2πZ √
2k
1 r/2 
= 1 + r2 d 1 + r2 dθ
2 0 0
Z 2π h i√2k
1 2 
2 3/2
= 1+r dθ
2 0 3 0
Z 2π
1 
= (1 + 2k)3/2 − 1 dθ
3 0

2π  
= (1 + 2k)3/2 − 1 .
3

14π
Como A(S) = , temos:
3

296
2π   14π
(1 + 2k)3/2 − 1 = ⇒ (1 + 2k)3/2 − 1 = 7
3 3

⇒ (1 + 2k)3/2 = 8 = 23

⇒ (1 + 2k)1/2 = 2

⇒ 1 + 2k = 4

⇒ 2k = 3

3
⇒ k= .
2
9.
Solução:
A superfı́cie S está ilustrada na figura que se segue.
z
4
C
S

2 y
2

Uma parametrização de S é dada por

S : ϕ(t, z) = (2 cos t, 2 sen t, z)

com (
0 ≤ t ≤ 2π
(t, z) ∈ D : .
0 ≤ z ≤ 4 − 4cos2 t = 4sen2 t
Temos

→ −
→ →


i j k
∂ϕ ∂ϕ
× = −2 sen t 2 cos t 0 = (2 cos t, 2 sen t, 0)
∂t ∂z
0 0 1
donde
∂ϕ ∂ϕ
× = 2.
∂t ∂z

297
Como ZZ
∂ϕ ∂ϕ
A(S) = × dt dz
D ∂t ∂z
então ZZ Z 2πZ 4sen2 t
A(S) = 2 dt dz = 2 dz dt
D 0 0
Z 2π
= 8 sen2 t dt
0
h i2π
1 sen 2t
= 8· t−
2 2 0

= 8π u.a.

10.
Solução:
O esboço de S é:
z z

S S1

1 2 y 1 2 y

x x

Por simetria, tem-se que A(S) = 2A(S1 ), onde S1 é dada por:


p
S1 : x2 + (y − 1)2 = 1 , x ≥ 0 , 0 ≤ z ≤ x2 + y 2 .

Se (x, y, z) ∈ S1 então x e y satisfazem x2 + (y − 1)2 = 1, x ≥ 0. Logo:


(
x = cos t
y = 1 + sen t .

Como x ≥ 0 então −π/2 ≤ t ≤ π/2. Portanto, uma parametrização para S1


é dada por
ϕ(t, z) = (cos t, 1 + sen t, z)

298
com
(
−π/2 ≤ t ≤ π/2
(t, z) ∈ D : p p .
0 ≤ z ≤ cos2 t + (1 + sen t)2 = 2(1 + sen t)

Tem-se ϕt = (− sen t, cos t, 0) e ϕz = (0, 0, 1) donde



− → −
→ − →

i j k
ϕt × ϕz = − sen t cos t 0 = (cos t, sen t, 0)

0 0 1

e kϕt × ϕz k = 1.
Como ZZ
A(S1 ) = kϕt × ϕz k dtdz
D
então
ZZ Z π/2 Z √2(1+sen t) Z π/2 p
A(S1 ) = dtdz = dzdt = 2(1 + sen t) dt
D −π/2 0 −π/2

√ Z π/2 √
= 2 1 + sen t dt .
−π/2

Da Trigonometria, tem-se:

t t
 1 = cos2+ sen2
2 2
 sen t = 2 sen t cos t
2 2
donde
 
t t t t t t 2
1 + sen t = cos2 + 2 sen cos + sen2 = cos + sen .
2 2 2 2 2 2
Logo:

t t

t t
1 + sen t = cos + sen = cos + sen ,
2 2 2 2
 
pois t ∈ − π/2 , π/2 . Então:
√ Z π/2  t t
 √ h t
i
t π/2
A(S1 ) = 2 cos + sen dt = 2 2 sen − 2 cos
−π/2 2 2 2 2 −π/2
 π/2 

t π/2
= 2 2 sen − cos t
2 −π/2 −π/2


√ 
2
= 2 2 2· −0
2

= 4 u.a.
Portanto,
A(S) = 2 · 4 = 8 u.a.
299
Aula 12

1.
Solução:
Temos: −
→ −
→ −

i j k
∂ϕ ∂ϕ
× = 1 0 2u = (−2u, 0, 1) .
∂u ∂v
0 1 0
∂ϕ ∂ϕ √

Donde × = 1 + 4u2 . Temos:
∂u ∂v

∂ϕ ∂ϕ √
dS = × dudv = 1 + 4u2 dudv .
∂u ∂v

Então:
ZZ ZZ
 p
z − x2 + xy 2 − 1 dS = u2 + 1 − u2 + uv 2 − 1 1 + 4u2 dudv
S D
ZZ p
= uv 2 1 + 4u2 dudv
D
Z 1Z 2 1/2
= u 1 + 4u2 v 2 dvdu
0 0
Z 1  2
2 1/2 v3
= u(1 + 4u ) du
0 3 0
Z 1
8 1 1/2 
= · 1 + 4u2 d 1 + 4u2
3 8 0

1 2
h 3/2 i1
= · 1 + 4u2
3 3 0

2
 √ 
= 5 5−1 .
9

2.
Solução:
A superfı́cie S e a sua projeção sobre o plano xy estão ilustradas nas figuras que
se seguem.

300
z y
1
1

S D
1 y 1 x
D
1
x

Definimos S por S : z = 1 − x − y = f (x, y), onde


(
0≤x≤1
(x, y) ∈ D :
0≤y ≤1−x
Temos que:
r  2  2
∂z ∂z p √
dS = 1 + + dxdy = 1 + (−1)2 + (−1)2 dxdy = 3 dxdy .
∂x ∂y
Então: ZZ ZZ √
x dS = x 3 dxdy
S D

√ Z 1Z 1−x
= 3 x dydx
0 0

√ Z 1
= 3 x(1 − x) dx
0

√ Z 1 
= 3 x − x2 dx
0
√ h x2 x3 i1
= 3 −
2 3 0

3
= .
6
3.
Solução:
Uma parametrização da esfera é

ϕ(φ, θ) = (a sen φ cos θ, a sen φ sen θ, a cos φ)

com (
0≤φ≤π
(φ, θ) ∈ D :
0 ≤ θ ≤ 2π .

301
Lembramos que (ver Aula 11, exemplo 1) kϕφ × ϕθ k = a2 sen φ. Como
dS = kϕφ × ϕθ k dφdθ então dS = a2 sen φ dφdθ (memorize este resultado).
Logo
ZZ ZZ
2 2
 
x + y dS = a2 sen2 φ cos2 θ + a2 sen2 φ sen2 θ a2 sen φ dφdθ
S D| {z }
= a2 sen2 φ
ZZ
4
= a sen3 φ dφdθ
D
Z π Z 2π
4 3
= a sen φ dθdφ
0 0
Z π
4
= 2πa sen3 φ dφ .
0

Da Trigonometria temos:

sen3 φ = sen2 φ · sen φ = 1 − cos2 φ sen φ .

Então:
ZZ Z π
2 2
 4

x +y dS = 2πa 1 − cos2 φ sen φ dφ
S 0
Z π
4

= −2πa 1 − cos2 φ d(cos φ)
0
h iπ
4 cos3 φ
= −2πa cos φ −
3 0
 
2
= −2πa4 −2 +
3
8πa4
= .
3

4.
Solução:
O esboço de S está representado na figura que se segue.

302
z

S
2

D
1 1
2 y
x

é descrita por S : z = f (x, y) = 2−x, com (x, y) ∈ D : x2 +y 2 ≤ 1.


A superfı́cie Sq
p
Como dS = 1 + (fx )2 + (fy )2 dxdy, então dS = 1 + (−1)2 + 02 dxdy =

= 2 dxdy.
Temos
ZZ ZZ ZZ √ √ ZZ 2
2 2
M= δ(x, y, z) dS = y dS = y 2 dxdy = 2 y dxdy .
S S D D

Usando coordenadas polares, temos


ZZ ZZ ZZ
2 2 2
y dxdy = (r sen θ)r drdθ = r3 sen2 θ drdθ
D Drθ Drθ
Z 2π Z 1
2
= sen θ r3 drdθ
0 0
Z 2π
1
= sen2 θ dθ
4 0
h i2π
1 1 sen 2θ
= · θ−
4 2 2 0

π
= .
4


Logo, M = u.m.
4

303
5.
Solução:
Inicialmente, encontremos a interseção:
(
z 2 = x2 + y 2
2 2
⇔ z 2 − 4z + 3 = 0 ⇔ z = 1 ou z = 3
4z = x + y + 3
Logo, o cone intercepta o parabolóide no plano z = 1, segundo uma circun-
ferência de raio 1 e também no plano z = 3, segundo uma circunferência de
raio 3. Portanto, a parte do cone que está acima do parabolóide é:
z

S
1

D 1 3 y

A superfı́cie S pode ser dada por


p
S : z = x2 + y 2 , (x, y) ∈ D : 1 ≤ x2 + y 2 ≤ 9 .

Neste caso, q
dS = 1 + (zx )2 + (zy )2 dxdy
onde
x y
zx = p e zy = p 2 2
x2 + y 2 x +y
donde
x2 + y 2
1 + (zx )2 + (zy )2 = 1 + 2 2 = 2 .
x +y
p √
Portanto, no caso do cone z = x2 + y 2 temos que dS = 2 dxdy. Assim:
ZZ ZZ ZZ p √
f (x, y, z) dS = z dS = x2 + y 2 · 2 dxdy
S S D
√ ZZ p
= 2 x2 + y 2 dxdy .
D

304
Para calcular a integral dupla, usamos coordenadas polares. Temos:
ZZ p ZZ Z 1Z 2π h 3 i1
r 2π
2 2
x + y dxdy = r · r drdθ = r2 dθdr = 2π = .
D Drθ 0 0 3 0 3

Logo, ZZ √
2 2π
f (x, y, z) dS = .
S 3

6.
Solução:
O esboço de S está representado na figura que se segue.
z

C1
S1

1 y
1
S2
C2
x

Tem-se:
ZZ ZZ p
M= ρ(x, y, z) dS = x2 + y 2 dS
S S
ZZ p ZZ p
= 2 2
x + y dS + x2 + y 2 dS .
S1 S2

ZZ p
Cálculo de x2 + y 2 dS
S1

Uma parametrização da curva C1 é



 x(t) = t

y(t) = 0 com 0 ≤ t ≤ 1 .

 z(t) = 1 − t

Logo:

x(t) = t = raio de uma circunferência transversal

z(t) = 1 − t = altura dessa circunferência .

305
Então, uma parametrização de S1 é dada por

ϕ(t, θ) = (t cos θ, t sen θ, 1 − t)

com
(
0≤t≤1
(t, θ) ∈ D :
0 ≤ θ ≤ 2π .

Tem-se ϕt = (cos θ, sen θ, −1) e ϕθ = (−t sen θ, t cos θ, 0) donde




→ −
→ −


i j k
ϕt × ϕθ = cos θ
sen θ −1

−t sen θ t cos θ 0

= t cos θ , t sen θ , t| cos2 θ {z
+ t sen2 θ}
= t

= t(cos θ, sen θ, 1) .

Logo,
√ √
kϕt × ϕθ k = |t| cos2 θ + sen2 θ + 1 = t 2 ,

pois 0 ≤ t ≤ 1 e, portanto:

dS = kϕt × ϕθ k dtdθ = t 2 dtdθ .

Então
ZZ p ZZ √ √
2 2
x + y dS = t2 cos2 θ + t2 sen2 θ t 2 dtdθ
S1 D

√ ZZ 2
= 2 t dtdθ
D

√ Z 1 2Z 2π
= 2 t dθdt
0 0

√ Z 12
= 2 2π t dt
0

√ t
h 3 i1
= 2 2π
3 0

2 2π
= .
3

306
ZZ p
Cálculo de x2 + y 2 dS
S2

A superfı́cie S2 é dada por

S2 : z = f (x, y) = 0, com (x, y) ∈ D : x2 + y 2 ≤ 1 .


q √
Como dS = 1 + (fx )2 + (fy )2 dxdy então dS = 1 + 0 + 0 dxdy
ou dS = dxdy.
Obs.: Se S é uma porção do plano z = 0 ou z = c (c = constante), segue que
dS = dxdy (memorize este resultado).
Logo, ZZ p ZZ p
x2 + y2 dS = x2 + y 2 dxdy .
S2 D

Passando para coordenadas polares, tem-se:





 x = r cos θ

y = r sen θ


 dxdy = rdrdθ
 2
x + y 2 = r2

e Drθ é dado por (


0≤r≤1
Drθ :
0 ≤ θ ≤ 2π .
Logo: ZZ p ZZ
2 2
x + y dS = r · r drdθ
S2 Drθ
ZZ
= r2 drdθ
Drθ
Z 1 Z 2π
2
= r dθdr
0 0
Z 1
= 2π r2 dr
0
h 3 i1
r
= 2π
3 0


= .
3
√ √
2 2π 2π 2π
Assim M = + ou M = (1 + 2) u.m.
3 3 3

307
7.
Solução:
A superfı́cie S pode ser vista na figura a seguir.
z

h C

D h
y

x
p
Temos S : z = x2 + y 2 , com (x, y) ∈ D : x2 + y 2 ≤ h2 , x ≥ 0 e y ≥ 0.

Donde dS = 2 dxdy. Como
ZZ

Iz = x2 + y 2 k dS ,
S

então
ZZ √ ZZ
2 2
√ 
Iz = k x +y 2 dxdy = k 2 x2 + y 2 dxdy .
D D

Passando para coordenadas polares, temos:


√ Z π/2Z h
Iz = k 2 r2 r drdθ
0 0

√ Z π/2Z h
= k 2 r3 drdθ
0 0

√ Z π/2 h 4 ih
r
= k 2 dθ
0 4 0
Z
h4 k √
π/2
= 2 dθ
4 0

h4 k 2π
= .
8

308
Mas
ZZ √ ZZ √ √ √
k 2 πh2 h2 k 2 π
M= k dS = k 2 dxdy = k 2 A(D) = = .
S D 4 4

M h2
Logo, Iz = .
2
8.
Solução:
Consideremos a esfera centrada em (0, 0, 0). Então tem-se S : x2 +y 2 +z 2 = R2 .
Considerando um diâmetro sobre o eixo z, devemos calcular o momento de inércia
em relação ao eixo z. Tem-se
ZZ ZZ
2 2
 
Iz = x + y ρ dS = ρ x2 + y 2 dS .
S S

Sabemos que S é dada por

ϕ(φ, θ) = (R sen φ cos θ, R sen φ sen θ, R cos φ)

com (
0≤φ≤π
(φ, θ) ∈ D :
0 ≤ θ ≤ 2π .
Tem-se x2 + y 2 = R2 sen2 φ e dS = R2 sen φ dφdθ. Então:
ZZ ZZ
2 2
 2
 4
Iz = ρ R sen φ · R sen φ dφdθ = ρR sen3 φ dφdθ
D D
Z π Z 2π
4 3
= ρR sen φ dθdφ
0 0
Z π
4

= 2ρπR sen2 φ · (sen φ) dφ
0
Z π
4

= −2ρπR 1 − cos2 φ d(cos φ)
0
h iπ
cos3 φ
= −2ρπR4 cos φ −
3 0
 
4 2
= −2ρπR −2 +
3
8ρπR4
= .
3
Como S é homogênea, então

M = ρA(S) = ρ(4πR2 ) = 4ρπR2 .


2M R2
Logo Iz = .
3

309
9.
Solução:
O esboço de S está representado na figura que se segue.

R y
R

Uma parametrização de S é dada por

ϕ(t, z) = (R cos t, R sen t, z) com (t, z) ∈ D : 0 ≤ t ≤ 2π , 0 ≤ z ≤ 1 .

Temos ϕt = (−R sen t, R cos t, 0) e ϕz = (0, 0, 1) donde




→ −
→ −


i j k
ϕt × ϕz = −R sen t R cos t 0 = (R cos t, R sen t, 0)

0 0 1

e kϕt × ϕz k = R. Como dS = kϕt × ϕz k dtdz então dS = R dtdz.


Obs.: Daqui por diante, no caso do cilindro x2 + y 2 = R2 , use o fato de
que dS = R dtdz.
O momento de inércia é dado por
ZZ ZZ
2 2
 
Iz = x + y δ(x, y, z) dS = k R2 cos2 t + R2 sen2 t R dtdz
S | {z } D
k
Z 2πZ 1
3
= kR dzdt
0 0

= 2kπR3 .

Como M = kA(S) = k(2πR) · 1 = 2kπR então Iz = M R2 .

310
10.
Solução:
Sem perda de generalidade, podemos considerar a casca cilı́ndrica S conforme
figura a seguir.
z

L/2

a y
a

−L/2

Então, S é dada por:

S : ϕ(t, z) = (a cos t, a sen t, z)

com 
 0 ≤ t ≤ 2π
(t, z) ∈ D :
 −L/2 ≤ z ≤ L/2

Temos dS = a dtdz. Como um diâmetro pelo centro do cilindro é o eixo y,


então o momento de inércia em relação ao eixo y é dado por
ZZ

Iy = x2 + z 2 f (x, y, z) dS
S

onde f (x, y, z) = k. Então:


ZZ

Iy = k a2 cos2 t + z 2 a dtdz
D
Z 2πZ L/2 
= ka a2 cos2 t + z 2 dzdt
0 −L/2
Z 2π h iL/2
z3
= ka a2 cos2 tz + dt
0 3 −L/2
Z 2π
z 3 L/2
h  i
= ka a2 cos2 t z + dt
0 3 −L/2

L2 t 2π
h   i
a2 sen 2t
= kaL t+ +
2 2 12 0
 
a2 L2
= 2kπaL + .
2 12

311
M a2 M L2
Como M = kA(S) então M = k · 2πaL = 2kπaL. Logo, Iy = + .
2 12

Aula 13

1.
Solução:
O esboço de S está representado na figura a seguir.



n
S

1 y
1

x


Como −→
n · k > 0 então − →
n aponta para cima e, portanto, temos que

→ (x, y, z)
n = = (x, y, z) pois a = 1. O fluxo é dado por:
a
ZZ ZZ

→ − →
F · n dS = (x − y − 4, y, z) · (x, y, z) dS
S S
ZZ

= x2 − xy − 4x + y 2 + z 2 dS
S
ZZ

= x2 + y 2 + z 2 −xy − 4x dS
S
| {z }
= 1
ZZ
= (1 − xy − 4x) dS
S
ZZ ZZ
= dS − (xy − 4x) dS
S S
ZZ
= A(S) − (xy − 4x) dS
S
ZZ
1
= · 4π · 12 − (xy − 4x) dS
2 S
ZZ
= 2π − (xy − 4x) dS .
S
312
ZZ
Ora, para calcular (xy − 4x) dS devemos parametrizar S. Então temos que
S
S : ϕ(φ, θ) = (sen φ cos θ, sen φ sen θ, cos φ), com (φ, θ) ∈ D : 0 ≤ φ ≤ π/2
e 0 ≤ θ ≤ 2π. Também temos que dS = a2 sen φ dφdθ = sen φ dφdθ. Logo:
ZZ
(xy − 4x) dS =
S
ZZ

= sen2 φ sen θ cos θ − 4 sen φ cos θ sen φ dφdθ
D
ZZ ZZ
3
= sen φ sen θ cos θ dφdθ − 4 sen2 φ cos θ dφdθ
D D
Z π/2 Z 2π Z π/2 Z 2π
3 2
= sen φ sen θ cos θ dθdφ − 4 sen φ cos θ dθdφ
0 0 0 0
Z π/2 h i2π Z π/2
3 sen2 θ  2π
= sen φ dφ − 4 sen2 φ sen θ 0 dθ
0 2 0 0

= 0.

Portanto: ZZ

→ −
F ·→
n dS = 2π .
S

2.
Solução:
O esboço de S está representado na figura a seguir.

y
1

1
x+y =1



n S y =1−x
1 y
D

1 1 x
x y=0

A superfı́cie pode ser descrita por S : z = 1 − x − y = f (x, y), com


(x, y) ∈ D : 0 ≤ x ≤ 1 e 0 ≤ y ≤ 1 − x. Um vetor normal a S é

313


dado por N = (−fx , −fy , 1) = (1, 1, 1) Como −

n aponta para baixo então

→ (−1, −1, −1)
n = √ .
3
q √
Temos que dS = 1 + (fx )2 + (fy )2 dxdy = 3 dxdy . Então:
ZZ ZZ

→ − →  (−1, −1, −1)
F · n dS = xzey , −xzey , z · √ dS
S S 3
ZZ 
xzey − xzey − z
= √ dS
S 3
ZZ
−z
= √ dS
S 3
ZZ
−(1 − x − y) √
= √ · 3 dxdy
D 3
ZZ
= (−1 + x + y) dxdy
D
Z 1Z 1−x
= (−1 + x + y) dydx
0 0
Z 1h i1−x
y2
= − y + xy + dx
0 2 0

−1
= .
6
3.
Solução:
A superfı́cie S está
z ilustrada na figura a seguir:

3 2

π/6 1
S
1 √
y 3
2 2 φ 1 π
tg φ = √ ⇒ φ =
3 6

Uma parametrização para S é dada por:


S : ϕ(φ, θ) = (2 sen φ cos θ , 2 sen φ sen θ , 2 cos φ)
h i
π 5π
com (φ, θ) ∈ D = , × [0, 2π].
6 6

314
Temos:
a=2
dS = a2 sen φ dφ dθ = 4 sen φ dφ dθ .

Como −

n é exterior a S, então


→ (x, y, z) a=2 (x, y, z)
n = = .
a 2

Então: ZZ ZZ

→ − (x, y, z)
F ·→
n dS = (0, 0, −z) · dS
S S 2
ZZ
= − z 2 dS
S
ZZ
= − 4 cos2 φ · 4 sen φ dφ dθ
D
Z 5π/6Z 2π
= −16 cos2 φ sen φ dθ dφ
π/6 0
Z 5π/6
= 32π cos2 φ d(cos φ)
π/6
h i5π/6
cos3 φ
= 32π
3 π/6
" √   √ 3 #
3
32π 3 3
= − −
3 2 2

32π 3 3
= − ·
3 8

= −4π 3 .

4.
Solução:

A superfı́cie S está ilustrada na fi-


gura ao lado.
z
Temos: 5

S : ϕ(t, z) = (4 cos t , 4 sen t , z)




S n
com
( C
0 ≤ t ≤ π/2
(t, z) ∈ D :
0 ≤ z ≤ 5 − 4 sen t
4 5 y
x 4

315
Além disso, dS = a dt dz = 4 dt dz. Como −

a=4
n aponta para o eixo z, então:

→ (−x , −y , 0) (−x , −y , 0)
n = = .
a 4

Portanto:
ZZ ZZ

→ −  (−x , −y , 0)
F ·→
n dS = − x , −y , 3y 2 z · dS
S S 4
ZZ
1 
= x2 + y 2 dS
4 S| {z }
= 16
ZZ
= 4 dS
S
ZZ
= 4 dS
S
ZZ
= 4 4 dt dz
D
Z π/2Z 5−4 sen t
= 16 dz dt
0 0
Z π/2
= 16 (5 − 4 sen t) dt
0
 π/2
= 16 5t + 4 cos t 0
 5π 
= 16 −4
2

= 40π − 64 .

5.
Solução:
O esboço de S está representado na figura que se segue.



n

S
2

2
3 y
3
x
316
Como S é a porção do plano z = 2 − x que se projeta no plano xy segundo o
disco x2 + y 2 ≤ 9, definimos S por S : z = 2 − x, com (x, y) ∈ D : x2 + y 2 ≤ 9.


Como − →
n · k ≥ 0 então o campo − →n aponta para cima.

→ −

Sabemos que N = (−zx , −zy , 1) = (1, 0, 1). Como a última componente de N

→ (1, 0, 1) √
é positiva então N aponta para cima. Logo − →n = √ e dS = 2 dxdy.
2
Então:
ZZ ZZ

→ −
F ·→

n dS = 2 − x, y(2 − x), 0 · (1, 0, 1) dxdy
S D
ZZ
= (2 − x) dxdy .
D

Passando para coordenadas polares, tem-se:




 x = r cos θ
y = r sen θ

 dxdy = rdrdθ

e Drθ é dado por (


0≤r≤3
Drθ :
0 ≤ θ ≤ 2π .
Então: ZZ ZZ

→ −
F ·→
n dS = (2 − r cos θ)r drdθ
S Drθ
ZZ

= 2r − r2 cos θ drdθ
Drθ
Z 3 Z 2π 
= 2r − r2 cos θ dθdr
0 0
Z 3 2π
= 2rθ − r2 sen θ 0
dr
0
Z 3
= 4πr dr
0

r 2 3
= 4π
2 0

= 18π .

317
6.
Solução:
O esboço de S está representado na figura que se segue.



n1

3 S1

S2



n2

2 2 3


n y
3 3

x
S3

Devemos olhar para S como reunião das superfı́cies S1 , S2 e S3 , isto é,


S = S1 ∪ S2 ∪ S3 . Então:

ZZ ZZ ZZ ZZ

→ − −
→ − −
→ − −
→ −
F ·→
n dS = F ·→
n1 dS + F ·→
n2 dS + F ·→
n3 dS .
S S1 S2 S3

ZZ

→ −
Cálculo de F ·→
n1 dS
S1

Tem-se S1 : z = 3 − x − y, com (x, y) ∈ D : x2 + y 2 ≤ 4 donde temos que



→ (1, 1, 1)
N = (−zx , −zy , 1) = (1, 1, 1). Como −

n1 aponta para cima então −

n1 = √
√ 3
e dS = 3 dxdy. Então:

318
ZZ ZZ

→ −
F ·→
n1 dS = (x, y, 3 − x − y) · (1, 1, 1) dxdy
S1 D
ZZ
= (x + y + 3 − x − y) dxdy
D
ZZ
= 3 dxdy
D

= 3A(D)

= 3 · π · 22

= 12π .

ZZ

→ −
Cálculo de F ·→
n2 dS
S2

Tem-se S2 : x2 + y 2 = 4, com 0 ≤ z ≤ 3 − x − y. Como −



n2 é exterior a S2 ,

→ (x, y, 0)
então n2 = , conforme Aula 11. Então:
2

ZZ ZZ

→ − 1
F ·→
n2 dS = (x, y, z) · (x, y, 0) dS
S2 2 S2
ZZ
1 
= x2 + y 2 dS
2 S2
ZZ
1
= 4 dS
2 S2
ZZ
= 2 dS
S2

onde S2 é parametrizada por S2 : ϕ(t, z) = (2 cos t, 2 sen t, z) com

(
0 ≤ t ≤ 2π
(t, z) ∈ D :
0 ≤ z ≤ 3 − 2 cos t − 2 sen t

319
Sabemos que dS = a dtdz = 2 dtdz, conforme Aula 11. Então:
ZZ ZZ

→ − →
F · n2 dS = 2 2 dtdz
S2 D
Z 2πZ 3−2 cos t−2 sen t
= 4 dzdt
0 0
Z 2π
= 4 (3 − 2 cos t − 2 sen t) dt
0
h i2π
= 4 3t − 2 sen t + 2 cos t
0

= 24π .

ZZ

→ −
Cálculo de F ·→
n3 dS
S3

Tem-se S3 : z = 0, com (x, y) ∈ D : x2 + y 2 ≤ 4. Logo, dS = dxdy. Observe




que −

n3 = − k = (0, 0, −1). Então:
ZZ ZZ ZZ

→ − →
F · n3 dS = (x, y, 0) · (0, 0, −1) dxdy = 0 dxdy = 0 .
S3 D D

Logo ZZ

→ −
F ·→
n dS = 12π + 24π = 36π .
S

7.
Solução:
As figuras a seguir, mostram a curva C e a superfı́cie S.

z z
3 3



n
C S
1 1

y y
1 1
x x

320
Uma parametrização para C é dada por:
 
σ(t) = (1, 0, 1)+t (0, 0, 3)−(1, 0, 1) = (1, 0, 1)+t(−1, 0, 2) = (1−t , 0 , 1+2t) ,

com t ∈ [0, 1]. Logo: 



 x(t) = 1 − t

y(t) = 0


 z(t) = 1 + 2t

com t ∈ [0, 1]. Uma parametrização para S é dada por:


 
S : ϕ(θ, t) = x(t) cos θ , x(t) sen θ , z(t) = (1−t) cos θ , (1−t) sen θ , 1+2t

com θ ∈ [0, 2π] e t ∈ [0, 1].


Um vetor normal a S é dado por:


→ −
→ −

i j k

→ ∂ϕ ∂ϕ
N = × = −(1 − t) sen θ (1 − t) cos θ 0

∂θ ∂t
− cos θ − sen θ 2

= 2(1 − t) cos θ , 2(1 − t) sen θ , 1 − t .

Temos:
φ=


− →
ZZ
= F ·−
n dS
S
ZZ
(1 − t)(1 + 2t) sen θ , −(1 − t)(1 + 2t) cos θ) , (1 − t)2 · 2(1 − t) cos θ , 2(1 − t) sen θ , 1 − t dt
` ´ ` ´
=
D
ZZ
2(1 − t)2 (1 + 2t) sen θ cos θ − 2(1 − t)2 (1 + 2t) sen θ cos θ + (1 − t)3 dt
ˆ ˜
=
D
Z 1 Z 2π
= (1 − t)3 dt
0 0
#1
(1 − t)4
"
= −2π
4 0

1
!
= −2π 0−
4
π
= .
2

321
8.
Solução:
A superfı́cie S está ilustrada na figura a seguir.
z



n
a
x
a
y

Uma parametrização para S é dada por S : ϕ(t, z) = (a cos t, a sen t, z) onde


(x, y, 0)
(t, z) ∈ D : 0 ≤ t ≤ 2π , 0 ≤ z ≤ a2 cos2 t. Temos que − →n =
a
e ds = a dtdz. Como ZZ

→ −
Φ= F ·→n ds
S
então:
ZZ ZZ
(x, y, 0) 1 
Φ= (x, y, z) · ds = x2 + y 2 ds
S a a S
ZZ
1
= a2 ds
a S
ZZ
= a a dtdz
D
ZZ
2
= a dtdz
D
Z 2πZ a2 cos2 t
2
= a dzdt
0 0
Z 2π
2
= a a2 cos2 t dt
0
Z 2π
= a4 cos2 t dt
0
h i2π
1 sen 2t
= a4 · t+
2 2 0

= πa4 .

322
9.
Solução:
O esboço de S = S1 ∪ S2 está representado na figura a seguir.

z


n1

1
S1

S2


n2

D 1 y
1

Usando propriedade de fluxo, temos


ZZ ZZ ZZ

→ − → −
→ − → −
→ −
F · n dS = F · n1 dS + F ·→
n2 dS .
S S1 S2

ZZ

→ −
Cálculo de F ·→
n1 dS
S1

Temos S1 : z = 1 = f (x, y), com (x, y) ∈ D : x2 + y 2 ≤ 1. Temos também que



→ −

n1 = k e dS = dxdy. Então:
ZZ ZZ

→ − →
F · n1 dS = (−x, 0, 2 · 1) · (0, 0, 1) dS
S1 S1
ZZ
= 2 dS
S1

= 2A(S)

= 2 π · 12

= 2π .

323
ZZ

→ −
Cálculo de F ·→
n2 dS
S2

Temos S2 : z = x2 + y 2 = g(x, y), com (x, y) ∈ D : x2 + y 2 ≤ 1. Um vetor




normal a S é dado por N = (−gx , −gy , 1) = (−2x, −2y, 1) que aponta para
cima. Como −→
n2 aponta para baixo, então −

n2 = p(2x, 2y,2 −1) 2 .
1 + 4x + 4y

→ p
Temos que dS = k N k dxdy = 1 + 4x2 + 4y 2 dxdy. Então:
ZZ ZZ

→ − → 
F · n2 dS = −x, 0, 2 x2 + y 2 · (2x, 2y, −1) dxdy
S2 D
ZZ

= −2x2 − 2x2 − 2y 2 dxdy
D
ZZ

= −2r2 − 2r2 cos2 θ r drdθ
Drθ
Z 2πZ 1 
= −2 1 + cos2 θ r3 drdθ
0 0
h 4 i1 Z 2π
r 
= −2 1 + cos2 θ dθ
4 0 0
h  i
1 1 sen 2θ 2π
= − θ+ θ+
2 2 2 0


= − .
2

10.
Solução:
O esboço de S está representado na figura que se segue.

324
z



n
2

S
1

D 1 2 y
1
2

x
p
Temos S : z = x2 + y 2 = f (x, y), com (x, y) ∈ D : 1 ≤ x2 + y 2 ≤ 4.


Um vetor normal a S é dado por N = (−fx , −fy , 1) = p −x
2 2
, p −y
2 2
,1 .
x +y x +y


Como −

n aponta para cima, então temos que, −

n = N

− e também temos que
r kN k
2 2 √
dS = 1 + px 2+ y 2 dxdy = 2 dxdy. Logo:
x +y

ZZ ZZ  

→ − p −x −y
F ·→

n dS = x, y, 2 x2 + y 2 · p , p , 1 dxdy
S D x2 + y 2 x2 + y 2
ZZ  
−x2 − y 2 p
= p + 2 x2 + y 2 dxdy
D x2 + y 2
ZZ
x2 + y 2
= p dxdy
D x2 + y 2
ZZ
r2
= r drdθ
Drθ r
Z 2πZ 2
= r2 drdθ
0 1
h 3 i2 Z 2π
r
= dθ
3 1 0

14π
= .
3

325
Aula 14

1.
Solução:
O esboço do sólido W está representado na figura que se segue.

z


n1

4 S1

S2



n2

D 2 y
2

Vemos que ∂W = S1 ∪ S2 , orientada positivamente. Logo,


ZZ ZZ ZZ

→ −
→ −

F · ~n dS = F · ~n1 dS + F · ~n2 dS .
S S1 S2

ZZ


Cálculo de F · ~n1 dS
S1

= 4 = f (x, y), com (x, y) ∈ D : x2 + y 2 ≤ 4, ~n1 = ~k e


Temos S1 : z q

também dS = 1 + (fx )2 + (fy )2 dx dy = 1 + 0 + 0 dx dy = dx dy. Então:
ZZ ZZ


F · ~n1 dS = (x, y, 4) · (0, 0, 1) dx dy = 4 A(D) = 4π · 22 = 16π.
S1 D

ZZ


Cálculo de F · ~n2 dS
S2

Temos S2 : z = x2 + y 2 = g(x, y), com (x, y) ∈ D : x2 + y 2 ≤ 4. Um vetor




normal a S2 é dado por N = (−gx , −gy , 1) = (−2x, −2y, 1) que está voltado
(2x, 2y, −1)
para cima. Como n~2 aponta para baixo então ~n2 = p . Temos
1 + 4x2 + 4y 2

→ p
dS = k N k dx dy = 1 + 4x2 + 4y 2 dx dy.

326
Então

ZZ ZZ

→ 
F · ~n2 dS = x, y, x2 + y 2 · (2x, 2y, −1) dx dy
S2 D
ZZ

= 2x2 + 2y 2 − x2 − y 2 dx dy
D
ZZ

= x2 + y 2 dx dy .
D

Passando para coordenadas polares, temos:

ZZ ZZ Z 2 Z 2π h 4 i2

→ 2 3 r
F · ~n2 dS = r · r dr dθ = r dθ dr = 2π = 8π .
S2 Drθ 0 0 4 0

Então:
ZZ


F · ~n dS = 16π + 8π = 24π .
S

Por outro lado:

ZZZ ZZZ ZZ Z 4


div F dV = 3 dV = 3 dz dx dy
W W D x2 +y 2
ZZ

= 3 4 − x2 − y 2 dx dy
D
ZZ

= 3 4 − r2 r dr dθ
Drθ
Z 2 Z 2π
3

= 3 4r − r dθ dr
0 0

r4 2
h i
= 6π 2r2 −
4 0

= 6π(8 − 4)

= 24π .

327
2.
Solução:
Seja S = S ∪ S1 , onde S1 : z = 0, (x, y) ∈ D : x2 + y 2 ≤ a2 com ~n1 = −~k.
z
a


n
S

S1
a y
a −

n1
x

Seja W o sólido limitado por S. Como ∂W = S está orientada positivamente e




F é de classe C 1 em R3 , podemos aplicar o teorema de Gauss em S.
Temos
ZZ ZZ ZZZ

→ −
→ −

F · ~n dS + F · ~n1 dS = div F dV
S S1 W
ZZZ
= (2 + 3 + 1) dV
W

= 6 V (W )
1 4 3
= 6· · πa
2 3

= 4πa3 .

ZZ


Cálculo de F · ~n1 dS
S1
q √
Temos dS = 1 + (zx )2 + (zy )2 dx dy = 1 + 0 + 0 dx dy = dx dy. Logo,
ZZ ZZ

→ 
F · ~n1 dS = 2x + e0 , 3y − 0, 0 − 2 · (0, 0, −1) dx dy
S1 D
ZZ
= 2 dx dy
D

= 2 A(D)

= 2πa2 .

328
ZZ


Como F · ~n dS = 2πa3 , então 2πa3 + 2πa2 = 4πa3 donde 2πa2 = 2πa3 .
S
Logo, a = 1
3.
Solução:
De x2 + y 2 + z 2 − 4z = 0, temos x2 + y 2 + (z − 2)2 = 4. Logo, o esboço de S
é a figura que se segue.
z
4

1
S

→ y
n

Seja a superfı́cie S = S ∪ S1 , onde S1 é dada por S1 : z = 1 = f (x, y),


com (x, 2 2 ~
q y) ∈ D : x + y ≤ 3 e com ~n1 = k e, também,
dS = 1 + (fx )2 + (fy )2 dx dy = dx dy.

z
4

2 −

n1

S1 1
S

→ y
n

x
Seja W o sólido limitado por S. Como estamos nas condições do teorema de
Gauss, temos:
ZZ ZZ ZZZ

→ −
→ −

rot F · ~n dS + rot F · ~n1 dS = div rot F dV .
S S1 W

329


Da Aula 7, temos que div rot F = 0. Logo,
ZZ ZZ ZZZ

→ −

rot F · ~n dS + rot F · ~n1 dS = 0 dV = 0 .
S S1 W

ZZ


Cálculo de rot F · ~n1 dS
S1

Temos

− → −
→ −


i j k


→ ∂ ∂ ∂
rot F =

∂x ∂y ∂z
2
x y + 2 x3 + y 4 2xz − 1

= 0, −2z, 3x2 − x2

= 0, −2z, 2x2 .

Logo,
ZZ ZZ

→ 
rot F · ~n1 dS = 0, −2 · 1, 2x2 · (0, 0, 1) dS
S1 S1
ZZ
= 2x2 dS
S1
Z √ Z
3 2π
3
= 2 r cos2 θ dθ dr
0 0
Z √
3 h i2π
1 sen 2θ
= 2 r3 · θ+ dr
0 2 2 0
Z √
3
= 2π r3 dr
0
h 4 i√ 3
r
= 2π
4 0


= .
2

Portanto,
ZZ

→ 9π
rot F · ~n dS = − .
S 2

330
4.
Solução:
Temos

ZZ ZZ
∂f
dS = ∇f · ~n dS .
S ∂~n S

Do teorema de Gauss, temos

ZZ ZZZ ZZZ ZZZ


∇f · ~n dS = div ∇f dV = ∇ · ∇f dV = ∇2 f dV .
S W W W

Logo,

ZZ ZZZ
∂f
dS = ∇2 f dV .
S ∂~n W

5.
Solução:
Seja S = S ∪ S1 , onde S1 é a tampa da lata. Logo, S1 é dada por S1 : z = 1,
com (x, y) ∈ D : x2 + y 2 ≤ 1 e com ~n1 = ~k e dS = dx dy.

z


n1

S1 1

S −

n

1 y
1 −

n

331
Seja W o sólido limitado por S. Pelo teorema de Gauss, temos
ZZ ZZ
∂f
dS = ∇f · ~n dS
S ∂~
n S
ZZZ
= ∇ · ∇f dV
W
ZZZ
= ∇2 f dV
W
ZZZ

= x2 + y 2 dV
W
Z 2πZ 1Z 1
= r2 · r dr dz dθ
0 0 0
Z 2πZ 1Z 1
= r3 dr dz dθ
0 0 0
h 4 i1 Z 2πZ 1
r
= dz dθ
4 0 0 0


=
4
π
= .
2
Mas ZZ ZZ ZZ
∂f ∂f ∂f
dS = dS + dS .
S ∂~n S ∂~n S1 ∂~n1

ZZ
∂f
Cálculo de dS
S1 ∂~n1

Temos
ZZ ZZ
∂f
dS = ∇f · ~n1 dS
S1 ∂~n1 S1
ZZ  
∂f ∂f ∂f
= (x, y, 1), (x, y, 1), (x, y, 1) · (0, 0, 1) dS
S1 ∂x ∂y ∂z
ZZ
∂f
= (x, y, 1) dS
S1 ∂z
ZZ
1
= dS
S1 3

1
= A(S1 )
3
1
= · π · 12
3
π
= .
3

332
Logo, ZZ
∂f π π π
dS = − = .
S ∂~n 2 3 6

6.
Solução:
O esboço da superfı́cie S : z = 2 − x2 − y 2 está representado abaixo.
z
2


n
S

x
ZZ

→ −
O cálculo direto de F ·→
n dS seria difı́cil. Então fechemos a superfı́cie S
S
com a superfı́cie S1 , o disco x2 + y 2 ≤ 1, contido no plano z = 1, orientado com

→ −

n1 = − k . Seja S = S ∪ S1 , da figura que se segue.
z
2


n
S

1 S1


n 1

Seja W a região do espaço limitada por S. Pelo Teorema de Gauss, temos:


ZZ ZZZ ZZZ

→ − → −

F · n dS = div F dxdydz = (0 + 0 + 1) dxdydz
S W W
ZZZ
= dxdydz
W

333
com

W = (x, y, z) ∈ R3 | (x, y) ∈ D , 1 ≤ z ≤ 2 − x2 − y 2

onde D é dado por D : x2 + y 2 ≤ 1. Passando para coordenadas cilı́ndricas,


temos:



 x = r cos θ



 y = r sen θ
z = z


 dxdydz = rdrdθdz



 x2 + y 2 = r 2

e Drθz é dado por




 0≤r≤1

Drθz : 0 ≤ θ ≤ 2π


 1 ≤ z ≤ 2 − r2

Então:
ZZ ZZZ

→ −
F ·→
n dS = r drdθdz
S Wrθz
Z 1 Z 2−r 2Z 2π
= r dθdzdr
0 1 0
Z 1 Z 2−r 2
= 2π r dzdr
0 1
Z 1 
= 2π r 2 − r2 − 1 dr
0
Z 1 
= 2π r 1 − r2 dr
0
Z 1 
= 2π r − r3 dr
0
h i1
r2 r4
= 2π −
2 4 0

π
=
2
ou
ZZ ZZ

→ − −
→ − π
F ·→
n dS + F ·→
n1 dS = .
S S1 2

334
ZZ

→ −
Cálculo de F ·→
n1 dS
S1

Temos S1 : z = 1, (x, y) ∈ D : x2 + y 2 ≤ 1, −

n1 = −k e dS = dxdy. Então:
ZZ ZZ

→ − −

F ·→
n1 dS = F (x, y, 1) · (0, 0, −1) dxdy
S1 D
ZZ
  
= arctg y 2 , ln x2 + 1 , 1 · (0, 0, −1) dxdy
D
ZZ
= − dxdy
D

= −A(D)

= −π · 12

= −π .

Logo,
ZZ

→ − π 3π
F ·→
n dS = + π = .
S 2 2

7.
Solução:
Seja S uma superfı́cie fechada contendo a origem.

z


n
1111111111111111111111111111111111
0000000000000000000000000000000000
0000000000000000000000000000000000
1111111111111111111111111111111111
0000000000000000000000000000000000
1111111111111111111111111111111111
0000000000000000000000000000000000
1111111111111111111111111111111111
0000000000000000000000000000000000
1111111111111111111111111111111111
0000000000000000000000000000000000
1111111111111111111111111111111111
0000000000000000000000000000000000
1111111111111111111111111111111111
0000000000000000000000000000000000
1111111111111111111111111111111111
0000000000000000000000000000000000
1111111111111111111111111111111111
S
0000000000000000000000000000000000
1111111111111111111111111111111111
0000000000000000000000000000000000
1111111111111111111111111111111111
0000000000000000000000000000000000
1111111111111111111111111111111111 y
0000000000000000000000000000000000
1111111111111111111111111111111111
0000000000000000000000000000000000
1111111111111111111111111111111111
0000000000000000000000000000000000
1111111111111111111111111111111111
0000000000000000000000000000000000
1111111111111111111111111111111111
0000000000000000000000000000000000
1111111111111111111111111111111111
0000000000000000000000000000000000
1111111111111111111111111111111111
0000000000000000000000000000000000
1111111111111111111111111111111111
x

Seja W a região sólida limitada por S. Como W não está contida no domı́nio de

→ 3
E ,ZR
Z − {(0, 0, 0)}, então não podemos aplicar o Torema de Gauss no cálculo

→ −
de E ·→ n dS. Então consideremos uma esfera S1 : x2 + y 2 + z 2 = a2 , com
S
a > 0 tal que S1 ⊂ W .

335
z


n
1111111111111111111111111111111111
0000000000000000000000000000000000
0000000000000000000000000000000000
1111111111111111111111111111111111
0000000000000000000000000000000000
1111111111111111111111111111111111
0000000000000000000000000000000000
1111111111111111111111111111111111
0000000000000000000000000000000000
1111111111111111111111111111111111
0000000000000000000000000000000000
1111111111111111111111111111111111
0000000000000000000000000000000000
1111111111111111111111111111111111


0000000000000000000000000000000000
1111111111111111111111111111111111
n1
0000000000000000000000000000000000
1111111111111111111111111111111111
S
0000000000000000000000000000000000
1111111111111111111111111111111111
0000000000000000000000000000000000
1111111111111111111111111111111111 y
0000000000000000000000000000000000
1111111111111111111111111111111111
S1
0000000000000000000000000000000000
1111111111111111111111111111111111
0000000000000000000000000000000000
1111111111111111111111111111111111
0000000000000000000000000000000000
1111111111111111111111111111111111
0000000000000000000000000000000000
1111111111111111111111111111111111
0000000000000000000000000000000000
1111111111111111111111111111111111
0000000000000000000000000000000000
1111111111111111111111111111111111
0000000000000000000000000000000000
1111111111111111111111111111111111
x


Seja W1 a região sólida limitada por S e S1 . Logo W1 ⊂ dom E . Temos
∂W1 = S ∪ S1 . Seja − →
n1 a normal a S1 apontando para o interior de S1 . Como
∂W1 está orientada positivamente, podemos aplicar o Teorema de Gauss. Temos
então, ZZ ZZZ

→ − → −

E · n dS = div E dxdydz
∂W1 W1
ou ZZ ZZ ZZZ

→ − −
→ − −

E ·→
n dS + E ·→
n1 dS = div E dxdydz .
S S1 W1


Verifique que div E = 0. Então:
ZZ ZZ ZZ

→ − → −
→ − → −

E · n dS = − E · n1 dS = E · (−−

n1 ) dS .
S S1 S1

ZZ


Cálculo de E · (−−

n1 ) dS
S1

Se −

n1 aponta para o interior de S1 então −−

n1 aponta para o exterior de S1 . Logo

→ (x, y, z)
− n1 = . Então
a
ZZ ZZ

→ −
→ εQ(x, y, z) (x, y, z)
E · (−n1 ) dS = · dS
S1
2 2 2 3/2
S1 (x +y +z ) a
ZZ
εQ x2 + y 2 + z 2
= dS
a 2 2
S1 (x + y + z )
2 3/2

ZZ
εQ a2
= dS
a S1 (a2 )3/2
ZZ
εQ
= dS
a2 S1

εQ
= A(S1 )
a2
εQ
= · 4πa2
a2

= 4πεQ .
336
Logo, ZZ

→ −
E ·→
n dS = 4πεQ .
S

8.
Solução:
A superfı́cie S não é fechada e pode ser visualizada na figura que se segue.

8


n


n
5



n

3 y
3

x

→ z
Como div F = 1 − 2 + 1 = 0, va-
mos usar o teorema de Gauss. Para
isso, é necessário fechar S através
S1
da superfı́cie S1 , porção do plano 3 y
z = 0 com x2 + y 2 ≤ 9, orientada 3

→ −

n1
com − →
n1 = − k .
x

Seja W o sólido limitado por S e S1 . Como ∂W = S ∪ S1 está orientada


positivamente, podemos aplicar o teorema de Gauss. Temos então,
ZZ ZZZ ZZZ

→− → −

F · n dS = div F dxdydz = 0 dxdydz = 0
∂W W W

ou ZZ ZZ

→− −
→−
F· →
n dS + F· →
n1 dS = 0 .
S S1

337
Mas,
ZZ ZZ ZZ

→−
F·→ x 2

n1 dS = x, −2y+e cos 0, 0+x ·(0, 0, −1) dS = − x2 dS
S1 S1 S1



onde S1 é dada por z = 0, com (x, y) ∈ D : x2 + y 2 ≤ 9, e, −

n1 = − k . Logo,
r  2
∂z ∂z 2 √
dS = 1 + + dxdy = 1 + 0 + 0 dxdy = dxdy .
∂x ∂y

Então:
ZZ ZZ

→−
F· →
n1 dS = − x2 dxdy (em coordenadas polares)
S1 D
Z 2π Z 3
= − r3 cos2 θ drdθ
0 0
Z 2π
34
= − cos2 θ dθ
4 0

34 1
= − · · 2π
4 2
81π
= − .
4

Logo, ZZ

→− 81π
F· →
n dS = .
S 4

9.
Solução:
A superfı́cie S não fechada pode ser visualizada na figura a seguir.



n
S

1
1 y

338


Como −

n tem a componente k não negativa, então −

n é exterior a S. Temos:

→ ∂2f ∂2f
div F = − +2=2
∂x∂y ∂y∂x

pois f é de classe C 2 e portanto, vale aqui o teorema de Schwartz.


Para aplicarmos o teorema de Gauss, devemos considerar o sólido W limitado


por S e S1 porção do plano z = 0, com x2 + y 2 ≤ 1, orientada com −

n1 = − k .
Temos então:
ZZ ZZ ZZZ

→− → −
→− → −

F · n dS + F · n1 dS = div F dxdydz
S S1 W

= 2V (W )

1
= 2· · π · 12 · h
3
2πh
= .
3

Mas:
ZZ ZZ  

→− ∂f ∂f
F· →
n1 dS = (x, y, 0), − (x, y, 0), 2(0 + 1) · (0, 0, −1) dS
S1 S1 ∂y ∂x
ZZ
= (−2) dS
S1

= −2A(S1 )

= −2π .

Logo, ZZ

→− 2π
F· →
n dS = (h + 3) .
S 3

10.
Solução:
A figura que se segue mostra o sólido W .

339
z

4


n
W

1


n


n
y

Como estamos nas condições do teorema de Gauss, temos:


ZZ ZZZ

→− → −

F · n dS = div F dxdydz
S=∂W W
ZZZ

= x2 + y 2 + z 2 dxdydz
W
ZZZ
= ρ2 · ρ2 sen φ dρdφdθ
Wρφθ
ZZZ
= ρ4 sen φ dρdφdθ
Wρφθ

onde
n o
π
Wρφθ = (ρ, φ, θ) ∈ R3 | 0 ≤ θ ≤ 2π , 0 ≤ φ ≤ , 1 ≤ ρ ≤ 4 cos φ .
4

Então:

340
ZZ Z 2πZ π/4Z 4 cos φ

→−
F· →
n dS = ρ4 sen φ dρdφdθ
S 0 0 1
Z 2πZ π/4 h 5 i4 cos φ
ρ
= sen φ dφdθ
0 0 5 1
Z 2πZ π/4
1 
= 45 cos5 φ sen φ − sen φ dφdθ
5 0 0
Z 2π h iπ/4
1 45
= − cos6 φ + cos φ dθ
5 0 6 0

 √ 6 √
!Z
5 5 2π
1 4 2 2 4
= − + + −1 dθ
5 6 6 2 6 0
 √ 
2π 45 7 2
= · + −1
5 6 8 2
 √ 
2π 43 · 7 2
= + −1
5 3 2
 √ 
2π 445 2
= +
5 3 2

π
 √ 
= 890 + 3 2 .
15

Aula 15

1.
Solução:
Calculemos a interseção das superfı́cies:
(
x 2 + y 2 + z 2 = a2
⇒ x2 + y 2 + (y − a)2 = a2
z = a−y
 
a 2 a2
⇒ x2 + 2 y − =
2 2

a 2
x2 y−
⇒  √ 2 + 
a 2
2
=1
a 2
2 2

  √
a a 2 a
que é uma elipse de centro 0, e semi-eixos e . Esta elipse é a projeção
2 2 2
de C sobre o plano xy. A curva C com a orientação escolhida pode ser visualizada
na figura que se segue.

341
z

a
C

a/2 a y
a
x

Considere a superfı́cie S, porção do plano z = a − y, limitada por C, que pode


ser vista na figura a seguir.

a


n
S

C = ∂S

a/2 a y
a
D
x

A projeção de S sobre o plano xy é a região D dada por



a 2
x2 y−
 √ 2 + 
a 2
2
≤ 1.
a 2
2 2

De acordo com a orientação de C = ∂S, segue que −



n aponta para cima. Então

→  

→ N −
→ ∂z ∂z (0, 1, 1)
n = − , onde N = − , − , 1 = (0, 1, 1). Logo, − →n = √

N ∂x ∂y 2

e dS = 2 dxdy. Temos:

→ −
→ −

i j k



→ ∂ ∂ ∂
rot F = = (1, 1 − 2, 1 − 3) = (1, −1, −2) .
∂x ∂y ∂z



3y + z x + 4y 2x + y

342
Do teorema de Stokes, temos:

I ZZ

→ − −
→ →
F · d→
r = rot F · −
n dS
C S
ZZ
(0, 1, 1) √
= (1, −1, −2) · √ 2 dxdy
D 2
ZZ
= (−3) dxdy
D

= −3 · A(D)

a 2 a
= −3π · ·
2 2

3 2 πa2
= − .
4

2.
Solução:
A curva com a orientação que escolhemos pode ser visualizada na figura a seguir.

C
1

1 y
1
x

p
Consideremos a superfı́cie S, porção do cone z = x2 + (y − 1)2 que se projeta
no plano xy segundo o disco D : x2 + y 2 ≤ 1 (veja a figura que se segue).

343
z



n
1

1 y
D 1
x

De acordo com a orientação escolhida para C, resulta que −



n aponta para cima.



→ N
Então n = ˛˛˛˛→
− ˛˛˛˛ onde
N

→  ∂z
−   
∂z x −(y − 1)
N = − , − , 1 = −p , p , 1 .
∂x ∂y x2 + (y − 1)2 x2 + (y − 1)2

r

→ x2 + (y − 1)2 √
Como N = 1+ 2 2
= 2 então
x + (y − 1)

 
1 −x −(y − 1)
n= √ p , p , 1
2 x2 + (y − 1)2 x2 + (y − 1)2


e dS = 2 dxdy. Temos:



→ −
→ −

i j k


→ ∂ ∂ ∂
rot F =
∂x ∂y ∂z

2
2 x z
2xy (1 − y)z + x + x +e
2

= (−1 + y, −x, 2x + 1 − 2x)

= (−1 + y, −x, 1) .

344
Do teorema de Stokes, vem:
I

→ −
F · d→
r =
C
ZZ

→ →
= rot F · −
n dS
S
ZZ  
−x −y + 1
= (−1 + y, −x, 1) · p , p , 1 dxdy
D x2 + (y − 1)2 x2 + (y − 1)2
ZZ  
x − xy + xy − x
= p +1 dxdy
D x2 + (y − 1)2
ZZ
= dxdy
D

= A(D)

= π.

3.
Solução:
Considere a superfı́cie S, porção da esfera x2 + y 2 + z 2 = a2 , limitada por C,
que pode ser vista na figura que se segue.
z

a y
a

x
Temos que S pode ser parametrizada por:

ϕ(φ, θ) = (a sen φ cos θ, a sen φ sen θ, a cos φ)


h i h i
π π
com (φ, θ) ∈ D = 0, 2 × 0, 4 . De acordo com a orientação de C, segue
(x, y, z)
que −

n é exterior à esfera. Logo, −

n = . Temos que dS = a2 sen φ dφdθ.
a

345
Do teorema de Stokes, temos:
I ZZ

→ − → −
→ →
F ·dr = rot F · −
n dS
C S
ZZ
(x, y, z)
= (x, −2y, z) · dS
S a
ZZ
1 
= x2 − 2y 2 + z 2 dS
a S
ZZ
1 
= a2 − 3y 2 dS
a S
ZZ
1 
= a2 − 3a2 sen2 φ sen2 θ a2 sen φ dφdθ
a D
ZZ

= a3 sen φ − 3 sen3 φ sen2 φ dφdθ
D
Z π/2 h   iπ/4
3 3 sen 2θ
= a θ sen φ − sen3 φ θ − dφ
0 2 2 0
Z π/2 h   i
3 π 3 π 1 3
= a sen φ − − sen φ dφ
0 4 2 4 2

cos3 φ π/2
h   i
π 3 π 1
= a3 − cos φ − − − cos φ +
4 2 4 2 3 0
   
π 3 π 1 1
= a3 − − 1−
4 2 4 2 3
 
π π 1 a3
= a3 − + = .
4 4 2 2
4.
Solução:


a) Como dom F = R3 e


→ −
→ −

i j k


→ ∂ ∂ ∂ = (0, 2x − 2x, 2y − 2y) = −

rot F = 0 ,
∂x ∂y ∂z


2xz + y 2xy + 3y e + x2
2 2 z



então pelo teorema das equivalências, segue que F é conservativo.
b) Encontremos os pontos de interseção:

 2 2
 z = 9−x −y

z = −4 ⇒ −4 = 9 − x2 − 4 ⇒ x2 = 9 ⇒ x = ±3 .


 y = 2

346
Então as extremidades da curva C são A = (−3, 2, −4) e B = (3, 2, −4).
Orientemos C de A para B. Também do teorema das equivalências segue que a
integral de linha não depende do caminho que liga A a B. Então consideremos
o segmento de reta AB, dado por z = −4 e y = 2, com −3 ≤ x ≤ 3 donde
dz = 0 e dy = 0. Então:
Z Z

→ −
F · d→
  
r = 2xz + y 2 dx + 2xy + 3y 2 dy + ez + x2 dz
C C
Z 3
= (−8x + 4) dx
−3
h i3
= − 4x2 + 4x
−3

= 24 .

5.
Solução:
O esboço da superfı́cie S está representado na figura que se segue.

z
C
1 −

n

Como − →
n tem componente z positivo, então − →
n aponta para o interior de S.
Logo, a orientação de C = ∂S é no sentido anti-horário quando vista de cima.
Use a regra da mão direita para se convencer.
Pelo Teorema de Stokes, temos
ZZ I

→ → −
→ −
rot F · −
n dS = F · d→
r
S C

onde C é dada por C : z = 1, x2 + y 2 = 1. Logo, uma parametrização de C é

347
dada por C : γ(t) = (cos t, sen t, 1), com 0 ≤ t ≤ 2π. Temos:
I Z 2π

→ − −

F · d→

r = F γ(t) · γ ′ (t) dt
C 0
Z 2π
= (sen t, 0, 0) · (− sen t, cos t, 0) dt
0
Z 2π
= − sen2 t dt
0
h i2π
1 sen 2t
= − t−
2 2 0

= −π .

Logo, ZZ

→ →
rot F · −
n dS = −π .
S

6.
Solução:
Da descrição de S, temos

x = r cos θ (1)
y = r sen θ (2)
z=r (3)

com
0 ≤ r ≤ 1 e 0 ≤ θ ≤ 2π (4)
p
De (1), (2), (3) e (4), temos que S é um cone dado por S : z = x2 + y 2 ,
com 0 ≤ z ≤ 1.

S


n
y

348
Note que o bordo de S, ∂S, é a circunferência x2 + y 2 = 1, contida no plano
z = 1. Como −→n é exterior a S então o bordo ∂S está orientado no sentido horário
quando vista de cima. Parametrizando ∂S − , temos ∂S − : γ(t) = (cos t, sen t, 1),
com 0 ≤ t ≤ 2π donde γ ′ (t) = (− sen t, cos t, 0). Pelo Teorema de Stokes, temos
ZZ I

→ → −
→ −
rot F · −
n dS = F · d→
r
S ∂S
Z

→ −
= − F · d→
r
∂S −
Z 2π

→ 
= − F γ(t) · γ ′ (t) dt
0
Z 2π 
= − cos2 t sen t , 2 sen3 t , 3 · (− sen t, cos t, 0) dt
0
Z 2π 
= − − cos2 t sen2 t + 2 sen3 t cos t dt
0
Z 2π Z 2π
1 2 2
= 4 cos t sen t dt − 2 sen3 t cos t dt
4 0 0
Z 2π Z 2π
1
= sen2 2t dt − 2 sen3 t cos t dt
4 0 0
Z 2π Z 2π
1 2
= sen 2t d(2t) − 2 sen3 t d(sen t)
8 0 0

1 1
h
sen 4t
i2π h sen4 t i2π
= · 2t − −2
8 2 2 0 4 0

π
= .
4

7.
Solução:


Calcular a integral diretamente é uma tarefa difı́cil. Então calculemos o rot F .
Temos:


→ −
→ −


i j k



rot F = ∂/∂x ∂/∂y ∂/∂z


−y −x −z −y −x −z
e − ze e − xe e − ye

= − e−z + e−z , −e−x + e−x , −e−y + e−y

= (0, 0, 0)



= 0 .
349

→ −

Como dom F = R3 então, pelo Teorema das Equivalências, segue que F é um
campo gradiente, isto é, existe um campo escalar ϕ(x, y, z), tal que:


∂ϕ

 = e−y − ze−x (1)


 ∂x



→ 3 ∂ϕ
∇ϕ = F em R ⇔ = e−z − xe−y (2)

 ∂y



 ∂ϕ = e−x − ye−z

(3)
∂z

Integrando (1), (2), (3) em relação a x, y e z, respectivamente, temos:

ϕ(x, y, z) = xe−y + ze−x + A(y, z)


ϕ(x, y, z) = ye−z + xe−y + B(x, z)
ϕ(x, y, z) = ze−x + ye−z + C(x, y)

Por inspeção temos que:

A(y, z) = ye−z
B(x, z) = ze−x
C(x, y) = xe−y .

Logo, ϕ(x, y, z) = xe−y + ze−x + ye−z . Então, temos que

Z

→ −
F · d→
 
r = ϕ γ(1) − ϕ γ(0)
C

onde:
 
ln 2 π 1−e
γ(1) = , sen , = (1, 1, 1)
ln 2 2 1−e
 
ln 1 1 − e0
γ(0) = , sen 0 , = (0, 0, 0) .
ln 2 1−e

Logo,

Z

→ −
F · d→

r = ϕ(1, 1, 1) − ϕ(0, 0, 0) = e−1 + e−1 + e−1 − (0 + 0 + 0) = 3e−1 .
C

350
8.
Solução:
O esboço de C está representado a seguir.

A 1

x 5
y


z = 5 − y2

Resolvendo o sistema z = 1 otemos A = (4, −2, 1) e

x+z = 5
B = (4, 2, 1). Temos:



→ −
→ −


i j k



= (x − x, y − y, z − z) = −

rot F = ∂/∂x ∂/∂y ∂/∂z 0 .


yz + x3 xz + 3y 2 xy + 4


→ −

Como dom F = R3 então, pelo Teorema das Equivalências, F é um campo


gradiente, isto é, existe ϕ(x, y, z) tal que ∇ϕ = F em R3 ou


∂ϕ
= yz + x3


 (1)

 ∂x


∂ϕ
= xz + 3y 2 (2)

 ∂y


 ∂ϕ = xy + 4


(3)
∂z

351
Integrando (1), (2), (3) em relação a x, y e z, respectivamente, temos:


x4

 ϕ(x, y, z) = xyz + + A(y, z)


 4

ϕ(x, y, z) = xyz + y 3 + B(x, z)





 ϕ(x, y, z) = xyz + 4z + C(x, y)

Por inspeção, vemos que

A(y, z) = y 3 + 4z
x4
B(x, z) = + 4z
4
x4
C(x, y) = + y3 .
4

Logo, temos que


x4
ϕ(x, y, z) = xyz + + y 3 + 4z
4



é uma função potencial de F . Assim,

Z

→ −
F · d→
r = ϕ(B) − ϕ(A)
C

= ϕ(4, 2, 1) − ϕ(4, −2, 1)

= (8 + 4 + 8 + 4) − (−8 + 4 − 8 + 4)

= 24 + 8

= 32 .

9.
Solução:
Esboçando o cilindro parabólico z = y 2 e o plano x+z = 1, vemos que os pontos
A1 , A2 e A3 são comuns às duas superfı́cies. Ligando-os, temos um esboço de
C na figura que se segue.

352
z
A2
1
A3

C
1
y
A1

Z

→ −
Observe que calcular F · d→
r pela definição é uma tarefa extremamente com-
C
plicada. Temos:



→ −
→ −

i j k


→ ∂ ∂ ∂ = 1, −3x2 , 2 6= −
 →
rot F = 0 .
∂x ∂y ∂z


−2y + esen x −z + y x3 + esen z



Logo, F não é conservativo. Para aplicar o teorema de Stokes, devemos fechar
C utilizando o segmento de reta C1 que liga A3 a A2 .

z
A2 C1
1
A3


n

S
C

1
y
A1

Seja S a porção do plano x + z = 1, limitada por C = C ∪ C1 e que se projeta


no plano yz segundo a região D cujo esboço se segue.

353
z
z=1
1

D
z = y2

−1 1 y

Descrevemos S por S : x = 1 − z = f (y, z), com (y, z) ∈ D : −1 ≤ y ≤ 1


e y 2 ≤ z ≤ 1. Considerando a orientação de C = ∂S, segue que a normal a S


está voltada para cima. Um vetor normal a S é N = (1, −fy , −fz ) = (1, 0, 1).
(1, 0, 1) √
Logo, −→n = √ e dS = 2 dydz. Pelo teorema de Stokes, temos:
2

Z ZZ

→ − −
→ →
F · d→
r = rot F · −
n dS
C S
ZZ
= (1, −3(1 − z)2 , 2) · (1, 0, 1)dydz
D
ZZ
= (1 + 2)dydz
D
ZZ
= 3 dydz
D
Z 1Z 1
= 3 dzdy
−1 y 2
Z 1 
= 3 1 − y 2 dy
−1
h i1
y3
= 3 y−
3 −1
 
1
= 6 1−
3

= 4.

ou
Z Z

→ − −
→ −
F · d→
r + F · d→
r = 4.
C C1

354
Z

→ −
Cálculo de F · d→
r
C1

Temos C1− : z = 1, com x = 0 e −1 ≤ y ≤ 1 donde dz = dx = 0. Então:

Z Z Z

→ − −
→ −
F · d→
r =− F · d→
r = − Q(0, y, 1)dy
C1 C1− C1−
Z 1
= − (−1 + y)dy
−1

y2 1
h i
= − −y +
2 −1

= 2.

Logo,
Z

→ −
F · d→
r = 4 − 2 = 2.
C

10.
Solução:
Da parametrização de C, temos x = 4 cos t, y = 4 sen t e z = 4 − 4 cos t, com
0 ≤ t ≤ 2π donde x2 + y 2 = 16 e z = 4 − x. Logo, C é a curva interseção do
cilindro x2 + y 2 = 16 com o plano z = 4 − x, orientada no sentido anti-horário
quando vista de cima.

4
x 4 y

355
Seja S a porção do plano z = 4 − x, limitada por C.



n
C

S 4

4
4 y
x

Da regra da mão direita, vemos que − →n aponta para cima. A superfı́cie S pode
ser descrita por S : z = 4 − x = f (x, y), com (x, y) ∈ D : x2 + y 2 ≤ 16. Temos

→ (1, 0, 1) √
N = (−fx , −fy , 1) = (1, 0, 1), donde −→n = √ e dS = 2 dxdy. Temos:
2


− → −
→ −

i j k


→ ∂ ∂ ∂
rot F = = (1, 1, 1) .
∂x ∂y ∂z


z − y ln(1 + y 2 ) ln(1 + z 2 ) + y

Logo, do teorema de Stokes, temos

Z ZZ ZZ

→ − −
→ →
F · d→
r = rot F · −
n dS = (1, 1, 1) · (1, 0, 1)dxdy
C S D
ZZ
= (1 + 1)dxdy
D

= 2 A(D)

= 2 · π · 42

= 32π .

356
Aula 16

1.
Solução:
y
Como C está orientada no sentido horário não
2
podemos aplicar o teorema de Green. Então C−
denotemos por C − a circunferência orientada
D 1
no sentido anti-horário. Seja D a região deli-
mitada pela circunferência.
x

Temos então:
I ZZ   ZZ ZZ

→ −→ ∂Q ∂P
F·d r = − dxdy = (3x+3)dxdy = 3 (x+1) dxdy .
C− D ∂x ∂y D D

Passando para coordenadas polares:

x2 + y 2 = 2y ⇒ r2 = 2r sen θ ⇒ r = 2 sen θ .

Então: I Z πZ 2 sen θ

→ −
F · d→
r = 3 (r cos θ + 1)r drdθ
C− 0 0
Z πZ 2 sen θ 
= 3 r2 cos θ + r drdθ
0 0
Z πh 3
r2 2 sen θ
i
r
= 3 cos θ + dθ
0 3 2 0
Z π  
8
= 3 sen3 θ cos θ + 2 sen2 θ dθ
0 3
h   iπ
8 sen4 θ 1 sen 2θ
= 3 · +2· θ−
3 4 2 2 0

= 3(0 + π)

= 3π .
Mas, I I

→ − −
→ −
F · d→
r =− F · d→
r .
C C−
Logo, I

→ −
F · d→
r = −3π .
C

357
2.
Solução:

→ 
a) Seja F = (P, Q) = sen xy + xy cos xy, x2 cos xy , para todo (x, y) ∈ R2 .
Temos:
∂Q ∂P  
− = 2x cos xy − x2 y sen xy − x cos xy + x cos xy − x2 y sen xy
∂x ∂y

= 0

em R2 . Como R2 é um conjunto simplesmente conexo então, pelo teorema das


equivalências segue que I é independente do caminho.


b) Também do teorema das equivalências temos que F é conservativo.


Construção de uma função potencial para F

Temos:
∂ϕ
= sen xy + xy cos xy (1)
∂x
∂ϕ
= x2 cos xy (2)
∂y
Integrando (2) em relação a y, temos:

ϕ(x, y) = x sen xy + f (x) (3)

onde f (x) é uma “constante” de integração. Derivando (3) em relação a x e


usando (1), temos:

sen xy + xy cos xy + f ′ (x) = sen xy + xy cos xy

ou f ′ (x) = 0. Logo, f (x) = c. Fazendo c = 0, temos uma função potencial

ϕ(x, y) = x sen xy ,

em R2 . Logo,
 
I = ϕ σ(1) − ϕ σ(0)

= ϕ(0, 1) − ϕ(−1, 1)

= 0 − (−1) sen(−1)

= sen(−1)

= − sen 1 .

358
3.
Solução:
Temos que
ZZ ZZ
M= f (x, y, z) dS = x2 dS ,
S S

onde S está ilustrada na figura que se segue.

S
3

2
2 y
3

Uma parametrização para S é dada por ϕ(t, z) = (2 cos t, 2 sen t, z) onde



 0 ≤ t ≤ 2π
(t, z) ∈ D :
 0 ≤ z ≤ 3 − 2 cos t

∂ϕ ∂ϕ
Temos que = (−2 sen t, 2 cos t, 0) e = (0, 0, 1) donde
∂t ∂z


→ −
→ →


i j k

∂ϕ ∂ϕ
× = −2 sen t 2 cos t 0 = (2 cos t, 2 sen t, 0) .
∂t ∂z

0 0 1

359

∂ϕ ∂ϕ
Como dS = × dtdz então dS = 2 dtdz. Logo,
∂t ∂z
ZZ
M = (2 cos t)2 · 2 dtdz
D
Z 2π Z 3−2 cos t
= 8 cos2 t dzdt
0 0
Z 2π
= 8 (3 cos2 t − 2 cos3 t) dt
0
Z 2π Z 2π
2
= 24 cos t dt − 16 cos3 t dt .
0 0

Temos: Z 2π h i2π
1 sen 2t
cos2 t dt = t+ =π
0 2 2 0

e Z Z
2π 2π
3
cos t dt = cos2 t · cos t dt
0 0
Z 2π
= (1 − sen2 t) d(sen t)
0
h i2π
sen3 t
= sen t −
3 0

= 0.

Logo,
M = 24π u.m.

4.
Solução:
A curva C é constituı́da de três partes e pode ser visualizada na figura a seguir.

1
1 y
x

360
Calcular I usando a definição é uma tarefa penosa. Calculemos então através
do teorema de Stokes. Seja S a porção do plano x + y + z = 1 delimitada pela
curva C. De acordo com a orientação de C, devemos tomar − →
n apontando para
cima. As figuras que se seguem mostram S e sua projeção sobre o plano xy.

z
y
1


n 1

C = ∂S
S x+y =1

D
1
1 y
x 1 x

Temos S : z =
 1−x−y = f (x,  y), com (x, y) ∈ D : 0 ≤ x ≤ 1 , 0 ≤ y ≤ 1−x.

→ ∂f ∂f
Como N = − , − , 1 = (1, 1, 1) aponta para cima então temos que
∂x ∂y

→ √

→ N (1, 1, 1)
n = −

= √ e dS = 3 dxdy. Por outro lado
N 3


→ −
→ −


i j k


− ∂ ∂ ∂


rot F = = (2z, 2x, −2y) .
∂x ∂y ∂z

x2
e + y 2 ey2 − z 2 ez2 − x2

361
Do teorema de Stokes, temos:
I ZZ

→ − → −
→→
F· d r = rot F · −
n dS
C S
ZZ
 (1, 1, 1) √
= 2(1 − x − y), 2x, −2y · √ · 3 dxdy
D 3
ZZ
= (2 − 2x − 2y + 2x − 2y) dxdy
D
ZZ
= 2 (1 − 2y) dxdy
D
Z 1Z 1−x
= 2 (1 − 2y) dydx
0 0
Z 1 h i1−x
= 2 y − y2 dx
0 0
Z 1 
= 2 1 − x − 1 + 2x − x2 dx
0
Z 1 
= 2 x − x2 dx
0
h i1
x2 x3
= 2 −
2 3 0

1
= .
3

5.
Solução:
Note na figura que se segue que a superfı́cie S não é fechada.

362
z



n



Da complexidade do campo F , calcular o fluxo diretamente não é uma tarefa
simples. Então tentemos usar o teorema de Gauss. Para isso, devemos fechar
a superfı́cie S com a porção S1 do plano z = 1, delimitada pela circunferência


x2 + y 2 = 3, orientada com − →
n1 = − k . A figura abaixo mostra o sólido W
delimitado por S e S1 .



n

S1 1


n1

D √

3
3 y

363
Como estamos nas condições do teorema de Gauss, temos:
ZZ ZZ ZZZ

→− → −
→− → −

F · n dS + F · n1 dS = div F dxdydz
S S1 W
ZZZ
= (0 + 0 + 1) dxdydz
W
ZZZ
= dxdydz
W
Z Z "Z 4−x2 −y 2
#
= dz dxdy
D 1
ZZ

= 3 − x2 − y 2 dxdy
D
Z √
2πZ 3 
= 3 − r2 r drdθ
0 0
Z √
2πZ 3 
= 3r − r3 drdθ
0 0
Z 2π h i√ 3
r2 r4
= 3· − dθ
0 2 4 0
 
9 9
= − · 2π
2 4

= .
2

Mas
ZZ ZZ

→−
F· →

n1 dS = e arctg 1, e ln(x2 + 1), 1 · (0, 0, −1) dS
S1 S1
ZZ
= − dS
S1

= −A(S1 )
√ 2
= −π 3

= −3π .

Logo, ZZ

→− 9π 15π
F· →
n dS = + 3π = .
S 2 2

364
6.
Solução:
O cálculo direto da integral parece uma tarefa impossı́vel, mas o Teorema de
x2 y
Green fornece um outro método de resolução. Temos P = − + y2
1 + x2
e Q = x + arctg x donde
 
∂Q ∂P 1 x2
− = 1+ 2
− − 2
+ 2y
∂x ∂y 1+x 1+x
1 + x2
= 1+ − 2y
1 + x2

= 2 − 2y

= 2(1 − y) .

Então: ZZ   ZZ
∂Q ∂P
I= − dxdy = 2 (1 − y)dxdy
D ∂x ∂y D
onde D é a região do plano limitada por C, conforme a figura que se segue.

2 x = 4 − 2y

x=0
D C = ∂D

4 x

A região D pode ser vista como tipo II:



D = (x, y) | 0 ≤ y ≤ 2 , 0 ≤ x ≤ 4 − 2y .

Logo, Z Z
2 4−2y
I = 2 (1 − y) dxdy
0 0
Z 2
= 2 (1 − y)(4 − 2y) dy
0
Z 2 
= 4 2 − 3y + y 2 dy
0
h i2
3y 2 y3
= 4 2y − +
2 3 0

8
= .
3

365
7.
Solução:
a) Considerando-se que x2 + y 2 = 2x é equivalente a (x − 1)2 + y 2 = 1, então
o esboço de S está representado na figura que se segue.

z z

4 4

S S

y y
−1 −1
2 2
x x

Seja (x, y, z) ∈ S. Então x e y satisfazem (x − 1)2 + y 2 = 1. Logo




x = 1 + cos t
com 0 ≤ t ≤ 2π .
 y = sen t

Como −1 ≤ z ≤ x2 + y 2 = 2x então −1 ≤ z ≤ 2 + 2 cos t.


Adotando t e z como parâmetros, temos que uma parametrização para S é dada
por ϕ(t, z) = (1 + cos t, sen t, z) com

 0 ≤ t ≤ 2π
(t, z) ∈ D :
 −1 ≤ z ≤ 2 + 2 cos t

b) Temos

ϕt = (− sen t, cos t, 0)
ϕz = (0, 0, 1)

366
donde −
→ −
→ − →
i j k


ϕt × ϕz = − sen t cos t 0 = (cos t, sen t, 0) .


0 0 1
ZZ
Logo, kϕt × ϕz k = 1. Como A(S) = kϕt × ϕz k dtdz então
D
ZZ
A(S) = dtdz
D
Z 2π Z 2+2 cos t
= dzdt
0 −1
Z 2π
= (3 + 2 cos t) dt
0
h i2π
= 3t + 2 sen t
0

= 6π u.a.

c) O fluxo é dado por: ZZ



→ −
Φ= F ·→
n dS
S

onde −

n = (− cos t, − sen t, 0) pois −

n aponta para dentro de S.
Como dS = kϕt × ϕz k dtdz então dS = dtdz. Logo:
ZZ


Φ= F (ϕ(t, z)) · −

n dtdz =
D
ZZ
= (3 + 3 cos t − 3 − sen t , 3 sen t + 1 + cos t − 1 , esen z ) · (− cos t, − sen t, 0) dtdz
D
ZZ
= (3 cos t − sen t, 3 sen t + cos t, esen z ) · (− cos t, − sen t, 0) dtdz
D
ZZ

= −3 cos2 t + sen t cos t − 3 sen2 t − sen t cos t dtdz
D
ZZ
= − 3 dtdz
D
ZZ
= −3 dtdz
D
ZZ
Como dtdz = 6π (ver item (a)) então
D

Φ = −18π .

367
8.
Solução:
O esboço de C está representado na figura que se segue.

z z

4 4

C C

1 2 y 1 2 y

x x

Considerando o sistema

 z = x2 + y 2
 x2 + (y − 1)2 = 1 ou x2 + y 2 = 2y

temos z = 2y. Isto significa que a curva interseção C está contida no plano
z = 2y. Então, seja S a superfı́cie porção do plano z = 2y, limitada por C.
Logo, ∂S = C. Temos
S : z = 2y = f (x, y)

com
(x, y) ∈ D : x2 + (y − 1)2 ≤ 1 ou x2 + y 2 ≤ 2y .

O esboço de D está representado na figura que se segue.

D
1

368
q √ √
Como dS = 1 + (zx )2 + (zy )2 dxdy então dS = 1 + 0 + 22 dxdy = 5 dxdy.
De acordo com a orientação de C = ∂S, devemos tomar −

n apontando para cima.
Então



→ N (−zx , −zy , 1) (0, −2, 1)
n = −
→ = −→ = √ .
N N 5

Temos que:


−→ −
→ −

i j k




rot F = ∂/∂x ∂/∂y ∂/∂z = (0, 0, x − 2) .



x2 √

2y 1 + z8
2

Pelo Teorema de Stokes, temos:

I ZZ

→ − −
→ →
F · d→
r = rot F · −
n dS
C S
ZZ
(0, −2, 1) √
= (0, 0, x − 2) · √ · 5 dxdy
D 5
ZZ
= (x − 2) dxdy .
D

Passando para coordenadas polares, temos:




 x = r cos θ


y = r sen θ

 dxdy = rdrdθ



x + y 2 = 2y ⇒ r2 = 2r sen θ ⇒ r = 0 ou r = 2 sen θ
2

Do esboço de D, temos que Drθ é dado por



0≤θ≤π
Drθ :
 0 ≤ r ≤ 2 sen θ

369
Então: I ZZ

→ −
F · d→
r = (r cos θ − 2)r drdθ
C Drθ
ZZ

= r2 cos θ − 2r drdθ
Drθ
Z πZ 2 sen θ 
= r2 cos θ − 2r drdθ
0 0
Z πh i2 sen θ
r3
= cos θ − r2 dθ
0 3 0
Z π  
8
= cos θ sen3 θ − 4 sen2 θ dθ
0 3
h  iπ
8 sen4 θ 4 sen 2θ
= · − θ−
3 4 2 2 0

= −2π .

9.
Solução:
a) Temos

→ −
→ −


i j k



∂ ∂ ∂
rot F =
∂x ∂y ∂z

2
3x yz + ez x3 z x3 y + xez + 3z 2


= x3 − x3 , 3x2 y + ez − ez − 3x2 y, 3x2 z − 3x2 z


= (0, 0, 0) = 0 .


→ −
→ −

Como dom F = R3 , que é um conjunto simplesmente conexo e rot F = 0 ,


então pelo teorema das equivalências, segue que F é conservativo.

b) O esboço de C está representado na figura que se segue.

370
z

A = (−1, 1, 0)

1
1
y
x B = (1, 1, 0)

O trabalho W é dado por:


Z

→ −
W = F · d→
r .
C


→ −

Como F é conservativo, logo existe uma função ϕ(x, y, z), tal que ∇ϕ = F
em R3 , isto é,
∂ϕ
= 3x2 yz + ez (1)
∂x
∂ϕ
= x3 z (2)
∂y
∂ϕ
= x3 y + xez + 3z 2 (3)
∂z

Integrando (1), (2) e (3) em relação a x, y e z respectivamente, temos

ϕ(x, y, z) = x3 yz + xez + f (y, z) (4)

ϕ(x, y, z) = x3 yz + g(x, z) (5)

ϕ(x, y, z) = x3 yz + xez + z 3 + h(x, y) (6)

De (4), (5) e (6) vemos que f (y, z) = z 3 , g(x, z) = xez e h(x, y) = 0. Logo

ϕ(x, y, z) = x3 yz + xez + z 3


é uma função potencial de F . Então:

W = ϕ(B) − ϕ(A) = ϕ(1, 1, 0) − ϕ(−1, 1, 0) = e0 − −e0 = 2 u.ω .

371
10.
Solução:
O esboço de S está representado na figura que se segue.

z
∂S

S


n
y

Como − →n aponta para baixo, então o bordo de S, ∂S, está orientado no sentido
horário quando visto de cima. Parametrizando ∂S, no sentido anti-horário, temos
∂S − : x cos t, y = sen t e z = 1, com 0 ≤ t ≤ 2π. Logo, dx = − sen t dt,
dy = cos t dt e dz = 0. Do teorema de Stokes, temos:
ZZ Z

→ − → −
→ −
rot F · n dS = F · d→r
S ∂S
Z

→ −
= − F · d→
r
∂S −
Z
= − P (x, y, 1) dx + Q(x, y, 1) dy + R(x, y, 1) · 0
∂S −
Z
= − ye1−1 dx − xe1−1 dy
∂S −
Z
= − y dx − x dy
∂S −
Z 2π  
= − (sen t)(− sen t) − (cos t)(cos t) dt
0
Z 2π 
= − − sen2 t − cos2 t dt
0
Z 2π
= dt
0

= 2π .

372

You might also like